CIC Exam Study Cards 2022

Réussis tes devoirs et examens dès maintenant avec Quizwiz!

Isolation Initiation and Duration: CRE (Carbapenem-resistant Enterobacteriaceae) *CP (Carbapenemase) testing pending

Isolation Type: Enhanced Contact (Until CP-CRE r/o by IP) Duration: Patient to remain in precautions. Reevaluate when CP results are known.

Obligate Anaerobes

only anaerobic growth; ceases in presence of oxygen NO OXYGEN

Diplococcus

pair of cocci. The diplococcus (plural diplococci) is a round bacterium (coccus) that typically occurs in the form of two joined cells. Examples of gram-negative diplococci: Neisseria sp., Moraxella catarrhalis, Acinetobacter, and Brucella

Six Sigma

Minimize process variation which produces defects.

Which of the following is not a component of a strategic plan? A. Brainstorming B. Organizational analysis C. Forming conclusions D. Action planning

A. Brainstorming

Odds ratio

AD/BC

What are the 5 vaccines that are CONTRAINDICATED for HCP during pregnancy?

1. HPV 2. Influenza (LAIV) 3. MMR 4. Varicella 5. Zoster

True or False The CDC does not recommend sampling for airborne microbial contaminants before, during, or after construction projects.

True

If the CIC processional wants to utilize a visual representation that clearly shows which problems are of most concern, the best choice is: A. Run chart B. Ishikawa diagram C. Pareto chart D. Flow Chart

C. Pareto chart

Foodborne illness: Salmonella Spp.

Incubation Period = 6-48 hours Signs & Symptoms = Diarrhea, fever, abdominal cramps, vomiting Duration = 4-7 days Food Sources = Eggs, poultry, meat, unpasteurized milk or juice, cheese, contaminated raw fruits and vegetables

Which Influenza strain is associated with annual epidemic diseases?

Influenza B - however, the clinical illness tends to be milder than illnesses caused by Influenza A.

Wet mount specimens and pathogens

Specimens: Sputum, drainage from leasons, body fluid aspirates, stool, vaginal discharge and urine sediment. Pathogens: Motile trophozoites of Giardia lamblia in stool, Trichomonas vaginalis in vaginal discharge or urine sediment, and Entamoeba histolytica from liver abscess aspirate.

Sterilizaion

process that destroys all microorganisms both pathogenic and non-pathogenic including spores and viruses

Attributable Risk (AR)

= incidence exposed - incidence unexposed = (a/(a+b)) - (c/(c+d))

Odds Ratio (OR)

=ad/bc It is used for all types of studies with nominal data, but it is mostly used for retrospective and cross-sectional studies. The odds ratio is sometimes called the "Cross-product ratio" or "relative odds"

Outcome Measure

A measure that indicates the result of the performance (or nonperformance) of a function or process. Examples: HAIs, influenza or TB skin test conversions, patient falls, sharps injuries

Gap Analysis

A technique used to compare best practices with the current process and determine the steps to take to move from a current state to a desired FUTURE STATE. What is -> What should be -> Identifying "gaps" "use your inner eye to see the future" - Hermione Granger

What are 7 common risk factors for MRSA infection among patients in healthcare settings?

1. Prior exposure to antimicrobial agents 2. Prolonged hospital stay 3. Stay in ICU or Burn Unit 4. Proximity to patients colonized or infected with MRSA 5. Use of invasive devices 6. Surgical Procedures 7. Underlying illnesses

The agency responsible for prevention of dialysis-associated disease and guidance for equipment disinfection is: A. CDC, DHQP B. FDA C. Health Resources and Services Administration (HRSA) D. Environmental Protection Agency

A. CDC, DHQP

Legitimate power is based upon the perception that the leader has a right to exercise influence because of the leader's: A. Title or position within the organization B. Information, knowledge, or expertise C. Ability to reward or punish D. Personal characteristics and personality

A. Title or position within the organization Rationale: Power is an integral part of management and leadership. The five main types of power include: 1. Coercive 2. Expert 3. Legitimate 4. Referent 5. Reward Legitimate power refers to the ability to administer to another certain feelings of obligation or the notion of responsibility. References: APIC Text Online, Chapter 1 - Infection Prevention and Control Programs; Chapter 3 - Education and Training

A patient comes into the emergency department complaining of pain in the genital Area with a history of unprotected sexual contact. The nurse practitioner wants to rule out trichomonas vaginalis. What type of test will be ordered? A. Wet mount B. Gram stain C. Blood culture D. Cytological exam

A. Wet mount Specimens routinely collected for wet mount include sputum Viewed for fungal elements; stool examined for larvae, adult worms, ovum, cysts, or parasites; cerebral spinal fluid for cryptococcus neoformans; vaginal secretions for trichomonas vaginalis; and urine sediment for white blood cells, bacteria, yeast, and parasites (T. vaginalis).

Bloom's Taxonomy: Evaluating

Appraise, argue, defend, judge, select, support, value, evaluate. Example: "The Morphe eyeshadows are a better quality and value than the Tarte eyeshadows"

Bloom's Taxonomy: Analyzing

Appraise, compare, contrast, criticize, differentiate, discriminate, distinguish, examine, experiment, question, test Example: "This is an eyeshadow and this is a pressed pigment"

The average risk for transmission of HIV by the exposure type is (select ALL that Apply): A. 1.8% for Sexual Contact B. 0.3% for percutaneous exposure C. 0.1% for mucous membrane contact D. Less than 0.1% for nonintact skin contact

B. 0.3% for percutaneous exposure C. 0.1% for mucous membrane contact D. Less than 0.1% for nonintact skin contact 1.8% risk of seroconversion for Hepatitis C with a percutaneous exposure APIC chapter - Occupational Health

systematic review and meta-analysis

Collects all previous studies on the topic and statistically combined their results

Enrichment medium growth media

Contains special nutrients necessary for growth of fastidious bacteria hard to grow bacteria Chocolate agar utilized for the growth of Neisseria meningitidis

Urinary tract infections in the postpartum period have three important risk factors. Which of the following is not one of them: A. Induction of labor B. Tocolysis C. Cesarean delivery D. Twin births

D. Twin births Rationale: Urinary tract infections (UTIs) in the postpartum period have three important risk factors: cesarean delivery, tocolysis, and induction of labor. Twin births are not associated with an increased risk of UTI. Reference: APIC Text, 4th edition, Chapter 33 - Urinary Tract Infection CBIC Core Competency: Preventing/Controlling the Transmission of Infectious Agents

Discrete data

Discrete data contains whole numbers and are mutually exclusive. Examples: infected or not infected, male or female, and blood type

True or False OSHA's BBP standard provides directives for employers to develop an exposure plan that included providing the Hepatitis B vaccine to employees within 21 days of employment.

False - Hepatitis B vaccine need to be available to employees within 10 days of their first assignment.

If cleaning is done manually, the two essential components are___________ and _______________.

Friction and fluidics

5.25% sodium hypochlorite

Household bleach disinfectant. It has a broad spectrum of antimicrobial activity. It is effective against hepatitis B virus and HIV. It is a fast acting disinfectant.

By what mechanism do antibacterial work?

Interfere with cell wall biosynthesis, inhibit bacterial ribosomes, interfere with DNA replication or RNA transcription, or inhibit metabolic pathways.

What are the two levels of measurement for Numerical Data?

Interval and Ratio

Cleaning-disinfection

Is a one-step process, unless the surface is heavily contaminated

Isolation Initiation and Duration: Shingles (Varicella Zoster) Immunosuppressed or disseminated

Isolation Type: Airborne Contact Duration: Disseminated shingles is defined as lesions on 3 or more contiguous dermatomes. Remain in precautions until lesions dry and crusted. No susceptible caregivers should enter the room.

Isolation Initiation and Duration: C diff

Isolation Type: Contact Enteric Duration: 30 days from positive lab AND asymptomatic. If patient hospital duration is greater than 30 days from the last positive, then consult with infection prevention team on discontinuation of isolation and care plan.

Isolation Initiation and Duration: Norovirus

Isolation Type: Contact Enteric Duration: Until 48 hours of no symptoms (vomiting/diarrhea) and until environment has been terminally cleaned

Normal distribution

Normal distribution is when the values on both sides of the main or even. It is a bell shaped curve. The mean, median, and mode are all equal. One standard deviation is 68.2%. Two standard deviations is 95.5%. And three standard deviations is 99.7%

Microaerophiles

Only aerobic growth; oxygen required in low concentration

Aerotolerant Anaerobes

Only anaerobic growth; but continues in presence of oxygen

Qualitative Research Examples

Participant observation, Focus groups, interviews, field notes, and tape recordings or transcriptions.

Beta-lactam drugs

Possess bactericidal activity by inhibiting cell wall synthesis Examples: Penicillin, cephalosporins, monobactams, carbapenems

What is the most common type of CJD?

Sporadic - 85-95% of all cases

SWOT Analysis

Strengths, Weaknesses, Opportunities, Threats Utilized to investigate public health issues and improve healthcare outcomes. Conducting this type of analysis points out what the organization should plan for, and how to use resources and guide efforts within a formal framework.

Which subgroup of RSV is more prominent?

Subgroup A

What are common reservoirs for Influenza A?

Swine and poultry

Negative Predictive Value (NPV)

TN / (TN + FN)

What is the proper disinfection process for tonometer tips and what is the CDC Category Ranking?

Wipe clean tonometer tips and then disinfect them by immersing for 5-10 minutes in either 5000 ppm chlorine or 70% ethyl alcohol. None of these listed disinfectant products are FDA-cleared high-level disinfectants. Category II.

Pareto Charts

a graphic way of classifying problems by their level of importance, often referred to as the 80-20 rule. What is the biggest bang for your buck? Focus on the vital few that will give you the most benefits

Failure Mode and Effects Analysis (FMEA)

a systematic process for identifying potential design and process failures before they occur, with the intent to eliminate them or minimize the risk associated with them Primary prevention.

An IP has monitored the rate of hand hygiene compliance among different nursing units. She finds that the ICU staff's compliance is less than satisfactory even after providing conveniently located hand hygiene dispensers. She shares this feedback with the staff. Of the choices below, what other information should be included in her discussion with the staff to improve hand hygiene compliance? 1) The number of patients on the unit during the monitoring period compared to the number of staff, as well as the nurse to patient ratio 2) Information about hand contamination 3) Information about the association between hand hygiene practices and the transmission of infection 4) The effects of hand hygiene products on skin a. 1, 2, 3 b. 2, 3, 4 c. 1, 3, 4 d. 1, 2, 4

b. 2, 3, 4 Rationale: To improve hand hygiene compliance, HCP should be provided with evidence-based information about hand contamination, the effects of hand hygiene products on the physiology of normal skin, and the association between hand hygiene practices and transmission of infection. The process of change is complex, and single interventions often fail; therefore, a multimodal, multidisciplinary strategy is necessary to change and improve hand hygiene practices. Reference: APIC Text, 4th edition, Chapter 27 - Hand Hygiene CBIC Core Competency: Preventing/Controlling the Transmissio

The organization that would most likely survey a U.S.-based hospital's blood bank for compliance would be: a. Occupational Safety and Health Administration (OSHA) b. Food and Drug Administration (FDA) c. Environmental Protection Agency (EPA) d. Centers for Disease Control and Prevention (CDC)

b. Food and Drug Administration (FDA) Rationale: The FDA is responsible for the safety of the nation's blood supply. The FDA has specific standards for the collection, testing, and distribution of blood, as well as disposal of contaminated or untested blood. These standards apply to all facilities that have blood-banking operations. Reference: APIC Text, 4th edition, Chapter 4 - Accrediting and Regulatory Agencies CBIC Core Competency: Management and Communication

A healthcare facility is undergoing extensive renovation. Surveillance for which of the following organisms would be particularly important during any construction or renovation project in a health care facility? a. MRSA, VRE, and other MDROs b. Legionella and Aspergillus c. Gram-negative bacilli d. Mycobacterium abscessus

b. Legionella and Aspergillus Dust particles contaminated with bacteria and fungi are dispersed and pose a health risk for patients, staff, and visitors. Construction-related HAIs are primarily cause by fungi and lesser by bacteria

The key to cleaning and disinfection of environmental surfaces is the: a. use of specific antiseptics for each surface b. physical removal of visible dirt, organic material, and debris c. assignment of trained staff to each department d. maintenance of equipment used for cleaning

b. physical removal of visible dirt, organic material, and debris Antiseptics are used for Skin only

The incubation period for meningococcal meningitis is: a. 10 to 14 days b. 7 to 21 days c. 2 to 10 days d. 1 to 2 days

c. 2 to 10 days Rationale: The incubation period for meningococcal meningitis is between 2 to 10 days, commonly 3 to 4 days. Reference: Meningococcal infection. In: Heymann D. Control of Communicable Diseases Manual, 19th edition. Washington, DC: American Public Health Association, 2008. CBIC Core Competency: Identification of Infectious Disease Processes

What species of the streptococci can be transferred to an infant during delivery? a. Streptococcus pyogeses b. Xanthomonas c. Streptococcus agalactiae d. Enterococcus faecalis

c. Streptococcus agalactiae Rationale: Asymptomatic carriage of Streptococcus agalactiae or group B Streptococcus (group B strep) in gastrointestinal and genital tracts is common. Intrapartum transmission via ascending spread from vagina occurs. Neurologic sequelae include sight or hearing loss and cerebral palsy. Death occurs in 5 percent of infants. Reference: APIC Text, 4th edition, Chapter 9 - Streptococci CBIC Core Competency: Identification of Infectious Disease Processes

Bed bugs can survive without feeding for: a. 48 hours b. 24 days c. 6 month d. 1 year

d. 1 year Rationale: Immature bed bugs may live for several months without feeding, while adults may survive as long as one year without a meal. Under normal conditions, adult bed bugs will live for about ten to eleven months. Reference: APIC Text, 4th edition, Chapter 107 - Environmental Services CBIC Core Competency: Environment of Care

When using heat to treat a room for bed bugs, the temperature must be: a. 85°F or 29°C for one hour b. 110°F or 43°C for 90 minutes c. 125°F or 51°C for two hours d. 118°F or 48°C for one hour

d. 118°F or 48°C for one hour Rationale: Bed bugs are challenging to eradicate. Some pest control firms utilize specialized heating equipment to de-infest furnishings, rooms, and entire dwellings. The procedure involves heating up the infested item or area to temperatures lethal to bed bugs. Portable heaters and fans are used to treat the room with high heat (118°F or 48°C for one hour). Reference: APIC Text, 4th edition, Chapter 107 - Environmental Services CBIC Core Competency: Environment of Care

The most common reservoir for highly pathogenic avian influenza H5N1 visurs is: a. Migratory birds b. Pigeons and Doves c. Water fowl d. Domestic poultry

d. Domestic poultry

Which of the following is the most important focus of an infection prevention control program during construction and repairs within a healthcare facility? a. Staying within the proposed time frame for the project b. Minimizing noise c. Keeping costs low d. Reducing dust

d. Reducing dust

Goal-Directed Checklist

enhance patient safety/quality of care by following a very prescriptive checklist. SSI Prevention check list

Type II error

failing to reject a false null hypothesis

selective media

suppress growth of unwanted bacteria (or flora) and encourage growth of desired microbes.

What are the three most commonly reported waterborne pathogens?

1. Pseudomonas spp 2. Legionella 3. Atypical (or nontuburculosis) Mycobacteria

What is the High-Level Disinfection process for Endoscopes?

1) To detect damaged endoscopes, test each flexible endoscope for leaks as part of each reprocessing cycle. Remove from clinical use any instrument that fails the leak test, and repair this instrument. Category II 2) Immediately after use, meticulously clean the endoscope with an enzymatic cleaner that is compatible with the endoscope. Cleaning is necessary before both automated and manual disinfection. Category IA. 3) Disconnect and disassemble endoscopic components (e.g., suction valves) as completely as possible and completely immerse all components in the enzymatic cleaner. Steam sterilize these components if they are heat stable. Category IB. 4) Flush and brush all accessible channels to remove all organic (e.g., blood, tissue) and other residue. Clean the external surfaces and accessories of the devices by using a soft cloth or sponge or brushes. Continue brushing until no debris appears on the brush. Category IA 5) Use cleaning brushes appropriate for the size of the endoscope channel or port (e.g., bristles should contact surfaces). Cleaning items (e.g., brushes, cloth) should be disposable or, if they are not disposable, they should be thoroughly cleaned and either high-level disinfected or sterilized after each use. Category II. 6) Discard enzymatic cleaners (or detergents) after each use because they are not microbicidal and, therefore, will not retard microbial growth. Category IB. 7) Process endoscopes (e.g., arthroscopes, cystoscope, laparoscopes) that pass through normally sterile tissues using a sterilization procedure before each use; if this is not feasible, provide at least high-level disinfection. High-level disinfection of arthroscopes, laparoscopes, and cystoscopes should be followed by a sterile water rinse. Category IB. 8) Phase out endoscopes that are critical items (e.g., arthroscopes, laparoscopes) but cannot be steam sterilized. Replace these endoscopes with steam sterilizable instruments when feasible. Category II. 9) Mechanically clean reusable accessories inserted into endoscopes (e.g., biopsy forceps or other cutting instruments) that break the mucosal barrier (e.g., ultrasonically clean biopsy forceps) and then sterilize these items between each patient. Category IA. 10) Use ultrasonic cleaning of reusable endoscopic accessories to remove soil and organic material from hard-to-clean areas. Category II. 11) Process endoscopes and accessories that contact mucous membranes as semicritical items, and use at least high-level disinfection after use on each patient. Category IA 12) Use an FDA-cleared sterilant or high-level disinfectant for sterilization or high-level disinfection (Table 1). Category IA 13) After cleaning, use formulations containing glutaraldehyde, glutaraldehyde with phenol/phenate, ortho-phthalaldehyde, hydrogen peroxide, and both hydrogen peroxide and peracetic acid to achieve high-level disinfection followed by rinsing and drying (see Table 1 for recommended concentrations). Category IB. 14) Extend exposure times beyond the minimum effective time for disinfecting semicritical patient-care equipment cautiously and conservatively because extended exposure to a high-level disinfectant is more likely to damage delicate and intricate instruments such as flexible endoscopes. The exposure times vary among the Food and Drug Administration (FDA)-cleared high-level disinfectants (Table 2). Category IB. 15) Federal regulations are to follow the FDA-cleared label claim for high-level disinfectants. The FDA-cleared labels for high-level disinfection with >2% glutaraldehyde at 25°C range from 20-90 minutes, depending upon the product based on three tier testing which includes AOAC sporicidal tests, simulated use testing with mycobacterial and in-use testing. Category IC. 16) Several scientific studies and professional organizations support the efficacy of >2% glutaraldehyde for 20 minutes at 20ºC; that efficacy assumes adequate cleaning prior to disinfection, whereas the FDA-cleared label claim incorporates an added margin of safety to accommodate possible lapses in cleaning practices. Facilities that have chosen to apply the 20 minute duration at 20ºC have done so based on the IA recommendation in the July 2003 SHEA position paper, "Multi-society Guideline for Reprocessing Flexible Gastrointestinal Endoscopes 17) When using FDA-cleared high-level disinfectants, use manufacturers' recommended exposure conditions. Certain products may require a shorter exposure time example: 0.55% ortho-phthalaldehyde for 12 minutes at 20°C, 7.35% hydrogen peroxide plus 0.23% peracetic acid for 15 minutes at 20°C than glutaraldehyde at room temperature because of their rapid inactivation of mycobacteria or reduced exposure time because of increased mycobactericidal activity at elevated temperature (e.g., 2.5% glutaraldehyde at 5 minutes at 35°C). Category IB. 18) Select a disinfectant or chemical sterilant that is compatible with the device that is being reprocessed. Avoid using reprocessing chemicals on an endoscope if the endoscope manufacturer warns against using these chemicals because of functional damage (with or without cosmetic damage). Category IB 19) Completely immerse the endoscope in the high-level disinfectant, and ensure all channels are perfused. 20) As soon as is feasible, phase out nonimmersible endoscopes. Category IB 21) After high-level disinfection, rinse endoscopes and flush channels with sterile water, filtered water, or tapwater to prevent adverse effects on patients associated with disinfectant retained in the endoscope (e.g., disinfectant induced colitis). Follow this water rinse with a rinse with 70% - 90% ethyl or isopropyl alcohol. Category IB. 22) After flushing all channels with alcohol, purge the channels using forced air to reduce the likelihood of contamination of the endoscope by waterborne pathogens and to facilitate drying. Category IB. 23) Hang endoscopes in a vertical position to facilitate drying. Category II. 24) Store endoscopes in a manner that will protect them from damage or contamination. Category II 25) Sterilize or high-level disinfect both the water bottle used to provide intraprocedural flush solution and its connecting tube at least once daily. After sterilizing or high-level disinfecting the water bottle, fill it with sterile water. Category IB. 26) Maintain a log for each procedure and record the following: - Patient's name and medical record number, - Procedure, - Date, - Endoscopist, - System used to reprocess the endoscope, and - Serial number or other identifier of the endoscope used. Category II. 27) Design facilities where endoscopes are used and disinfected to provide a safe environment for healthcare workers and patients. Use air-exchange equipment (e.g., the ventilation system, out-exhaust ducts) to minimize exposure of all persons to potentially toxic vapors (e.g., glutaraldehyde vapor). Do not exceed the allowable limits of the vapor concentration of the chemical sterilant or high-level disinfectant (e.g., those of ACGIH and OSHA). Category IB, IC 28) Routinely test the liquid sterilant/high-level disinfectant to ensure minimal effective concentration of the active ingredient. Check the solution each day of use (or more frequently) using the appropriate chemical indicator (e.g., glutaraldehyde chemical indicator to test minimal effective concentration of glutaraldehyde) and document the results of this testing. Discard the solution if the chemical indicator shows the concentration is less than the minimum effective concentration. Do not use the liquid sterilant/high-level disinfectant beyond the reuse-life recommended by the manufacturer (e.g., 14 days for ortho-phthalaldehyde). Category IA

What are the six clinical characteristics of CJD?

1. Rapid and progressive dementia. 2. Ataxia 3. Myoclonic seizures 4. Visual or sensory deficits 5. Abnormal psychiatric behavior 6. Coordination deficits

What are the 3 most common bacterial agents associated with foodborne illness in the United States?

1. Salmonella spp. (11%) 2. Clostridium perfringens (10%) 3. Campylobacter spp. (9%)

What are the 4 antibacterial classes?

1. Beta-lactam drugs 2. Fluoroquinolones 3. Macrolides 4. Aminoglycosides

What are the 4 effective types of disinfectant available for CJD?

1. Chlorine 2. Phenol 3. Guanidine thiocyanate 4. Sodium hydroxide Chlorine is the most readily available disinfectant and provides the most consistent prion inactivation. However, the corrosive nature of chlorine makes it unsuitable for semi-critical devices, such as endoscopes.

3 Key Aspects of a Cleaning Procedure

1. Cleaning schedules and procedures progress from the least soiled areas to the most soiled (patient zone) and from high surfaces to low ones. 2. To ensure that all surfaces are reached cleaning should be performed in a systematic manner. 3. After selecting a starting point, such as the door or window, work typically progresses from that point in either a clockwise or counterclockwise direction.

A comprehensive influenza infection prevention plan includes what four components?

1. Early identification and isolation of patients. 2. Annual education about influenza and the benefits of vaccination for HCP and patients. 3. Restricting ill patients and HCP 4. Hand hygiene as well as vaccination of HCP.

What is the survival time of Staphylococcus aureus on environmental surfaces?

7 days to >12 months

Relative Risk (RR)

= [a/(a+b)]/[c/(c+d)] Use 2x2 table

odds ratio

= ad/bc

You are a new IP at an ACH. The EVS manager in your facility has suggested you switch to a phenolic disinfectant in the NICU. Why would you oppose to this change?

Because there is an association between the excessive use of a phenolic disinfectant and hyperbilirubinemia in newborns, phenolics are not recommended for use on nursery surfaces (eg isoletts, bassinets, scales) with which infants may have contact.

Facultative Anaerobes

Both aerobic and anaerobic growth; greater growth in presence of oxygen

What is the clinical presentation of a mites infestation?

Burrow-type pruritic lesions on hands, webs of fingers, wrists, and extensor surfaces of elbows and knees, as well as outer surfaces of feet, armpits, buttocks, and waist.

An IP is designing a new infection prevention training curriculum that incorporates concepts of active learning. Which of the following strategies will support the IPs goals? 1. Use of workbooks 2. PowerPoint Presentations 3. Simulation 4. Case Studies A. 1,3 B. 2,3 C. 3,4 D. 1, 4

C. 3,4 Active learning is a process whereby students engage in activities, such as reading, writing, discussion, or problem solving that promote analysis, synthesis, and evaluation of class content. Cooperative learning, problem-based learning, and the use of case methods and simulations are some approaches that promote active learning. Reference: APIC Text, 4th ed., Chapter 3 - Education and Training

Types of tissue consider at high risk for CJD include: 1) Heart 2) Blood 3) Eye 4) Brain A. 1,4 B. 1,3 C. 3,4 D. 2,4

C. 3,4 Rationale: Tissues known to be highly infectious include brain, dura matter, pituitary tissue spinal cord and eye; tissues with low infectivity include lung, liver, kidney, spleen, lymph and CSF. Reference: APIC Text, 4th edition, Chapter 73 - Creutzfeldt Jakob Disease and other Prion Infections

Isolation Initiation and Duration: RSV (Respiratory Syncytial Virus) (Immunocompetent adults)

Isolation Type: Droplet Duration: 7 days after illness onset or until 24 hours after resolution of fever and respiratory symptoms, whichever is longer.

Which research study design is very likely to be impacted by recall bias?

Case-control

Catalase Test and Coagulase Test

Catalase - Used to differentiate between streptococci (negative) and Staphylococci (positive) Coagulate - used to differentiate S. aureus from other Staphlyocooi such as S epidermidis (negative)

What are the three phases of pertussis and the symptoms associated with each phase?

Catarrhal phase - 7-10 day incubation with low grade fever, cough, coryza Paroxysmal phase - intense coughing spasms Convalescent phase - cough may last for 2-8 additional weeks

Bloom's Taxonomy: Understanding

Classify, describe, discuss, explain, identify, locate, recognize, report, select, translate, paraphrase. Example: "Eyeshadow goes on your eye lid"

CDC Definition of Cleaning

Cleaning is the removal of foreign material (e.g., soil, and organic material) from objects and is normally accomplished using water with detergents or enzymatic products. Thorough cleaning is required before high-level disinfection and sterilization because inorganic and organic materials that remain on the surfaces of instruments interfere with the effectiveness of these processes. Also, if soiled materials dry or bake onto the instruments, the removal process becomes more difficult and the disinfection or sterilization process less effective or ineffective. Surgical instruments should be presoaked or rinsed to prevent drying of blood and to soften or remove blood from the instruments.

SSI Surveillance: Organ/ Deep for 90 Days includes which procedures

Craniotomy and CABG

What is the risk of transmission and seroconversion of Human Immunodeficiency Virus (HIV) after percutaneous exposure and seroconversion after mucous membrane, nonintact skin exposure? A. 30% and 90% B. 90% and 30% C. 0.09% and 0.3% D. 0.3% and 0.09%

D. 0.3% and 0.09% Risk for transmission: - Seroconversion after percutaneous exposure is 0.3% - Seroconversion after mucous membrane, nonintact skin exposure is 0.09% - Risk of percutaneous transmission is increased with hollow-bore needles and when an increased volume of blood is injected. - Consistent adherence to Standard Precautions and use of needleless systems and other safety needles will decrease the risk of exposure.

HBV infection during pregnancy can result in severe disease for the mother, fetal loss, or chronic infection of the neonate if born alive. Without postexposure immunoprophylaxis, approximately what percentage of infants born to HBV-infected mothers in the US will develop chronic HBV infection, and approximately how many will eventually die from chronic liver disease? A. 45% and 1/2 B. 20% and 1/4 C. 40% and 2/3 D. 40% and 1/4

D. 40% and 1/4 approximately 40% percentage of infants born to HBV-infected mothers in the US will develop chronic HBV infection, and approximately 1/4 will eventually die from chronic liver disease Pregnancy is NOT a contraindication to vaccination of HBV.

What is the mode of transmission for RSV?

Direct contact with nasal droplets from infected individuals or by indirect contact with contaminated hands or fomites.

What is the definitive way to diagnose CJD?

Direct examination of brain tissue. Utilizing a Western blot assay. Routine diagnostic tests are rarely of value in making a diagnosis of CJD.

Glutaraldehyde

Environmental Protection Agency (EPA)-registered high-level disinfectant.

True or False The Advisory Committee on Immunization Practices (ACIP) recommends routine Influenza (LAIV) vaccinations for all persons aged >6 months, including pregnant women.

False The Influenza inactivated vaccine is recommended for pregnant women due to them being at increased risk for severe illness from influenza because of physiologic changes during pregnancy. HOWEVER, the Influenza live attenuated vaccine (LAIV) is contraindicated for pregnant women.

Which Influenza strain is predominantly the cause of pandemics?

Influenza A strain - 2009 H1N1 and 1819 Pandemic

Gram- negative bacteria - rods/bacilli

K. pneumoniae E. coli P. aeruginosa A. baumannii

Surveillance

The collection of data with the ultimate objective of dissemination of that data to support and improve public health activities.

Ture or False P. aeruginosa has been associated with hot tubs and whirlpools; burn patients are uniquely susceptible to infection, and prevention of contamination in this type of equipment has been studied extensively

True

True or False Organic materials such as blood and protein inactivate many disinfectants. Organic matter must be removed from surfaces before applying the disinfectant.

True Point of use enzymatic cleaner

Qualitative Research Methods

Used to provide more insight and pursue knowledge on an existing theory or to discover meaning. Moves from an event or process to the theory. Open-ended - observations, compare and contrast, to then formulate the theory. Exploratory. Used to identify variables - very instructed Study environments and groups of people. Focuses on the WHOLE group.

Affinity Diagrams

used to generate ideas that can be linked to form organized patterns of thought about a problem Sticky notes that are then split into categories

Steps for an outbreak investigation

1. Establish the existence of an outbreak 2. Verify the diagnosis 3. Define and identify cases. 4. Describe and orient the data in terms of time, place, and person. 5. Develop hypotheses 6. Evaluate hypothesis 7. Refine hypothesis and carry out additional studies 8. Implement control and prevention measures 9. Communicate findings Communication and collaboration with relevant personnel during an outbreak situation is essential. The first cases to be recognized may only be a small portion of the total number. An important early step is to review aspergillus cultures to determine the extent of the outbreak. APIC text chapter 12 - outbreak investigation

What are the 5 vaccines for pregnant HCP?

1. Poliomyelitis 2. MMR 3. Hepatitis A &B 4. Tetanus 5. Diphtheria - or a reliable history of having the disease before becoming pregnant. No evidence exists for risk from vaccinating pregnant HCP with INACTIVATED vaccines or toxoids.

Which isolation precaution is required for Cytomegalovirus (CMV)? A. Standard B. Contact Droplet C. Droplet D. Airborne

A. Standard Rationale: CMV is transmitted most frequently by sexual contact or by direct contact with infected urine, saliva, semen, vaginal secretions, or breast milk. HCW should follow standard precautions to avoid exposure to these body fluids.

Attributable Risk

AR=IE-IU Attributable risk (AR) is the risk difference between exposed and unexposed groups. In equation form it is: = Risk(exposed)−Risk(unexposed) = [A/(A+B)]−[C/(C+D)] It is the quantity of disease that one could hope to avoid by removal of the risk factor in question.

Aminoglycosides

Act at the site of bacterial ribosomes. It is used for combination therapy for serious or multidrug-resistant infections. Example: gentamicin, amacacin

Toxic anterior segment syndrome (TASS)

Acute, sterile inflammation following anterior segment surgery of the eye.

Bloom's Taxonomy: Creating

Assemble, construct, create, design, develop, formulate, write Example: "I am creating my own eyeshadow collection"

What organism is uesed in a Biological Indicator (BI) for Ethylene Oxide (EtO) sterilizaion?

Bacillus artophaeus

A critical review of the literature contains: A. The significance of the results for clinical practice. B. A description of the findings in each article. C. A synthesis of the analysis of the information in the reviewed papers. D. A summary of each author's results.

C. A synthesis of the analysis of the information in the reviewed papers. A literature review surveys articles, books and other sources (e.g. dissertations, conference proceedings) relevant to a particular issue, area of research, or theory, providing a description, summary, and critical evaluation of each work. The purpose is to offer an overview of significant literature published on a topic. Reference: APIC Text, 5th edition, Chapter 20 Research study design

According to the CDC guidelines for caring for Ebola patients, the initial step in doffing personal protective equipment (PPE) under direct supervision is to: A. Remove apron B. Disinfect outer gloves C. Inspect PPE D. Remove outer gloves

C. Inspect PPE Rationale: The CDC doffing process for Ebola is as follows: PPE should be doffed in the designated removal area. Place all PPE waste in leak-proof infectious waste container. 1. Engage Trained Observer 2. Inspect 3. Disinfect outer gloves 4. Remove apron (if used) 5. Inspect 6. Disinfect and Remove outer gloves 7. Inspect and disinfect inner gloves

HCV is most commonly transmitted by all of the following except: A. Sexually transmitted B. Exposure to blood via transfusion C. Perinatal exposure D. Percutaneous injury or sharing needles

C. Perinatal exposure Studies have shown that acute HCV infection in the first and second trimesters cause fetal hepatic injury.

There are three principal goals of an Infection Prevention and Control Program (IPC). To help achieve those goals, the IPC has defined key functions. All of the following are the principal goals for IPC except: A. Protect the patient B. Protect the healthcare worker, visitors, and others in the healthcare environment C. Protect the integrity of the program through education, collaboration, and evidence best practices D. Accomplish goals in a cost-effective manner whenever possible

C. Protect the integrity of the program through education, collaboration, and evidence best practices

What are the 3 common Prion Diseases?

CJD, Kuru, and GSS Syndrome

What is the correct diagnosis for a patient with the following lab results: HBsAg: + Total Anti-HBc: + IgM Anti-HBc: - Anti-HBs: - A. Susceptible, never infected B. Acute, resolving infection C. Past infection, recovered and immune D. Chronic infection

D. Chronic infection HBsAg: + = Positive for Surface antigen = either by exposure or vaccination Total Anti-HBc: + = Positive for Core antibodies = from infection IgM Anti-HBc: - = Negative for currently making anti-bodies for the core Anti-HBs: - = Negative for antibodies for the surface antigen HBsAg: if this is positive, that means you currently have an infection (regardless of acute or chronic) IgG-HBsAg: if this is positive, you have won the battle; either you are Cured or Vaccinated

All of the following are significant events that require HCP to have respirator fit testing repeated, except: A. Annual fitting B. Whenever there is a respirator redesign C. Facial changes, such as extreme weight loss or gain D. When they are assigned to an Airborne isolation room

D. When they are assigned to an Airborne isolation room

What organism is uesed in a Biological Indicator (BI) for heat and/or steam sterilizaion?

Geobacillus stearothermophilus

By what mechanism do antivirals work?

Inhibit information of DNA precursors, DNA polymerase, and HIV reverse transcription. They interfere with viral uncoating or confer resistance on uninfected cells.

Isolation Initiation and Duration: Chickenpox (Varicella)

Isolation Type: Airborne Contact Duration: Until lesions dry and crusted.

Isolation Initiation and Duration: Salmonella

Isolation Type: Standard with Private room and private bath Contact Enteric if incontinent Duration: If incontinent, Contact Enteric Precautions for duration of illness

Isolation Initiation and Duration: E. coli O157:H7

Isolation Type: Standard with Private room and private bath Contact Enteric if incontinent Duration: If incontinent, for duration of illness

Isolation Initiation and Duration: Giardia

Isolation Type: Standard with Private room and private bath Contact Enteric if incontinent Duration: If incontinent, for duration of illness

Foodborne illness: E. coli (producing toxin)

"Traveler's Diarrhea" Incubation Period = 1-3 days Signs & Symptoms = Watery diarrhea, abdominal cramps, some vomiting. Duration = 3-7 or more days Food Sources = Water or food contaminated with human feces.

Isolation Initiation and Duration: Influenza

Isolation Type: Droplet Duration: 7 days after illness onset or until 24 hours after resolution of fever and respiratory symptoms, whichever is longer

Isolation Initiation and Duration: Rhinovirus

Isolation Type: Droplet Duration: 7 days after illness onset or until 24 hours after resolution of fever and respiratory symptoms, whichever is longer

Isolation Initiation and Duration: Streptococcal (Group A) Pneumonia

Isolation Type: Droplet Duration: Until 24 hours effective antibiotic therapy

Isolation Initiation and Duration: Meningitis (Haemophilus influenzae & Neisseria meningitidis)

Isolation Type: Droplet Duration: Until 24 hours of effective antibiotic therapy

Isolation Initiation and Duration: Pertussis (Wooping cough)

Isolation Type: Droplet Duration: Until 5 days of effective antibiotic therapy

After high-level disinfection, what is the process for Semicritical equipment that will have contact with mucous membranes of the upper respiratory tract and what is the CDC Category Ranking?

After high-level disinfection: 1. rinse all items. Use sterile water, filtered water or tapwater 2. Followed by an alcohol rinse for Semicritical equipment that will have contact with mucous membranes of the upper respiratory tract (e.g., nose, pharynx, esophagus). Category II. There is no recommendation to use sterile or filtered water rather than tapwater for rinsing semicritical equipment that contact the mucous membranes of the rectum (e.g., rectal probes, anoscope) or vagina (e.g., vaginal probes). Unresolved Issue.

Which research study design is suitable for rare outcomes?

Case-control

Ground Theory (Qualitative research)

Attempts to identify underlying themes surrounding a particular social or psychological process to formulate theory. Purpose is not to provide proof for an existing theory but to collect and analyze data in a manner that allows theory to emerge directly from the data. Data are studied, coded, and compared and contrasted with one another until relationship between themes emerge. Very different from quantitative - looking for the same terms and themes among different people. Example - Mardigras - asking inperson interviews among different people and then analyzing the information received to find themes. Found that multiple interviews talked about family, food, culture, religion. Then taking the common themes and developing a theory.

Bacterial meningitis CSF analysis

Cloudy Increased neutrophils Increased Protein Decreased Glucose Increased WBC

What is the completeness of information (high, low, full) for each research study design: Cohort Case-Control Cross-Sectional

Cohort = High Case-Control = Low Cross-Sectional = Full, but only cross-sectonal

What is the ranking (high to low) for Cost and Time for each research study design: Cohort Case-Control Cross-Sectional

Cohort = High Case-Control = Low Cross-Sectional = Low

What is the size (large or small) of subjects/population required for each research study design: Cohort Case-Control Cross-Sectional

Cohort = Large Case-Control = Small Cross-Sectional = Large

What is the loss to follow-up for each research study design: Cohort Case-Control Cross-Sectional

Cohort = More Case-Control = Less Cross-Sectional = None

What is the changes in the characteristics of the subjects in each research design: Cohort Case-Control Cross-Sectional

Cohort = More likely Case-Control = Less Likely Cross-Sectional = None

Formative Evaluation (internal)

Conducted during the educational session to provide IMMEDIATE feedback and to allow appropriate changes to be made. Example - concept check during the course. If there is discrepancy in the answers by the learners, the educator and re-educate on the topic before moving forward. Focuses on the PROCESS Method: Feedback

Epidemiologic triangle of Disease causation

Consists of: 1. an external agent, 2. a susceptible host, 3. in an environment that brings the host and agent together.

Selective media growth media

Contains chemicals or antibiotics designed to inhibit normal commensal bacteria. Bismuth sulfate agar for the isolation of Salmonella spp.

What is ratio data?

Data is ordered/ranked and the numerical distance between points is consistent (and can be measured). But what makes it the king of measurement is that the zero point reflects an absolute zero (unlike interval data's arbitrary zero point). In other words, a measurement of zero means that there is nothing of that variable. Examples of ratio data: - Weight, height, or length - The temperature in Kelvin (since zero Kelvin means zero heat) - Length of time/duration (e.g. seconds, minutes, hours) In all of these examples, you can see that the zero point is absolute. For example, zero seconds quite literally means zero duration. Similarly, zero weight means weightless. It's not some arbitrary number. This is what makes ratio-type data the most sophisticated level of measurement. With ratio data, not only can you meaningfully measure distances between data points (i.e. add and subtract) - you can also meaningfully multiply and divide. For example, 20 minutes is indeed twice as much time as 10 minutes. You couldn't do that with credit scores (i.e. interval data), as there's no such thing as a zero credit score. This is why ratio data is king in the land of measurement levels.

Isolation Initiation and Duration: Parainfluenza (Adults)

Isolation Type: Droplet *Consider adding Contact Precautions if the patient is unable to practice respiratory etiquette/control secretions. Duration: 7 days after illness onset or until 24 hours after resolution of fever and respiratory symptoms, whichever is longer

Isolation Initiation and Duration: Human metapneumovirus

Isolation Type: Droplet Contact Duration: 7 days after illness onset or until 24 hours after resolution of fever and respiratory symptoms, whichever is longer

Isolation Initiation and Duration: Parainfluenza (infants & young children only)

Isolation Type: Droplet Contact Duration: 7 days after illness onset or until 24 hours after resolution of fever and respiratory symptoms, whichever is longer

Isolation Initiation and Duration: Streptococcal (Group A) disease (Major wound or serious, invasive disease)

Isolation Type: Droplet Contact Duration: Until 24 hours effective antibiotic therapy

Isolation Initiation and Duration: RSV (Respiratory Syncytial Virus) (Infants, young adults, and Immunocompromised patients)

Isolation Type: Droplet Contact Duration: 7 days after illness onset or until 24 hours after resolution of fever and respiratory symptoms, whichever is longer. In immunocompromised hosts, extend duration of Droplet and Contact Precautions due to prolonged shedding of virus.

Isolation Initiation and Duration: Candida auris

Isolation Type: Enhanced Contact Duration: Patient to remain in precautions (No discontinuation policy at this time)

Isolation Initiation and Duration: CP-CRE (Carbapenemase Producing -Carbapenem-resistant Enterobacteriaceae)

Isolation Type: Enhanced Contact Duration: Patient to remain in precautions (No discontinuation policy at this time)

Isolation Initiation and Duration: CRAB (Carbapenem-resistant Acinetobacter baumannii)

Isolation Type: Enhanced Contact Duration: Patient to remain in precautions (No discontinuation policy at this time)

Isolation Initiation and Duration: Hepatitis A

Isolation Type: Enteric Contact Duration: Following Washington State DOH Guidance recommendation as of 12/14/21.

Isolation Initiation and Duration: Camplylobacter

Isolation Type: Standard with Private room and private bath. Contact if incontinent Duration: If incontinent, for duration of illness

Isolation Initiation and Duration: Cryptosporidium

Isolation Type: Standard with private room and private bath. Contact Enteric if incontinent Duration: If incontinent, for duration of illness

Kurtosis

Ketosis represents how flat or peaked a curve is shaped. Measure of the fatness of the tails of a probability distribution relative to that of a normal distribution. Indicates likelihood of extreme outcomes.

Low-Level Disinfection

Kill most vegetative bacteria, some fungi, and some viruses Required for items that come into contact with intact skin

Intermediate-level Disinfection

Kill vegetative bacteria, mycobacteria, most fungi, most viruses and some spores

High-Level Disinfection

Kills spores as well as all other forms of microorganisms required for instruments that come into contact with mucous membranes. example - endoscope, vaginal probes

Sterilization - low temperature

Ethylene Oxide (ETO), Hydrogen Peroxide Gas Plasma, peracetic acid, ozone

True or False Sinks that are small, shallow, and with a tap that directed water over the drain are recommended.

FALSE Sinks that are small, shallow, and with a tap that directed water over the drain are NOT recommended.

Phenolic disinfectants

Powerful tuberculocidal disinfectants. They are a form of formaldehyde, have a very high pH, and can damage the skin and eyes. Not recommend for use in an infant nursery.

What is the alternative vaccine for individuals who have mild to severe allergic reactions to eggs?

Recombinant Influenza Vaccine (RIV)

Loss to follow-up bias

Refers to patients who at one point in time were actively participating in clinical research trial, but have become lost at the point of follow-up in the trial

Type I error

Rejecting null hypothesis when it is true

Nutrient agar growth media

Supports the growth of a wide variety of bacteria. TSA - Trypticase soy agar with 5% sheep's blood

Positive Predictive Value (PPV)

TP/(TP+FP)

True or False Habituation to an alarm, or alarm fatigue, recently identified as a sentinel event alert from TJC

TRUE

True or False The ICRA should influence the facility design, the procedures to prevent exposure from construction debris, and the selection of equipment and surfacing materials to manage infection risk.

TRUE

True or False Faucet aerators on sinks can enhance the growth of waterborne organisms.

TRUE Aerators are not recommended, but if they must be used, especially in an area with immunocompromised patients, a systematic cleaning routine should be established.

Disinfection

The elimination of many or all pathogenic organisms with the exception of bacterial spores. Surfaces must be cleaned before they can be properly disinfected.

Specificity of a test for infection or disease is calculated as:

The number of TRUE Negatives divided by the total number of persons WITHOUT disease, x 100 Specificity = (True Neg. / Total # without Disease)100

Which of the following parameters affect(s) the effectiveness of ethylene oxide (ETO) sterilization? 1) Gas concentration 2) Temperature 3) Relative humidity 4) Exposure time a. 1, 2 b. 1, 3, 4 c. 1, 2, 3, 4 d. 1, 4

c. 1, 2, 3, 4 Rationale: Ethylene oxide (ETO) sterilization is affected by four parameters. The concentration of gas should be between 450 and 1200 mg/L, the temperature range should be between 37 and 63°C, the relative humidity should be between 40 and 80 percent, and the exposure time should be between1 and 6 hours. Reference: CDC Guideline for Disinfection and Sterilization in Healthcare Facilities, 2008 Page: 61-62 CBIC Core Competency: Cleaning, Sterilization, Disinfection, Asepsis

quasi-experiment

Non-randomly assigned groups of patients to receive either a treatment or placebo

What is the proper dilution that OSHA stipulates for blood and bodily fluid spills to be decontaminated with an EPA-registered bleach for Nonporours surface and a porous surface?

Nonporous = 1:100 Porous = 1:10

Run Charts

type of chart used to show variations in the process over time or to show trends (such as improvements or the lack of improvements) in the process. Demonstrate special cause variation when tere is a steady pattern of observation points falling aboce or below the mean average.

Quality Outcome Measures

what is the patient outcome? SSI CAUTI CLABSI Sentinel Event

How long is the incubation period for RSV?

2-8 days

How long can mites survive on cloth furniture or stuffed animals?

3 days

SSI Surveillance period: Superficial incision

30 days for all procedures

What is the average time frame of symptom onset to time of death for someone with CJD?

7-9 months

Mortality rate

= (#of deaths/population at risk)x100,000 Example in 2006, the resident population of New Mexico is estimated at 2,010,787. The total number of deaths in New Mexico during the 2006 calendar year was 15,231. Crude mortality rate equals (15,221÷2,010,787)×100,000 = 757.5 resident deaths per 100,000 population in 2006 in New Mexico.

Incident density formula

= ((#of new cases)/(exposure time x device days))x100 Incorporate time into the denominator. Example: in February there at 8 new cases a ventilator associated pneumonia in ICU. We want to be able to understand that number as a portion of the total number of days that patients were on ventilators. There were 24 patients on the ventilator that month and each was on mechanical ventilation for three days. The number of ventilator days equals 24×3 = 72. The VAP rate per 1,000 vent days = (8/72)x1,000 = 111

An example of an obligate intracellular parasitic bacterium would be an organism responsible for: 1. Hepatitis 2. Q fever 3. Malaria 4. Chlamydia A. 2,3 B. 2,4 C. 3,4 D. 1,2

B. 2,4

You receive a call from a young man who thinks he was exposed to HIV. His baseline HIV test (ELISA) was negative. At what time period after exposure would we be most likely to detect HIV antibodies? A. 1 to 3 months B. 3 Weeks C. 6 Months D. 12 months

A. 1 to 3 months The window period for HIV

Important considerations regarding blood culture specimens should include: 1. Collect prior to the initiation of antimicrobial therapy 2. Collect from a central venous catheter whenever possible 3. Ensure that the volume of the specimen collected is sufficient 4. Culture of specific sites is not recommended for surveillance A. 1,3 B. 2,4 C. 1,4 D. 3,4

A. 1,3

The IP wants to ensure that educational programs are based on the most rigorous and reliable sources of clinical evidence. Which of the following sources would best meet this need? A. Standards issued by national or international authoritative sources B. Best-practice guidelines from professional organizations C. Consensus statements published by leading subject matter experts D. Literature review of publications during the past 5 years

A. Standards issued by national or international authoritative sources Rationale: Standards issued by national or international authoritative sources represent the penultimate standard of care used to guide practice, reduce variation, and improve the quality of patient care. References: APIC Text, 4th edition, Chapter 1 - Infection Prevention and Control Programs; APIC Text, 4th edition, Chapter 4 - Accrediting and Regulatory Agencies

What is/are the highest risk antimicrobial(s) associated with causing CDI? A. Third-generation cephalosporins B. Fluoroquinolones C. Clindamycin and Vancomycin D. Metronidazole

A. Third-generation cephalosporins

By what mechanism do antifungals work?

Alter permeability of fungal membrane Inhibit membrane biosynthesis or DNA synthesis

How long is the incubation period for a primary infestation of mites?

As early as 10 days, but typically 4-6 weeks. Patients with reinfestation, symptoms usually appear within 1-3 days.

cross-sectional study

Assesses the prevalence of an outcome in a broad population at one point in time. Provide a "snapshot" of a population at a moment in time. Point Prevalence. (Prevalence, correlation or survey studies) Exam in the outcome and risk factors reviewed in a population group at one point in time. In the studies the outcomes are measured the incidence rate cannot be determined.

An IP recommends that CHG be used to prepare a site for the insertion of a PICC because it: 1) Requires only 30 seconds to dry 2) Can be used for all ages 3) Is not associated with allergic reactions 4) Can be easily rinsed after the line is inserted A. 3,4 B. 1,3 C. 2,3 D. 1,4

B 1,3 Requires only 30 seconds to dry and Is not associated with allergic reactions. CHG does not need to be removed or rinsed from the skin. CHG is not approved for use with infants younger than 2 months of age

What is the most common method of evaluating major incidents, sentinel events, and medical errors? A. SWOT Analysis B. RCA C. Statistical Process Control D. PDSA

B. RCA

While performing the infection prevention and control program assessment, it is noted that program mission, goals and objectives have not been developed. A special departmental staff meeting is held to accomplish this task. Which of the following best describes a mission statement for an infection prevention and control program? A. The statement tells what the future of the program will be B. The statement tells why the program exists C. The statement tells who is responsible for the program D. The statement tells what the program will do

B. The statement tells why the program exists Rationale: A mission statement defines what the program is, why it exists, its reason for being. The mission statement should define who your primary customers are, identify the products and services you produce, and describe the distinctive features of your program. Reference: APIC Text Online, Chapter 1 - Infection prevention and control programs

A food service worker is diagnosed with Hepatitis A. How long should this employee be on work restrictions? A. Until 14 days after symptoms resolve B. Until 7 days after onset of jaundice C. Until 14 days after onset of jaundice D. Until 10 days after symptoms resolve

B. Until 7 days after onset of jaundice Rationale: According to the ACIP, food service workers who are diagnosed with Hepatitis A must be restricted from food handling until 7 days after the onset of jaundice.

The Director of the Infection Prevention and Control Department has assigned one of her IPs to cofacilitate in a root cause analysis of an adverse event in collaboration with the Performance Improvement team. The IP plans to use process improvement tools and techniques during the analysis. Which of the following methods would best outline the possible causes of the event? A. Brainstorming B. Affinity diagrams C Fishbone diagram D. Pareto chart

C Fishbone diagram

___________ is a systematic, proactive method for evaluating a process to identify where and how it might fail and to assess the relative impact of different failures in order to identify the parts of the process that are most in need of change. A. PDSA B. RCA C. FMEA D. DMAIC

C. FMEA

A microbe that can grow in the absence of oxygen, but is also able to utilize oxygen for growth is a/an: A. Aerobe B. Obligate anaerobe C. Facultative anaerobe D. Microaerophilic anaerobe

C. Facultative anaerobe

Which of the following practices have studies shown is most likely to result in improved infection prevention? A. Strictly adhering to the EPA-registered product label contact time for low level disinfection of environmental surfaces. B. Utilizing a one minute contact time for low level disinfection of environmental surfaces regardless of the EPA-registered product label contact time. C. Thoroughly cleaning and disinfecting all surfaces in a room that have potentially come into contact with hands. D. Thoroughly cleaning and disinfecting all high touch surfaces in a room.

C. Thoroughly cleaning and disinfecting all surfaces in a room that have potentially come into contact with hands. Rationale: Studies have shown that less than half of all surfaces and equipment that may be contaminated are properly cleaned during a terminal cleaning. Any surfaces or equipment that may have come into contact with hands have the potential to be contaminated; this is inclusive of but also beyond surfaces designated as "high touch." Reference: APIC Text, 4th edition, Chapter 31 - Cleaning, Disinfection and Sterilization

Enrichment Growth Media

Chocolate agar, for hard to grow bacteria

Bloom's Taxonomy: Applying

Choose, demonstrate, dramatize, employ, illustrate, interpret, operate, schedule, sketch, solve, use, write Example: "Watch me demonstrate how to use a fluffy dome brush to apply the eye shadow to my crease"

Fungal meningitis CSF analysis

Clear/hazy Increase WBC's Normal to increase protein Normal to decreased glucose Normal to increase lymphocytes Normal to increased monocytes Normal to decreased in neutrophils Increased Agglutination capacity

Viral meningitis CSF analysis

Clear/hazy Normal to increased protein Normal to decreased glucose Normal to increased WBC Normal to increase lymphocytes Normal to increased monocytes Normal to decreased neutrophils

Which research study design is suitable for rare exposures?

Cohort

What is the assessment of temporal relationship for each research study design: Cohort Case-Control Cross-Sectional

Cohort = Good Case-Control = Difficult Cross-Sectional = Not possible

Case-control study

Compares histories of a group of people with a condition to a group of people without the condition. Begins with the identification of individuals who have the OUTCOME of interest. After case-control status is defined, exposures are assessed and evaluated. Good for studying rare outcomes or outcomes that develop over a long time. Determine if two groups differ in exposure. (Case reference, case comparison. Or retrospective studies) Examine the population of individuals with and without an outcome of interest, study for exposure to one or more risk factors. The studies are quicker, less expensive, and easier. Generally yields only estimate of relative risk (odds ratio). Example: Foodborne illness investigations

indirect association

Indirect association is the mixing of effects among exposure, disease, and a third factor (e.g. a confounding variable) that is associated with the exposure and independently affect the outcome.

Gram- Positive bacteria cell wall

Contains lipoteichoic acid Large/thick peptidoglycan wall

How many days should women avoid becoming pregnant after an MMR vaccination? A. 1-3 Weeks B. 3 Months C. 14 Days D. 28 Days

D. 28 Days The MMR vaccine and its component vaccines SHOULD NOT be administered to individuals known to be pregnant. The risk to the fetus from administration of the live virus vaccines cannot be excluded for theoretical reasons, women should be counseled to avoid becoming pregnant for 28 days after vaccination with Measles or Mumps vaccines or MMR or other rubella-containing vaccines.

Which of the following would be an appropriate way to evaluate education provided to a group of employees on Hand Hygiene (HH)? 1. Exit Interviews 2. One-on-One Interviews 3. Direct observation 4. Pre-test and Post-test A. 2,3 B. 1,2 C. 1,4 D. 3,4

D. 3,4 Direct observation and Pre-test and Post-test As the instructor, the efficacy in the observations of HH and using the pre and post test to see if the learners understand when ABHR vs Soap and Water is appropriate.

Considering measures of distribution, what percentage of values should the CIC professional expect to fall outside of the 3 deviations from the mean? A. 10% B. 5% C. 2% D. <1%

D. <1%

What is nominal data?

Data that describes qualitative characteristics or groups, no order or rank between categories. Examples: Gender, Ethnicity, Eye color, Blood type, Brand

What is interval data?

Data that's naturally quantitative (is usually measured in numbers). Specifically, interval data has an order (like ordinal data), plus the spaces between measurement points are equal (unlike ordinal data). Examples: - Credit scores (300 - 850) - GMAT scores (200 - 800) - IQ scores - The temperature in Fahrenheit Importantly, in all of these examples of interval data, the data points are numerical, but the zero point is arbitrary. For example, a temperature of zero degrees Fahrenheit doesn't mean that there is no temperature (or no heat at all) - it just means the temperature is 10 degrees less the 10. Similarly, you cannot achieve a zero credit score or GMAT score. In other words, interval data is a level of measurement that's numerical (and you can measure the distance between points), but that doesn't have a meaningful zero point - the zero is arbitrary. Long story short - interval-type data offers a more sophisticated level of measurement than nominal and ordinal data, but it's still not perfect. Enter, ratio data

Cleaning and Disinfecting Environmental Surfaces in Healthcare Facilities: CDC Mop Heads and Cleaning Cloths Recommendations

Decontaminate mop heads and cleaning cloths regularly to prevent contamination (e.g., launder and dry at least daily). Category II

case reports/series

Detailed history of a small number of individual cases

What is the CDC recommendation for mops, and changing floor mopping solutions?

Every 3 rooms and at least every 60 minutes. Used mops and cleaning cloths should NEVER be returned to containers of cleaning solution. They should be laundered or discarded after use. Handles and poles for mops, dusters and other items should also be wiped with disinfectant after use. A disinfectant must be used to clean floors in critical areas, such as isolation rooms.

True or False When stored in a spray or wash bottle or in a closed, brown opaque container, the bleach solution will remain stable for 28 days and will retain 50% of it initial value.

FALSE will remain stable for 30 days.

True or False A combination of AII (Airborne Infection Isolation) and PE (Protective environment) rooms are permitted and supported by ANSI, ASHRAE, ASHE, and FGI to utilize a design in which isolation rooms may be converted from an AII room to a PE room by switching the airflows.

FALSE!!! While combination AII/PE rooms are permitted, the ANSI, ASHRAE, ASHE ventilation standard included in the FGI guidelines DOES NOT SUPPORT designs in which the isolation room may be converted from an AII to a PE room by switching the airflows. The position in the standard prohibiting switchable isolation rooms is based on the complexity of pressure relationships and concerns for serious patient and healthcare provider outcomes if errors were to be made in correct room pressurization.

True or False The IP's role during the ICRA is to be experts in ventilation, plumbing, specialty materials, and equipment depending on the scope of the project.

False

True or False Each year in the United States 7% of hospitalized patients develop an HAI

False 4%

True or False OSHA recommended a schedule for flushing that follows American National Standards Institute (ANSI) recommendations for stationary and portable eyewash/drench shower stations flushing the system for a 5-minute period each week.

False Recommends flushing the system for a 3 minute period each week.

cohort study

Follows a group of people to track risk factors and outcomes overtime Groups are defined regarding their EXPOSURE to a factor of interest. (perspective or longitudinal studies) Followed-up to determine differences in rates at which disease develops in relation to exposure. Start with a sample of individuals with and without EXPOSURE to a potential risk factors who are followed for incidence of the outcome in each group. They can also be done retrospectively. Cohort studies have less patient selection and stronger evidence of causal association. Yields incidence rates, relative risk, attributable risk More EXPENSIVE

Ethography (Qualitative Research)

Guided by a desire to understand the lives of individuals and the meanings of human behavior Example: used in anthropology and sociology - Participant observation - Interviews - Field notes - Archive data Reflects the culture or subculture of people: - how people interpret experiences, categorize events, and behave in a manner that reflects cultural experience. Before starting an interview you need to understand how the culture communicates and interprets information so they can understand you.

Transtheoretical Model/Stage Theory

How ready is the individual to change their behavior? Different levels of change: 1. Precontemplation - unaware of the problem. Need to create awareness 2. Contemplation - "I might" stage - aware of problem 3. Preparation - "I will" stage - intends to take action 4. Action - "I am" stage - practicing the behavior 5. Maintenance - "i have" stage - work to sustain change 6. Relapse - 7. Termination - no desire to relapse

Quality Process Measures

How well the steps within the process are followed HH Bundle compliance Effective vaccine rates

Cleaning and Disinfecting Environmental Surfaces in Healthcare Facilities: CDC Chlorine Solutions Recommendations

If chlorine solution is not prepared fresh daily, it can be stored at room temperature for up to 30 days in a capped, opaque plastic bottle with a 50% reduction in chlorine concentration after 30 days of storage (e.g., 1000 ppm chlorine [approximately a 1:50 dilution] at day 0 decreases to 500 ppm chlorine by day 30). Category IB.

Inferential statistics

In case of any parental statistics, we begin by differentiating between two groups. The population is the entire collection of individuals that we are interested in studying. It is often possible to examine each member of the population individually. Instead, we can choose a representative subset of the population called a sample. Inferential statistics studies a statistical sample, and is able to say something about the population from which the sample came, on the basis of this analysis.

Foodborne illness: Staphylococcus aureus

Incubation Period = 1-6 hours Signs & Symptoms = Sudden onset of severe nausea and vomiting. Abdominal cramps. Diarrhea and fever may be present Duration = 24-48 hours Food Sources = Unrefrigerated or improperly refrigerated meats, potato and egg salads, cream pastries

Foodborne illness: Norovirus

Incubation Period = 12-48 hrs Signs & Symptoms = Nausea, vomiting, abdominal cramping, diarrhea, fever, headache. Diarrhea is more prevalent in adults, vomiting more common in children Duration = 12-60 hrs Food Sources = Raw produce, contaminated drinking water, uncooked foods and cooked foods that are not reheated after contact with an infected food handler; shellfish from contaminated waters

Foodborne illness: Yersinia enterocolitica

Incubation Period = 24-48 hours Signs & Symptoms = Appendicitis-like symptoms (diarrhea and vomiting, fever, and abdominal pain). Duration = 1-3 wk, usually self-limiting Food Sources = Undercooked pork, unpasteurized milk, tofu, contaminated water. Infection has occurred in infants whose caregivers handled chitterlings.

Foodborne illness: Toxoplasma gondii

Incubation Period = 5-23 days Signs & Symptoms = Generally asymptomatic; 20 percent may develop cervical lymphadenopathy and/or a flulike illness. In immunocompromised patients, central nervous system (CNS) disease, myocarditis, or pneumonitis is often seen. Duration = Months Food Sources = Accidental ingestion of contaminated substances (e.g., soil contaminated with cat feces on fruit and vegetables), raw or partly cooked meat (especially pork, lamb, or venison). Passed from mother (who acquired acute infection during pregnancy) to child.

Foodborne illness: Clostridium perfringens

Incubation Period = 8-16 hours Signs & Symptoms = Intense abdominal cramps, watery diarrhea Duration = usually 24 hours Food Sources = Meats, poultry, gravy, dried or precooked foods, time and/or temperature-abused foods

Which Influenza strain is associated with 13% of all influenza infections?

Influenza C Produces mild illness, usually a "common cold" or are detected with another respiratory virus, which makes it difficult to assess its clinical role.

Fluoroquinolones

Inhibit bacterial enzymes important in DNA replicaion -Ciprofloxacin, norfloxacin, levofloxacin, ofloxacin, moxifloxacin, gemifloxacin, enoxacin. -Inhibit prokaryotic enzymes topoisomerase II (DNA gyrase) and topoisomerase IV. Bactericidal. Must not be taken with antacids. -Gram-negative rods of urinary and GI tracts (including Pseudomonas), Neisseria, some gram-positive organisms. Toxicity: GI upset, superinfections, skin rashes, headache, dizziness. Less commonly, can cause leg cramps and myalgias. *Contraindicated in pregnant women, nursing mothers, and children < 18 years old due to possible damage to cartilage. Some may prolong QT interval. May cause tendonitis or tendon rupture in people > 60 years old and in patients taking prednisone. Resistance: chromosome-encoded mutation in DNA gyrase, plasmid-mediated resistance, efflux pumps.

Isolation Initiation and Duration: Shingles (Varicella Zoster) Immunocompetent with isolated lesions

Isolation Type: Standard Duration: Isolated lesions are defined as having lesions on 2 or fewer contiguous dermatomes. Lesions must be covered if open and weeping.

Isolation Initiation and Duration: VRE (Vancomycin-resistant Enterococcus)

Isolation Type: Standard Duration: Isolation precautions for positive lab is not required

Isolation Initiation and Duration: TB (Tuberculosis) - Active, not latent *Suspected or lab-confirmed

Isolation Type: Airborne Respirator Duration: Until active TB is ruled-out with 3 consecutive negative AFB smears collected at least 8 hours apart or 2 negative MTB (PCR)

Isolation Initiation and Duration: Measles (rubeola)

Isolation Type: Airborne Respirator & Contact Duration: Discontinuation by IP in consultation with local public health authority (4 days after onset of rash; duration of illness in immunocompromised host). In setting where respirator is not available, then only caregivers immune by titer should provide care to the patient

Isolation Initiation and Duration: Bed bugs & Cockroaches (Bugs Visualized)

Isolation Type: Contact Duration: Until bathed and moved to new room (double bag & seal patient belongings). Refer to local policy regarding pest management or contact EVS regarding need for pest control evaluation.

Isolation Initiation and Duration: CR-PA (Carbapenemase-resistant Pseudomonas aeruginosa)

Isolation Type: Contact Duration: Patient to remain in precautions (No discontinuation policy at this time)

standard deviation

Measure of dispersion.. Descriptive statistic that describes the variability of the values around the mean. 1 standard deviation = 68.2% 2 standard deviations = 95.4% 3 standard deviations = 99.7%

Pharmacodynamic Factors

Minimal inhibitory concentration (MIC) - the lowest concentration of drug that can inhibit microbial growth.

Acid-fast Bacteria

Mycobacteria Contain mycolic acid = cord factor (VLCFA) long chains of fatty acid. Waxy surface impervious to chemicals or dyes. Important Pathogens: TB Leprosy Opportunistic wound infections Grow inside of macrophages Mycobacterium spp Nocardia Actinomyces organisms

Who am I? I am a pleomorphic bacteria. I lack a cell wall and i am surrounded by a plasma membrane. Because I lack a rigid cell wall, I am resistant to cell-wall active abx (eg penicillin's and cephalosporins). I can grow on artificial media that provides me with sterols (exogenous cholesterol).

Mycoplasma bacteria APIC CH 24 - Mycoplasma pneumoniae

What are the two levels of measurement for Categorical Data?

Nominal and Ordinal

Summative Evaluation (external)

Occurs after the program is completed to determine the impact and overall effectiveness. Example: Pre-test and Post-test, direct observation of practice, an exit questionnaire, 1-1 interviews Focuses on the OUTCOME

Performance Improvement

Performance improvement is an ongoing continuous cycle that focuses on patient clinical outcomes, customer satisfaction, and service. Examples: PDSA Cycle

Fill in the blank: A _____________ is one that results in a pass or fail fit test and one that assesses the adequacy of respirator fit that relies on the individuals response to the test agent.

Qualitative Fit test (QLFT)

Root Cause Analysis

RCA takes a RETROSPECTIVE look at adverse outcomes and determines what happened, why it happened, and what an organization can do to prevent the situation from recurring. Information Collection: through structured interview, document reviews, and field observations. Avoids individual blame, considers human factors engineering, and analyzes redesign for a safer system. "7:30, where were we at 7:30?" - Hermione Granger Going back in time to save buckbeak

Rate

Rate = (x/y)k X = the numerator, which equals the number of times the event (i.e. an infection) has occurred during a specific time interval. Y = The denominator, which equals a population (ie number of patients at risk) K = A constant that we used to transform the results of the division into a uniform quality. All numbers such as 100, 1000, or 100,000 are often used.

Strategies to prevent infection of a dialysis access site include the following: 1) Utilize an arteriovenous (AV) graft if a fistula cannot be established 2) Use a tunneled cuffed catheter for acute dialysis if use will be more than 3 weeks 3) Subclavian access is preferred over jugular options 4) Use femoral catheters only in bedbound patients for no more than 5 days A. 1, 3, 4 B. 1, 2, 4 C. 2, 3, 4 D. 1, 2, 3

Rationale: The risk of infection related to vascular access for hemodialysis varies with the type of vascular access used. Risk for bacteremia increases sevenfold in patients with a dialysis catheter compared to those with a primary arteriovenous (AV) fistula. It is recommended that an AV fistula be created and used for long-term hemodialysis treatment because of the lower incidence of infection. If an AV fistula cannot be established, an AV graft is the next preferred type of access. Because of infection risk, creation of the fistula in the upper arm is preferred over the thigh. For acute hemodialysis, where access for less than 3 weeks' duration is anticipated, vascular access may be obtained using a noncuffed or cuffed catheter. However, if a catheter must be used for access for longer than 3 weeks, a tunneled, cuffed venous catheter should be used. The preferred insertion site is the right internal jugular. Because there is a greater incidence of central venous thrombosis and stenosis when the subclavian is used, subclavian access should be used only when jugular options are not available and permanent vascular access is not required. In addition, tunneled cuffed catheters should not be placed on the same side as a maturing AV access if possible. Femoral catheters should be placed only in bedbound patients only with good exit site care and should be left in place for no more than 5 days because of associated infection rates. Reference:APIC Text, 4th edition, Chapter 39 - Dialysis CBIC Core Competency: Preventing/Controlling the Transmission of Infectious Agents

Bloom's Taxonomy

Remembering, -> Understanding, -> Applying,-> Analyzing, -> Evaluating, -> Creating

Standardize infection ratio (SIR)

SIR = (# Observed) / (# Predicted)

What parasites are most commonly associated with healthcare-associated transmission?

Scabies, lice, and maggots.

Skewness (negative and positive)

Skewness is asymmetrical distribution. Negative Skew: Elongated tail at the left. More data in the left tail then will be expected in a normal distribution. Positive Skew: Elongated tail at the right. More data in the right tail then will be expected in the normal distribution.

Differential Media growth media

Stains colonies of specific organisms, while inhibiting the growth of others. Acetate agar to differentiate between E. Coli from Shigella

Goals

Statements that communicate the intent of the curriculum and provide a direction for planning the education session.

True or False If a diluted bleach solution is stored in an open container, the chlorine rapidly dissipates and therefore must be prepared daily.

TRUE

True or False The water temperature for showers and bathing should be appropriate for comfortable use, and the CDC 2010 guidelines permit a temperature range between 105F and 120F. The CDC recommends maintaining water temperature at 124F or higher and cold water temperature at 68F to control legionella.

TRUE

True or False Infectious levels of HBV DNA can be detected on environmental surfaces in the absence of visible blood.

TRUE The risk of HBV seroconversion after percutaneous injury ranges from 23-62%

What is the temperature and humidity requirement in Central Service (CS) assembly areas?

Temp: 68-73F Humidity: 30-60%

Specificity

The ability of a test to correctly identify a negative result. = (Number of true-Negative results) + (Number of False-Positive results) High Specificity = TRUE POSITIVES. you will not have any false positive tests. High chance for false negatives.

Attributable Risk Percent

The percentage of a disease that could be prevented by eliminating the exposure in the study population. = (AR/Incidence Exposed)*100 =((a/(a+b)) - (c/(c+d)) / (a/(a+b))) *100

The p value in statistical test results indicates...

The probability of having committed a Type I error. A Type I error occurs when one rejects the null hypothesis when it is true. "False positive result" The p value or calculated probability is the estimated probability of rejecting the null hypothesis of a study question when tat hypothesis is true.

Social Cognitive Theory

Triadic relationship between: Personal Determinants, Environmental Determinants, and Behavioral Determinants. People learn by observing Personal - knowledge and skills Behavioral - ability to perform the behavior Environmental - family, socioeconomic, resources in the facility, peers

True or False Inhalation of aerosols of contaminated water is the primary mechanism of transmission for Legionella, not person-to-person transmission

True

True or False TJC defines sentinel events and requires accredited facilities to complete a thorough and credible root-cause analysis and action plan within 45 calendar days of becoming aware of the event

True

True or False The average risk of seroconversion after a percutaneous injury involving blood infected with Hepatitis C virus is approximately 1.8%.

True

True or False Air circulation from the AII room to the general ventilation system or through a dedicated air handler unit in the AII room is not permitted in new construction. The exceptions are as follows: 1. AII rooms retrofitted from standard patient rooms 2. Where it is impractical to exhaust the air outdoors

True In this situation, recirculation of room air is permitted, provided the air first passes through a HEPA filter. In addition to the ventilation requirements, AII rooms must meet the basic patient room requirements.

True or False Proper sink design (ie depth of sink, length of the spout, distance between the spout and skink drain) is key to the prevention of waterborne outbreaks related to sinks.

True Outbreak of P. aeruginosa related to a shallow basin, close proximity to point of care, and water that splashed from the sink to surfaces around the sink as much as 1 meter away.

Assessing Education Needs

Typically completed prior to the educational session: Learner self-assessment Focus group discussion that is lead by the learners Interest-finder survey Test development - diagnostic practice tests Personal interviews Observational studies - HH audits Review of internal reports - sharps logs

What are the clinical features of RSV?

Upper Respiratory tract infection Including: 1. Sneezing 2. Rhinorrhea 3. Nasal Congestion 4. Fever Progression to the lower respiratory tract disease may result in bronchiolitis, tracheobronchitis, and pneumonia.

Which of the following methods of air sampling is most useful for detecting low numbers of fungal spores in areas that are highly filtered? a. Sieve impactor samplers b. Swabs of inanimate objects c. Particle counters d. Settle plates

a. Sieve impactor samplers

When an error does not result in an adverse event for a patient because the error was caught, it is called a(n): a. Adverse event b. No-harm event c. Near-miss event d. Error report

c. Near-miss event Rationale: A near-miss event is an unplanned event that could have resulted in injury, illness, or damage but did not, either by chance or through timely intervention. Reference: APIC Text, 4th edition, Chapter 18 - Patient Safety

The organism most likely to be found in an outbreak related to outpatient whirlpool wound therapy would be: a. Coagulate-negative Staphylococcus b. Streptococcus viridians c. Pseudomonas aeruginosa d. Hepatitis B Virus

c. Pseudomonas aeruginosa

Which of the following scenarios is an example of the preparation phase of the transtheoretical model? a. Auditing reveals that staff compliance with personal protective equipment (PPE) use has increased in the last month b. Data is showing a 96 percent compliance rate for PPE use in the last 10 months c. Staff are discussing strategies to increase PPE use at the monthly staff meeting d. Staff are questioning why they need to wear PPE when caring for patients in isolation

c. Staff are discussing strategies to increase PPE use at the monthly staff meeting Rationale: The principal concept behind the Transtheoretical Model (or Stage Theory) is readiness. For any given health-associated behavior, people will have diverse orientations to change. Some will be unaware that a particular change is a desirable option, whereas others will have already completed the change but remain at risk of reversing their progress or relapsing. The stages include Precontemplation, Contemplation, Preparation, Action, and Maintenance. The preparation phase is when a person or group of people starts planning for the behavior change. Reference: APIC Text, 4th edition, Chapter 5 - Infection Prevention and Behavioral Interventions CBIC Core Competency: Education and Research

Which of the following statements is true when the prevalence of a disease is very low? a. The sensitivity of a diagnostic test is greatly increased. b. The specificity of a diagnostic test is much greater. c. The negative predictive value (NPV) of a diagnostic test is very low. d. The positive predictive value (PPV) of a diagnostic test is lowered.

d. The positive predictive value (PPV) of a diagnostic test is lowered. PPV and NPV are disease prevalence dependent. Generally, a higher disease prevalence will INCREASE the PPV and DECREASE the NPV.

Process Measure

focuses on a process that leads to a certain outcome. Used to evaluate compliance with desired care or support practices. Examples: medication errors, influenza vaccination rates in personnel, TB Skin test, Hand Hygiene,

ethylene oxide

gas used to sterilize surgical instruments and other supplies. Gas sterilization is a low heat method for items that are not heat tolerant. Must be allowed time for aeration prior to use.

High-level disinfection

kills all microorganisms except spores and prions. Equipment that comes in contact with mucous membranes Dash such as respiratory therapy and anesthesia equipment, G.I. endoscopes, bronchial scopes, and laryngoscope blades must receive high level disinfection.

Differential media

make it easy to distinguish colonies of different microbes. Promotes specific organisms.

Sanitizing

the reduction in microbial population on an inanimate object to a safe or relatively safe level

RSV is apart of which virus family?

Paramyxovirus family

Fill in the blank: A _____________ is an assessment of the adequacy of respirator fit by numerically measuring the amount of leakage into the respirator. A user seal check is an action conducted by the respirator user to determine if the respirator is properly seated to the face.

Quantitative Fit Test (QNFT)

Randomized-controlled trial

Randomly selects a group of patients to receive a treatment and Another to receive placebo

The IP has worked with the Preoperative Services manager to plan and implement practices to help decrease the bioburden of microorganisms on patients' skin prior to planned surgeries. Which of the following might be included in their plan? 1) Active surveillance culturing for epidemiologically significant organisms 2) Preoperative showering using antimicrobial soap 3) Preoperative antibiotics given prior to the "cut time" 4) Treatment of remote site infections prior to surgery a. 1, 2, 3 b. 1, 2, 4 c. 2, 3, 4 d. 1, 3, 4

b. 1, 2, 4 Rationale: Prevention strategies to reduce SSIs include: • Administer antimicrobial prophylaxis in accordance with evidence-based standards and guidelines (within 1 hour prior to incision, 2 hours for vancomycin and fluoroquinolones). • Identify and treat remote infections before elective operation. Preoperative antibiotics are not given to reduce the microorganism bioburden prior to surgery. The following are examples of activities that may reduce the bioload of microorganisms on patients' skin: • Washing from clean to less-clean areas using clean washcloths to prevent cross-contamination • Preoperative showering using antimicrobial soap • Washing with antimicrobial soaps, such as chlorhexidine gluconate, to reduce carriage of resistant organisms, such as MRSA • Active surveillance culturing for epidemiologically significant organisms based on the facility's epidemiology • Encouraging or assisting patients in maintaining good oral hygiene and caring for the mouth to reduce the risk of mucositis in immunosuppressed persons • Encouraging good genital-area cleansing • Good hand hygiene practices using soap and water or alcohol-based hand rubs, as appropriate • Treatment of remote site infections prior to surgery • Additional personal risk-reduction strategies, include smoking cessation or weight loss if possible and appropriate Reference: APIC Text, 4th edition, Chapter 21 - Risk Factors Facilitating Transmission of Infectious Agents CBIC Core Competency: Preventing/Controlling the Transmission of Infectious Agents

An IP has data on the number of bloodstream infections per central line days in the Cardiac ICU. This is an example of which type of data? 1) Discrete data 2) Categorical data 3) Noncategorical data 4) Continuous data a. 1, 2 b. 1, 3 c. 1, 4 d. 2, 4

b. 1, 3 Rationale: Discrete data contain whole numbers and are mutually exclusive (e.g., infected or not infected, male or female, blood type). Discrete data can be categorical or noncategorical. - Categorical data can count both the number of events/occurrences and the number of nonevents/nonoccurrences (e.g., for 10 SSIs in 100 surgical cases, there are 10 events [SSIs] and 90 nonevents [no SSIs]). - Noncategorical data can count the events/occurrences but not the nonevents/nonoccurrences (e.g., number of patient falls per 1,000 patient days). With noncategorical data, the number at risk can be identified, but the actual number of "no infections" or "no falls" among those at risk cannot be identified. Continuous data contain information that can be measured on a continuum or scale and can have numeric values between the minimum and maximum value (a continuum) (e.g., age; serum cholesterol level; temperature, such as 98.6°F, 98.7°F, and 98.8°F; infection rates); continuous data require the process of measuring, rather than counting, and may contain whole numbers, decimals, or percentages. The type of data in this scenario is an example of both discrete data because it contains whole numbers and noncategorical data because it is only counting the events (i.e. infections), not the nonevents. Reference: APIC Text, 4th edition, Chapter 13 - Use of Statistics in Infection Prevention CBIC Core Competency: Surveillance and Epidemiologic Investigation

What are the 5 organisms known to be able to penetrate the intact epithelium of the conjunctiva or cornea?

1. Neisseria gonorrhoeae 2. Neisseria meningitidis 3. Streptococcus pneumoniae 4. Listeria monocytogenes 5. Corynebacterium diptheriae

Indications or Sterilization, High-level Disinfection, and Low-level Disinfection: CDC Recommendations

1) Before use on each patient, sterilize critical medical and surgical devices and instruments that enter normally sterile tissue or the vascular system or through which a sterile body fluid flows (e.g., blood). See recommendation 7g for exceptions. Category IA. 2) Provide, at a minimum, high-level disinfection for semicritical patient-care equipment (e.g., gastrointestinal endoscopes, endotracheal tubes, anesthesia breathing circuits, and respiratory therapy equipment) that touches either mucous membranes or nonintact skin. Category IA. 3) Perform low-level disinfection for noncritical patient-care surfaces (e.g., bedrails, over-the-bed table) and equipment (e.g., blood pressure cuff) that touch intact skin (see Recommendation 5g). Category II

What are the 4 leading causes of death associated with foodborne illnesses?

1. Nontyphoidal Salmonella spp (28%) 2. Toxoplama gondii (24%) 3. Listeria monocytogenes (19%) 4. Norovirus (11%)

What are the 4 leading causes of hospitalizations associated with foodborne illnesses?

1. Nontyphoidal Salmonella spp (35%) 2. Norovirus (26%) 3. Campylobacter spp. (15%) 4. Toxoplama gondii (8%)

Cleaning and Disinfecting Environmental Surfaces in Healthcare Facilities: CDC Decontamination of Spills of Blood or Other Potentially Infectious Materials Recommendations

1) For site decontamination of spills of blood or other potentially infectious materials (OPIM), implement the following procedures. Use protective gloves and other PPE (e.g., when sharps are involved use forceps to pick up sharps, and discard these items in a puncture-resistant container) appropriate for this task. Disinfect areas contaminated with blood spills using an EPA-registered tuberculocidal agent, a registered germicide on the EPA Lists D and E (i.e., products with specific label claims for HIV or HBV or freshly diluted hypochlorite solution. Category II, IC. a. If sodium hypochlorite solutions are selected use a 1:100 dilution (e.g., 1:100 dilution of a 5.25-6.15% sodium hypochlorite provides 525-615 ppm available chlorine) to decontaminate nonporous surfaces after a small spill (e.g., <10 mL) of either blood or OPIM. If a spill involves large amounts (e.g., >10 mL) of blood or OPIM, or involves a culture spill in the laboratory, use a 1:10 dilution for the first application of hypochlorite solution before cleaning in order to reduce the risk of infection during the cleaning process in the event of a sharp injury. Follow this decontamination process with a terminal disinfection, using a 1:100 dilution of sodium hypochlorite. Category IB, IC. 2) If the spill contains large amounts of blood or body fluids, clean the visible matter with disposable absorbent material, and discard the contaminated materials in appropriate, labeled containment. Category II, IC. 3) Use protective gloves and other PPE appropriate for this task

Cleaning of Patient-Care Devices: CDC Recommendations

1) In hospitals, perform most cleaning, disinfection, and sterilization of patient-care devices in a central processing department in order to more easily control quality. Category II. 2) Meticulously clean patient-care items with water and detergent, or with water and enzymatic cleaners before high-level disinfection or sterilization procedures. Category IB. a. Remove visible organic residue (e.g., residue of blood and tissue) and inorganic salts with cleaning. Use cleaning agents that are capable of removing visible organic and inorganic residues. Category IB. b. Clean medical devices as soon as practical after use (e.g., at the point of use) because soiled materials become dried onto the instruments. Dried or baked materials on the instrument make the removal process more difficult and the disinfection or sterilization process less effective or ineffective. Category IB. 3) Perform either manual cleaning (i.e., using friction) or mechanical cleaning (e.g., with ultrasonic cleaners, washer-disinfector, washer-sterilizers). Category IB. 4) If using an automatic washer/disinfector, ensure that the unit is used in accordance with the manufacturer's recommendations. Category IB. 5) Ensure that the detergents or enzymatic cleaners selected are compatible with the metals and other materials used in medical instruments. Ensure that the rinse step is adequate for removing cleaning residues to levels that will not interfere with subsequent disinfection/sterilization processes. Category II. 6) Inspect equipment surfaces for breaks in integrity that would impair either cleaning or disinfection/sterilization. Discard or repair equipment that no longer functions as intended or cannot be properly cleaned, and disinfected or sterilized. Category II

CDC's 10 Methods of Sterilization

1) Steam is the preferred method for sterilizing critical medical and surgical instruments that are not damaged by heat, steam, pressure, or moisture. Category IA. 2) Cool steam- or heat-sterilized items before they are handled or used in the operative setting. Category IB. 3) Follow the sterilization times, temperatures, and other operating parameters (e.g., gas concentration, humidity) recommended by the manufacturers of the instruments, the sterilizer, and the container or wrap used, and that are consistent with guidelines published by government agencies and professional organizations. Category IB. 4) Use low-temperature sterilization technologies (e.g., EtO, hydrogen peroxide gas plasma) for reprocessing critical patient-care equipment that is heat or moisture sensitive. Category IA. 5) Completely aerate surgical and medical items that have been sterilized in the EtO sterilizer (e.g., polyvinylchloride tubing requires 12 hours at 50°C, 8 hours at 60°C) before using these items in patient care. Category IB. 6) Sterilization using the peracetic acid immersion system can be used to sterilize heat-sensitive immersible medical and surgical items. Category IB. 7) Critical items that have been sterilized by the peracetic acid immersion process must be used immediately (i.e., items are not completely protected from contamination, making long-term storage unacceptable). Category II. 8) Dry-heat sterilization (e.g., 340°F for 60 minutes) can be used to sterilize items (e.g., powders, oils) that can sustain high temperatures. Category IB. 9) Comply with the sterilizer manufacturer's instructions regarding the sterilizer cycle parameters (e.g., time, temperature, concentration). Category IB. 10) Because narrow-lumen devices provide a challenge to all low-temperature sterilization technologies and direct contact is necessary for the sterilant to be effective, ensure that the sterilant has direct contact with contaminated surfaces (e.g., scopes processed in peracetic acid must be connected to channel irrigators). Category IB

What are the first 5 (A-H) suggested work restrictions for HCP exposed to or infected with an infectious disease in a healthcare setting, duration, and category from CDC?

1. Conjunctivitis = Restrict from patient care and contact with pt environment until discharge ceases = Category II 2. Diarrheal Disease = Restrict from pt contact, contact with pt environment, or food handling until symptoms resolve = Category IB 3. Diphtheria = exclude from duty until abx therapy completed and two negative cultures obtained 24hrs apart = Category IB 4.Eneroviral infections = restrict from infants until symptoms resolve = Category II 5. Hepatitis A = Restrict from pt contact, contact with pt environment and food handling until 7 days after jaundice = Category IB

Education Leadership Style (Situational Leadership)

1. Directing - Learner has low competence but high commitment. (a new hire) 2. Coaching - Learner has some competence but low commitment. (able to do the work, but its boring) 3. Supporting - Learner has moderate competence and variable commitment. (able to do the work but lacks confidence) 4. Delegating - Learner has high competence and high commitment. (able to do the task with confidence)

What are the 5 Infection Prevention objectives of an Occupational Health Program?

1. Educate HCP about principles of infection prevention and their individual responsibility for infection prevention. 2. Collaborate with the IP department in monitoring and investigating potentially harmful infectious exposures and outbreaks. 3. Provide care to personnel for work-related illnesses or exposures. 4. Identify work-related infection risks and institute appropriate preventative measures. 5. Contain costs by preventing infectious diseases that result in absenteeism and disability

What are rare complications of Influenza?

1. Encephalitis 2. Transverse myelitis 3. Reye's Syndrome 4. Myositis 5. Myocarditis 6. Pericarditis 7. Renal Failure

Match the definition to the appropriate Teaching Style: 1. Expert 2. Delegator 3. Demonstrator 4. Formal Authority 5. Facilitator A. IPs use their vast knowledge base to inform learners and challenge them to be well prepared. This can be intimidating to the learner. B. This style puts the IP in control of the learner's knowledge acquisition. The IP is not concerned with learner-educator relationships, but rather focuses on the content to be delivered. C. The IP coaches, demonstrates, and encourages a more active learning style. D. Learner-centered, active learning strategies are encouraged. The accountability for learning is placed on the learner. E. The IP role is that of a consultant and the learners are encouraged to direct the entire learning project.

1. Expert - A. IPs use their vast knowledge base to inform learners and challenge them to be well prepared. This can be intimidating to the learner. 2. Delegator - E. The IP role is that of a consultant and the learners are encouraged to direct the entire learning project. 3. Demonstrator - C. The IP coaches, demonstrates, and encourages a more active learning style. 4. Formal Authority- B. This style puts the IP in control of the learner's knowledge acquisition. The IP is not concerned with learner-educator relationships, but rather focuses on the content to be delivered. 5. Facilitator - D. Learner-centered, active learning strategies are encouraged. The accountability for learning is placed on the learner.

What are the 5 Teaching Styles and their respective role?

1. Expert = Transmitter of information 2. Formal Authority = Sets the standards 3. Personal Model = Teaches by direct examples 4. Facilitator = Guides by asking open ended questions 5. Delegator = Develops students' ability to function autonomously

What are the most important measures to prevent Respiratory Syncytial Virus (RSV) from spreading within a healthcare facility?

1. Hand Hygiene 2. Standard and Droplet Contact Precautions 3. Cohorting 4. Rapid diagnostic techniques

What 4 design elements does the FGI require the ICRA to meet?

1. Isolation rooms for protective environments and airborne infection isolaiton. 2. Special heating, air conditioning, or other ventilation needs 3. Water and plumbing system issues - The number, types, and locaiton of hand washing and disinfection stations, as well as emergency eye wash and shower stations. - Plumbing system design to address potential infection risks from waterborne pathogens 4. Selection of surface finishing and furnishing materials for infection prevention isolation rooms (general)

What are the second 6 (H-R) suggested work restrictions for HCP exposed to or infected with an infectious disease in a healthcare setting, duration, and category from CDC?

1. Measles = Exclude from duty (both active and post exposure for susceptible personnel) until 7 days after the rash appears or 5-21 days post exposure = Category IA and IB 2. Meningococcal infections = exclude from duty until 24hrs after effective therapy = Category IA 3. Mumps = Exclude from duty (both active and post exposure for susceptible personnel) until 9 days post onset of parotitis (swollen glands) or from 9th day after first exposure through 26th day after last exposure or until 9 days after onset of parotitis = Category IB and II 4. Pediculosis (lice) = Restrict from patient contact until treated and observed free from adult and immature lice = Category IB 5. Pertussis = Exclude from duty if symptomatic from beginning of catarrhal stage through third week after onset of paroxysms or until 5 days after start of effective antimicrobial therapy = Category IB 6. Rubella = Exclude from duty until 5 days after rash appears or from 7th day after first exposure through 21st day after last exposure = Category IA and IB

Match the Term with the Definition: 1. Potable Water 2. Gray Water 3. Black Water 4. Reverse osmosis water 5. Deionized water a. Wastewater containing sewage contaminants b. Purified water that has had charged ions removed. Used for various applications within healthcare, including the laboratory c. Water (as from a skink or bath) that does not contain serious contaminants (as from toilets or diapers) d. Water suitable for drinking e. Water is forced through a special membrane under pressure, which produces purified water. Typically requires remineralization with essential trace elements before use. Used for various applications within healthcare, including dialysis

1. Potable Water = d. Water suitable for drinking 2. Gray Water = c. Water (as from a skink or bath) that does not contain serious contaminants (as from toilets or diapers) 3. Black Water = a. Wastewater containing sewage contaminants 4. Reverse osmosis water = e. Water is forced through a special membrane under pressure, which produces purified water. Typically requires remineralization with essential trace elements before use. Used for various applications within healthcare, including dialysis 5. Deionized water = b. The purified water that has had charged ions removed. Used for various applications within healthcare, including the laboratory

What are the 8 Environmental sources of potential waterborne pathogens?

1. Potable water systems and cooling towers 2. Flush sinks 3. Faucet aerators 4. Hoppers and toilets 5. Eyewash/drench shower stations 6. Chests/Ice machines 7. Water baths used to thaw or warm blood products and their liquids 8. Whirlpool or spa-like baths

Gram Stain Procedure

1. Primary stain - Crystal violet- both cell walls affix the dye 2. Iodine Gram +: Dye complex trapped in wall Gram -: No effect of iodine 3. decolorizing agent - Ethanol or acetone Gram +: Crystals remain in cell wall Gram -: Ourter membrane weakend; wall loses dye 4. Counterstain -Safranin Gram +: Red dye masked by violet Gram -: Red dye stains the colorless cell

9 Water-associated Pathogens All of these genera are associated with serious disease and can potentially be transmitted directly or indirectly from faucets and skinks through inhalation or aerosols, such as those generated from construction activities or from showerheads

1. Pseudomonas 2. Acinetobacter 3. Moraxella 4. Aeromonas 5. Xanthomonas 6. Legionella 7. Aspergillus 8. Fusarium spp 9. Atypical (non-TB) Mycobacteria

What are the 5 key points of Specimen Collection and Transport?

1. Specimens should be collected aseptically and placed in a sterile container. 2. Some specimens may be placed directly on culture media (eg blood cultures, genital cultures) 3. Special handling techniques may be necessary for some specimens (eg anaerobic cultures) 4. Prompt delivery is essential 5. If transport is delayed, some specimens may be refrigerated: - Urine - stool - Sputum Others should be maintained at room temperature: - Genital - Eye - Spinal fluid

What are the 5 steps required for specimen collection and transport?

1. Specimens should be collected aseptically, placed in a sterile container. 2. Some specimens may be placed directly on culture media (eg blood cultures, genital cultures) 3. Special handling techniques may be necessary for some specimens (eg anaerobic cultures) 4. Prompt delivery essential 5. If transport is delayed, some specimens may be refrigerated (eg urine, stool, sputum); others should be maintained at room temperature ( eg genital, eye, spinal fluid).

What organisms are the most common cause of bacterial coinfection with Influenza?

1. Staphylococcus aureus 2. Streptococcus pneumoniae 3. Streptococcus pyogenes

Evaluation of a training program should include what 3 things?

1. The appropriateness of program design; 2. Adequacy of teaching and resources; 3. and knowledge, skills, and attitudes learned by the participants.

Superficial SSI

1. The infection must occur within 30 days of the operation AND involve only the skin or subcutaneous tissue. 2. Purulent drainage 3. Organism growth from the drainage AND 4. Superficial opening of the incision by the surgeon OR pt has pain or s/s of infection, or diagnosis of SSI by surgeon, attending, or designee.

Match the Isolation requirement to the corresponding virus, disease, or bacteria: Contact - Droplet - Airborne - Standard 1. Variola Major 2. Neisseria meningitides 3. Influenza 4. VRE 5. MRSA 6. Clostridium difficile 7. Bordetella pertussis 8. Bacteria meningitis 9. Measles 10. TB 11. SARS 12. Fungal meningitis

1. Variola Major (Smallpox)= Airborne 2. Neisseria meningitides = Droplet 3. Influenza = Droplet 4. VRE = Contact 5. MRSA = Contact 6. Clostridium difficile = Contact Enteric 7. Bordetella pertussis = Droplet 8. Bacteria meningitis = Droplet 9. Measles = Airborne 10. TB = Airborne 11. SARS = Airborne 12. Fungal meningitis = Standard

Acid-Fast Stain

1. apply primary stain- carbolfuchsin for 30 seconds 2. Heat fix cells to slide using flame 3. decolorize with acid-alcohol 15-20 seconds -non acid-fast bacteria decolorize, acid-fast remain pink 4. Apply counterstain- methylene blue (30 s) rinse w/ h20 -provides contrast entre acid-fast and background material -acid-fast are pink, non acid-fast are blue

Incident Rate Formula

= ((#of new cases)/ (population at risk))x100

Health Belief Model

A theory of health behaviors; the model predicts that whether a person practices a particular health habit can be understood by knowing the degree to which the person perceives a personal health threat and the perception that a particular health practice will be effective in reducing that threat. "The benefits of the behavior out weigh the barriers."

Ideal temperature for DPT storage is: A. 2 to 8 degrees Celsius B. 16-20 degrees Celsius C. 8-12 degrees Celsius D. Room temperature

A. 2-8 degrees Celsius Store diphtheria, tetanus, and pertussis vaccines are refrigerated between 2°C and 8°C. Do not freeze vaccine, or exposed to freezing temperatures. If the vaccine has been exposed to inappropriate conditions/temperatures or handled improperly, store the vaccine at the appropriate temperature, isolate from other vaccines, mark "do not use", and consult the vaccine manufacture and/or your state or local immunization program for guidance.

When interpreting the HICPAC/CDC System for categories and recommendations, which category is strongly recommended for implementation and strongly supported by well-designed experimental, clinical, or epidemiologic studies. A. Category IA B. Category IB C. Category IC D. Category II

A. Category IA

What type of meningitis would be most consistent with the following cerebrospinal fluid (CSF) report result: Glucose: Decreased Protein: Elevated WBC counts: 1,000/mm^3 A. Bacterial B. Viral C. Fungal D. Tuberculosis

A. Bacterial Rationale: Culture of blood and CSF are indicated for patients with suspected invasive meningococcal disease. The CSF of patients with untreated meningococcal meningitis is usually cloudy and has pleocytosis with a predominance of neutrophils, low glucose, and high protein levels. In most of the cases, the organisms are seen on Gram stain or can be identified using latex agglutination assays. The culture is almost invariably positive as long as the sample was obtained before the administration of antibiotics. Reference: APIC Text, 4th edition, Chapter 74 - Central Nervous System Infection CBIC Core Competency: Identification of Infectious Disease Processes

Which of the following recommendations related to disinfection and sterilization in healthcare facilities is a CDC category 1A recommendation? 1) "Before use on each patient, sterilize critical medical and surgical devices and instruments that enter normally sterile tissue or the vascular system or through which a sterile body fluid flows" 2) "Meticulously clean patient-care items with water and detergent, or with water and enzymatic cleaners before high-level disinfection or sterilization procedures" 3) "In hospitals, perform most cleaning, disinfection, and sterilization of patient-care devices in a central processing department in order to more easily control quality" 4) "Perform low-level disinfection for noncritical patient-care surfaces (e.g., bedrails, over-the-bed table) and equipment (e.g., blood pressure cuff) that touch intact skin" A. 1 B. 1, 3 C. 1, 2, 4 D. 1, 2, 3, 4

A. 1 Rationale: The CDC has established a system for cataloging recommendations based on the amount of data available to support the recommendation. Category 1A recommendations are strongly supported by epidemiologic, clinical data, or experimental data from well-designed studies. Sterilization of medical instruments that will come into contact with sterile tissue or the vascular system is a Category 1A recommendation. Reference: CDC Guideline for Disinfection and Sterilization in Healthcare Facilities, 2008 Page: 83-84 CBIC Core Competency: Cleaning, Sterilization, Disinfection, Asepsis

An Association for Professionals in Infection Control and Epidemiology (APIC) chapter would like to conduct a needs assessment to determine the educational interest of its members. What should a thorough needs assessment accomplish? 1. Develop course goals and objectives 2. Identify knowledge gaps among targeted learners 3. Determine the outcome of the educational activity 4. Provide precise information about instructor performance A. 1,2 B. 2,3 C. 3,4 D. 2,4

A. 1,2 1. Develop course goals and objectives 2. Identify knowledge gaps among targeted learners Needs assessments identify gaps or deficiencies in knowledge, skills, or attitude and serve as the basis for educational program development. Assessing the educational needs of the learner population, the institution, and the community, as it relates to infection prevention, is the first step in effective program planning. Reference: APIC Text, 4th edition, Chapter 3 - Education and Training

When developing a strategic plan, using the TJC requirements, you want to: 1) Prioritize the identified risk for acquiring and transmitting infections 2) Set goals that include limiting unprotected exposures to pathogens 3) Describe the activities to minimize infection and the process to evaluate the plan 4) Compare the previous fiscal year goals with the new fiscal year goals A. 1,2,3 B. 1,3,4 C. 2,3,4 D. 1,2,4

A. 1,2,3 Rationale: Healthcare quality improvement uses interdisciplinary teams to deploy changes and improvements. A quality-focused culture values the knowledge, skills, and expertise of the frontline workers and other professionals to use innovation, scientific methods, and effective change management approaches to improve healthcare services and ensure patient safety. When developing a strategic plan using the TJC requirements, you want to: 1) Prioritize the identified risk for acquiring and transmitting infections 2) Set goals that include limiting: - Unprotected exposures to pathogens - The transmission of infections associated with procedures - The transmission of infections associated with the use of medical equipment, devices, and supplies 3) Describe activities—including surveillance—to minimize, to reduce or eliminate the risk of infection 4) Describe the process to evaluate the infection prevention control plan

What of the following questions should be asked when evaluating results from a research study? 1) Were the instruments valid for the study? 2) Is this a peer-reviewed research journal? 3) Was the sample representative of the intended population? 4) Do the conclusions prove the hypothesis? A. 1,2,3 B. 2,3,4 C. 1,3,4 D. 1,2,4

A. 1,2,3 Rationale: Many factors should be considered in critically reviewing an article in the scientific literature. To evaluate articles that report original research, the reader should ask certain questions about each component of the paper. The following questions may serve as a basic guide: • Introduction: Is the study question important, appropriate, and stated clearly? • Materials and methods: Is the study population appropriate and adequately described? Is the choice of study design applicable to the purpose of the study? Are selection and exclusion criteria described? Were outcomes of groups evaluated equally and by persons blinded to the study treatment arm? What were the proportions lost to follow-up in each study arm described? • Results: Are the statistical tests appropriate for the study design? Is the sample size adequate? Are there factors that could have confounded results and were these taken into account? Do the data that are presented in the text, tables, and figures provide an answer to the stated research question(s)? • Discussion: Are the conclusions that are drawn reasonable and justified? Could other explanations account for the observed results? References: APIC Text, 4th edition, Chapter 20 - Research Study Design; APIC Text, 4th edition, Chapter 19 - Qualitative Research Methods

The following blood culture result should be considered a potential contaminant: A. A positive result of coagulase-negative staphylococci from two sets, 2 days apart, without symptoms B. A positive result of S. aureus from one bottle in a patient with a temperature of 38.6°C C. A positive result of E. coli from one bottle in an afebrile patient with diarrhea D. A positive result of Candida albicans in a fungal blood culture in a patient with a urinary tract infection

A. A positive result of coagulase-negative staphylococci from two sets, 2 days apart, without symptoms Rationale: According to the CDC CLABSI criteria, common commensals (such as coagulase-negative staphylococci) meet the criteria for a CLABSI if there are two positive cultures from two or more sets of blood cultures drawn less than 2 days apart and the patient has symptoms (fever greater than 38°C, chills, or hypotension). CDC CLABSI criteria: Patient has at least one of the following signs or symptoms: - fever (greater than 38°C), - chills, or hypotension and - positive laboratory results are not related to an infection at another site. And the same common commensal (i.e., diphtheroids [Corynebacterium spp. not C. diphtheriae], Bacillus spp. [not B. anthracis], Propionibacterium spp., coagulase-negative staphylococci [including S. epidermidis], viridans group streptococci, Aerococcus spp., and Micrococcus spp.) is cultured from two or more blood cultures drawn on separate occasions. Criterion elements must occur within a time frame that does not exceed a gap of 1 calendar day between two adjacent elements. Reference: Centers for Disease Control and Prevention (CDC). Patient Safety Component Manual: Device-associated module - Central line-associated bloodstream infection (CLABSI) event. CDC website. January 2014. Available at: http://www.cdc.gov/nhsn/PDFs/pscManual/4PSC_CLABScurrent.pdf CBIC Core Competency: Identification of Infectious Disease Processes

Developing a vision and mission, identifying and organizations external opportunities and threats, and determining enternal strengths and weaknesses are all a part of which of the following activities? A. Strategy formulation B. Strategy implementation C. Long range planning D. Short range planning

A. A strategy formulation Strategy formulation is the process by which an organization chooses the most appropriate course of action to achieve is to find goals. This process is essential to an organization success, because it provides a framework for the actions that will lead to the anticipated results. Strategic plan should be communicated to all employees so that they are aware of the organizations objectives, mission, and purpose. Strategy formulation force is an organization to carefully look at the changing environment and to be prepared for the possible changes that may occur. Strategic plan also enables an organization to evaluate its resources, allocate budgets, and determine the most effective plan for maximizing return on investment.

Which of the following refers to patient harm that is the result of treatment by the healthcare system rather than from the health condition of the patient? A. Adverse event B. Dire consequence C. Unanticipated event D. Sentinel event

A. Adverse Event

An urban community is experiencing an outbreak of Bordetella pertussis. Several employees have contacted the IP at their healthcare facility for information on tetanus, diphtheria, and pertussis (Tdap) vaccine. They question the need for the vaccine because they received it as a child. The IP should inform them that the Advisory Committee for Immunization Practices (ACIP) recommends that: A. All adults aged 19 and older should receive at least one dose of Tdap. B. If the employee is pregnant, they should not receive the vaccine. C. All individuals must receive the vaccine every 10 years. D. Individuals who previously had the disease do not need to receive the vaccine.

A. All adults aged 19 and older should receive at least one dose of Tdap. Rationale: Pertussis (whooping cough) is a highly communicable, acute, infectious respiratory disease caused by Bordatella pertussis. ACIP recommends a single Tdap dose for persons aged 11-18 years who have completed the recommended childhood diphtheria and tetanus toxoid and pertussis/Diptheria and tetanus toxoid and acellular pertussis (DTP/DTap) vaccination series for adults aged 19-64 years. Pregnant individuals should receive a dose of TDaP for every pregnancy. Individuals should receive the tetanus vaccine every 10 years - Clostridium tetani

The IP must prepare a cost-benefit analysis (CBA) for the proposed purchase of a new electronic surveillance system for the hospital. Which statement about the CBA is correct? A. All measures are expressed in monetary (US dollar) terms B. Only costs are expressed in monetary (US dollar) terms C. CBA is never based on existing financial data D. The monetary value of possible benefits is often highly speculative

A. All measures are expressed in monetary (US dollar) terms Rationale: CBA is a systematic method of estimating the strengths and weaknesses of alternatives that satisfy transactions, activities, or functional requirements for a business. It is a technique that is used to determine options that provide the best approach for adoption and practice in terms of benefits such as labor, time, and cost savings. CBAs can help determine which alternative is a sound investment or decision by comparing the total expected cost of each option against the total expected benefits. The comparison shows whether the benefits outweigh the costs and by how much. Reference: APIC text, 4th edition, CHapter 1 - IP Programs

Improved hydrogen peroxide contains: A. Anionic and/or nonionic surfactants B. Glutaraldehyde C. Peracetic acid D. Chlorine

A. Anionic and/or nonionic surfactants Rationale: Improved hydrogen peroxide contains very low levels of anionic and/or nonionic surfactant in an acidic product that act with hydrogen peroxide to produce microbial activity. This combination of ingredients speeds the antimicrobial activity of hydrogen peroxide and clean efficiency. Reference: APIC Text, 4th edition, Chapter 31 - Cleaning, Disinfection, and Sterilization CBIC Core Competency: Cleaning, Sterilization, Disinfection, Asepsis

Which Patient Safety culture evolution is related to having systems in place to manage all identified risks? A. Bureaucratic B. Proactive C. Generative D. Reactive

A. Bureaucratic Rationale: There are 5 evolutions of the Patient Safety Culture: 1. Pathological= Why waste our time on safety? 2. Reactive = We do something when we have an incident 3. Bureaucratic = We have systems in place to manage all identified risks 4. Proactive = We are always on the alert for the risks that might emerge 5. Generative = Risk management is an integral part of everything we do

A study design in which a population of individuals with and without exposure to a potential risk factor are identified and followed to compare the incidence of the outcome in each group would be: A. Cohort B. Case control C. Descriptive D. Analytical cross sectional

A. Cohort A study design where one or more samples (cohorts) are followed prospectively and subsequent status evaluations with respect to the disease or outcome are conducted to determine which initial participants exposure characteristics (risk factors) are associated with it. As the study is conducted, outcomes from participants in each cohort is measured in relationships with specific characteristics determined.

DMAIC refers to a data-driven quality strategy for improving processes, and is an integral part of the Six Sigma Quality Initiative. The DMAIC format includes all of these except: A. Control the improvements only until problems are no longer prevalent B. Improve the target process by designing creative solutions to fix and prevent problems C. Define the customer, project boundaries and improvement process D. Measure the performance of the process involved

A. Control the improvements only until problems are no longer prevalent Rationale: DMAIC (an abbreviation for Define, Measure, Analyze, Improve, and Control) refers to a data driven improvement cycle used for improving, optimizing and stabilizing business processes and designs. The DMAIC improvement cycle is the core tool used to drive Six Sigma projects. The purpose of control step is to sustain the gains of the process and to monitor the improvements to ensure continued and sustainable success.

The purpose of the annual infection prevention program risk assessment is to: A. Determine goals and objectives for the following year B. Describe support requirements of the program C. Outline the achievements and activities of the program D. Document the facility's risks of infection

A. Determine goals and objectives for the following year Rationale: An annual risk assessment must be performed to determine goals and objectives for the infection prevention program. These should be based on the institution's strategic goals and institutional data and findings from the previous year's activities. Infection Prevention resources and data systems needs should be evaluated in the context of these goals and objectives. Reference: APIC text, 4th edition, Chapter 1 - Infection Prevention Control Programs

A technician finds out after obtaining an EKG on a patient that the patient may have varicella-zoster (shingles) on a dermatome on the upper body. The Occupational Health Nurse checks the employee's records and realizes that the employee was never tested for varicella on hire. The first thing the Occupational Health Nurse should do is: A. Determine if the patient actually has an active case of varicella-zoster by involving IP or checking with the patient's physician to verify the diagnosis. B. Test the employee for varicella immunity and, if not immune, exclude from work from day 10 through day 21 after the exposure. C. Give the varicella vaccine to the employee. D. Give varicella-zoster immune globulin (VZIG) to the employee.

A. Determine if the patient actually has an active case of varicella-zoster by involving IP or checking with the patient's physician to verify the diagnosis.

Over the past 6 months, you have been following all the infections in the ICU. In your surveillance, you note an increasing time period between the time an antibiotic is ordered and the time it is first administered. Which action would follow the quality/performance improvement process? A. Develop an interdisciplinary team to look at the process and recommend possible improvements B. Have administration send the director a memorandum C. Continue to monitor it another 6 months and report results to the director D. Send an incident report to management and a copy to the department manager

A. Develop an interdisciplinary team to look at the process and recommend possible improvements Rationale: Performance improvement is an ongoing continuous cycle that focuses on patient clinical outcomes, customer satisfaction, and service. Program interventions determine effectiveness and efficiency and can determine whether proactive approaches or t=retrospective analysis can further improve program quality. Interdisciplinary teams are used to deploy changes and improvements. Reference: APIC Text Online, Chapter 16 - Quality Concepts

Fecal oral spread of hepatitis a virus is an example of A. Direct contact transmission B. Indirect contact transmission C. Droplet transmission D. Vehicle transmission

A. Direct contact transmission Hepatitis A is a liver disease caused by the hepatitis a virus. The hepatitis a virus is transmitted through ingestion of contaminated food and water or through direct contact with an infectious person. Direct contract transmission occurs when there is physical contact between an infected person and a susceptible person.

There are three major domains in the learning process: Cognitive, Affective, and Psychomotor. In the affective learning domain, the individual: A. Embraces new attitudes, values and beliefs in the learning process B. Demonstrates new skills or new ways of acting or doing C. Uses recall application and analytic skills in learning processes D. May demonstrate conflict and resistance to the learning

A. Embraces new attitudes, values and beliefs in the learning process The affective domain the domain that deals with attitudes, motivation, willingness to participate, valuing what is being learned, and ultimately incorporating the values of a discipline into a way of life. Stages in this domain have been described as the following: • Receiving (willing to listen) • Responding (willing to participate) • Valuing (willing to be involved) • Organizing (willing to be an advocate) • Characterization (willing to change one's behavior, lifestyle, or way of life) Reference: APIC Text, 4th edition, Chapter 3 education and learning

A patient with a confirmed diagnosis of varicella (chickenpox) is seen a busy ED. The staff at the registration desk immediately placed a mask on the patient until he could be moved to a negative airflow room in the ED, where he then removed his mask. A pregnant environmental services (EVS) employee, who is nonimmune to varicella, enters the room before an isolation sign is posted. She spends 6 minutes in the room with the patient, who is no longer wearing a mask. Which of the following is the most appropriate postexposure response? A. Give the EVS employee VZIG and place off work from day 10 through day 28 B. Administer the varicella vaccine immediately and place employee off work from day 10 through day 21 C. Have the employee wear a mask from day 10 through 21 after the exposure and watch for signs and symptoms of disease D. Give the EVS employee the vaccine and VZIG

A. Give the EVS employee VZIG and place off work from day 10 through day 28

You have isolated a bacterium from your skin. Chemical analysis shows that it contains proteins, peptidoglycan, lipids, DNA, and teichoic acid. What sort of bacteria is this? A. Gram-Positive B. Gram-Negative C. Cannot tell from this information D. Non-bacterial entity

A. Gram-Positive

Which of the following is the correct order for disinfecting endoscopes? A. High level disinfection, rinse scope with sterile water, flush the channels with 70-90% alcohol, dry using forced air. B. Rinse the scope using sterile water, high level disinfection, flush the channels with 70-90% alcohol, dry using forced air. C. Flush the channels with 70-90% alcohol, rinse the scope using sterile water, high level disinfection, dry using forced air. D. Dry using forced air, high level disinfection,, flush the channels with 70-90% alcohol, rinse the scope using sterile water.

A. High level disinfection, rinse scope with sterile water, flush the channels with 70-90% alcohol, dry using forced air.

Which of the following is an example of the criterion of "Strength of the Association" from Hill's criteria for causation? A. In a study of the association between antibiotic exposure and development of C. difficile infection, the odds ratio was 2:3 B. In a study of the association between antibiotic exposure and development of C. difficile infection, the authors' conclusions are consistent with those of three other studies C. In a study of the association between antibiotic exposure and development of C. difficile infection, antibiotic therapy began an average of 3 weeks before C. difficile infection developed D. In a study of the association between antibiotic exposure and development of C. difficile infection, prolonged antibiotic therapy was a greater risk factor for C. difficile infection than short-term antibiotic therapy

A. In a study of the association between antibiotic exposure and development of C. difficile infection, the odds ratio was 2:3 Rationale: Causal associations exist when evidence indicates that one factor is clearly shown to increase the probability of the occurrence of a disease. In a causal relationship, the reduction or diminution of a factor decreases the frequency of the disease being studied. The criteria currently used for causality were developed by Austin Bradford Hill and are known as Hill's criteria. These criteria use modern epidemiological methods to determine whether a factor is causal for a given disease. Strength of association is the first criterion: The incidence of disease should be higher in those who are exposed to the factor under consideration than in those who are not exposed; that is, the stronger the association between an exposure and a disease, the more likely the exposure is to be causal. The odds ratio is a statistical measure that gives us an indication of how strongly the risk factor is associated with the disease outcome. Reference: APIC Text, 4th edition, Chapter 10 - General Principles of Epidemiology CBIC Core Competency: Education and Research

An example of pneumonia generally acquired from an environmental source is: A. Legionella B. Pneumococcal C. Meningococcal D. Varicella

A. Legionella Legionella is a type of bacterium found in naturally in freshwater environment, like lakes and streams. It can become a health concern when it grows and spreads in human made water systems like: Showers and Fossetts Cooling towers for air-conditioning units in large buildings Hot tubs that aren't drained after each use Hot water tanks and heaters Large plumbing systems

A patient has been admitted with a wound infection. The lab reports that the stain of the wound culture is positive for AFB and the culture is positive for Mycobacterium marinum. Which of the following is the correct follow-up of this lab report? A. No further follow-up is needed B. Place the patient on airborne isolation C. Contact the local health department to report the Mycobacterium infection D. Place the patient on Contact Precautions

A. No further follow-up is needed Rationale: M. marinum is a bacterium found in water and may be the cause of wound infections. Infection with M. marinum is not transmissible, and infection with non-tuberculosis mycobacteria is not notifiable. No further infection control follow-up is needed. References: Appendix A. In: Siegel J, Rhinehart E, Jackson M, et al. 2007 Guideline for Isolation Precautions: Preventing Transmission of Infectious Agents in Health Care Settings. Available at: http://www.cdc.gov/hicpac/pdf/isolation/isolation2007.pdf; APIC Text, 4th edition, Chapter 95 - Tuberculosis and Other Mycobacteria CBIC Core Competency: Preventing/Controlling the Transmission of Infectious Agents

If your only consideration in choosing a chemical disinfectant was to select one that does not irritate the respiratory system, which of the following would you choose? A. Ortho-phthalaldehyde (OPA) B. Ethylene oxide (ETO) C. Glutaraldehyde D. Sodium hypochlorite

A. Ortho-phthalaldehyde (OPA) Ortho-phthalaldehyde (OPA) is a high-level disinfectant that does not have a significant odor and generally does not cause respiratory irritation. Drawbacks of OPA are that it can cause staining if it come in contact with skin, clothes, or surfaces and it is relatively expensive. Reference: Guideline for Disinfection and Sterilization in Healthcare Facilities, 2008 Rutala WA, Weber DJ, Healthcare Infection Control Practices Advisory Committee

Which of the following organisms is most likely to be associated with contaminated disinfectants? A. Pseudomonas species B. Hepatitis C C. Clostridium difficile D. Cryptosporidium parvum

A. Pseudomonas species Always store disinfectants per IFUs.

An increase in the number of CLABSI events are noted in the oncology ward during the last quarter. The IP speaks to the oncology staff, reviews the charts, and observes central line management practices on the unit. This scenario is best reflected by which quality tool? A. Root cause analysis B. Fishbone C. Strengths, weaknesses, opportunities, and threats D. Ishikawa diagram

A. Root cause analysis Root cause analysis is a process for identifying the basic or causal factors underlying variation in performance. A root cause analysis focuses primarily on systems and processes, not individual performance.

The emergency department has seen an increase in patients with complaints of diarrhea, fever, abdominal cramps and vomiting. An investigation reveals all of the patients went to a holiday chili cook off three days ago and ate from vendor X. Choose the most likely culprit: A. Salmonella spp B. Clostridium botulinum C. Listeria monocytogenes D. Staphylococcus aureus

A. Salmonella spp Most persons infected with salmonella bacteria develop diarrhea, fever, and abdominal cramps 12 to 72 hours after infection. The illness usually last 4 to 7 days, and most persons recover without treatment.

An IP wants to increase awareness of best practices regarding common infection prevention in fractions during the care process. Which instructional method could be used to deliver this training? A. Simulation B. Educational cart C. Positive deviance D. Case studies

A. Simulation Simulation-based education provide students with opportunities to practice their clinical and decision making skills through various real life situational experiences. The advantages of simulation-based education interventions include the ability to provide immediate feedback, repetitive practice learning, the integration of simulation into the curriculum, the ability to adjust the difficulty level, opportunities at individualized learning, and the adaptability to the first types of learning strategies

Healthcare-associated Herpes Simplex Virus (HSV) is most likely to manifest as a herpetic whitlow and is prevented by the use of what isolation, PPE, and which bodily fluid? A. Standard, gloves, mucous membranes B. Contact, gloves, blood C. Droplet, mask, saliva D. Droplet Contact, gloves, vaginal secretion

A. Standard, gloves, mucous membranes No vaccine is currently available for HSV, and prevention is by adhering to Standard Precautions. HA-HSV infection is most likely to manifest as a herpetic whitlow, is prevented by the use of gloves for contact with mucous membranes, and is unlikely to affect the genital tract or the fetus. HCP with herpetic whitlow should be excused from duty. Transmission of HSV occurs by intimate contact, such as oral-oral, oral-genital, and genital-genital contact. Transmission by fomites contaminated by body fluids has been documented. Humans are the sole natural host for HSV, and 55-90% of the general population has antibodies to HSV-1, HSV-2, or both. Herpetic whitlow is a viral condition where small blisters form on the fingers and the fleshy area around the fingertips. These sores or blisters are often painful and develop after direct contact with a contagious sore. The herpes simplex virus (HSV) causes this condition.

You are reviewing the serology marker results of a healthcare worker. The Hepatitis B serology marker results are as follows: HBsAg = Negative (-) anti-HBc = Negative (-) anti-HBs= Negative (-) What is the Hepatitis B status of this healthcare worker? A. Susceptible B. Immune due to natural infection C. Acutely infected D. Chronically infected

A. Susceptible Rationale: Hepatitis B Surface Antigen (HBsAg) = The presence of HBsAg indicates that the person is infectious (regardless of acute or chronic). It is also used to make the vaccine. Total Hepatitis B Core Antibody (anti-HBc) = Appears at the onset of symptoms in acute HBV and persists for life. Its presence indicates previous or ongoing infection. Hepatitis B Surface Antigen Antibody (anti-HBs)= The presence of anti-HBs is generally interpreted as indicating recovery and immunity form Hepatitis B. Reference: CDC Interpretation of Hepatitis B Serologic Test Results table www.cdc.gov/hepatitis

During an in-service program for new employees, the IP describes how Hepatis B and HIV are transmitted. A major difference in the epidemiology of the two diseases is: A. The ease of transmission through needle punctures B. The potential of airborne transmission C. Presence of the causative agent in various body fluids D. The ability of the disease to be transmitted during sexual intercourse

A. The ease of transmission through needle punctures Prevention of occupational blood exposures is the primary way to reduce transmission of Hepatitis B virus (HBV), Hepatitis C (HCV), and HIV among HCW. The risk of HBV seroconversion after a percutaneous injury ranges from 23-64%. The average risk of seroconversion after a percutaneous injury involving blood infected with HIV is significantly lower, at 1.8%.

A new employee to your facility is foreign-born and received the bacillus Calmette-Guerin (BCG) vaccine as a child. Her pre-employment assessment for TB should include: A. Two-step PPD as any other employee would receive B. No purified protein derivative (PPD) due to BCG vaccine C. Chest x-ray to rule out active TB D. One PPD due to BCG vaccine

A. Two-step PPD as any other employee would receive A TB skin test (TST) should be administered to all new employees, then read, and interpreted by trained personnel. Use the intradermal (Mantoux) method to administer the PPD-based TST. Tine tests should not be used. Baseline screening should be conducted at the time of hire.

An employee is exposed to blood and body fluids from a patient whose baseline testing revealed positive results in a rapid HIV test. The most appropriate follow-up test for the patient would be: A. Western Blot B. Viral Load C. HIV polymerase chain reaction D. CD4

A. Western Blot Rationale: Rapid HIV tests can offer a result in 15 minutes. A negative rapid test is reported as a definitive negative HIV test. A reactive test result needs to be confirmed with standard serologic tests. The most commonly used laboratory test for diagnosis of HIV infection is the serologic detection of antibodies to the virus. The standard serologic test consists of a screening enzyme immunoabsorbent assay (EIA) followed by a confirmatory Western blot (WB). In a patient with a positive EIA test, the test should be repeated. In a patient with a repeatedly positive EIA test, a confirmatory WB is performed. These serologic assays show sensitivity and specificity rates of 99.9%. Reference: APIC Text, 4th edition, Chapter 81 - HIV/AIDS CBIC Core Competency: Identification of Infectious Disease Processes

You are the infection prevention manager and are using the evidence-based practice process to reduce CAUTI rate in your ICU. You meet with the unit staff nurses for input. Which question would you post to the staff to reflect the first step of the EBP process? A. What is the exact clinical issue with which we are dealing? B. Is anyone prepared to evaluate the statistics we will find? C. What outcomes are we expecting from this work? D. Where can we look for information?

A. What is the exact clinical issue with which we are dealing? The evidence-based practice process has five steps: 1. Ask important questions about the care of individuals, communities, or populations. 2. Acquire the best available evidence regarding the question. 3. Critically appraise the evidence for validity and applicability to the problem at hand. 4. Apply the evidence by engaging in collaborative health decision making with the affected individuals and/or groups. Appropriate decision making it integrates the context, values and preferences of the care recipient, as well as available resources, including professional expertise. 5. Assesses the outcome and disseminate the results.

artifactual association

Artifactual association is a false association that can be due to chance or bias in a study method

An example of an obligate intracellular parasitic bacterium would be an organism responsible for: 1) Hepatitis 2) Q fever 3) Malaria 4) Chlamydia A. 2,3 B. 2,4 C. 3,4 D. 1,2

B. 2,4 Rationale: Obligate intracellular parasites cannot reproduce outside their host cell, meaning that the parasite's reproduction is entirely reliant on intracellular resources. Obligate intracellular parasites of humans include: - Viruses - Bacteria - Chlamydia - Rickettsia (typhus) - Coxiella (Q fever) - Some Mycobacteria spp - Protozoa - Apicomplexans - Plasmodium spp - Toxoplasma gondii - Cryptosporidium parvum - Trypanosomatids - Leishmania spp - Trypanosoma cruzi - Fungi - Pneumocytis jiovecii

Diluted bleach solutions stored in an open container must be changed every: A. 28 days B. 24 hours C. 12 hours D. after every use

B. 24 hours

Inadequate refrigeration of food may permit the growth of potentially harmful microbes. Interventions to prevent the growth of pathogens due to inadequate refrigeration include: 1) Train personnel to recognize and implement safe maintenance of refrigerators 2) Establish a method to record temperature on a regular basis 3) Make daily rounds to ensure that the freezer and refrigerator are clean 4) Schedule and perform regular preventive maintenance of freezer and refrigerator A. 1, 2, 3 B. 1, 2, 4 C. 2, 3, 4 D. 1, 3, 4

B. 1, 2, 4 Rationale: Interventions to prevent the growth of microbes include: • When selecting/purchasing equipment for cooling or freezing, compare features that best meet the intended use, including operating range (e.g., an automatic defrost cycle can damage temperature-sensitive items), size, location of use, cleanable surfaces, durability, and maintenance needs. • Provide accurate temperature monitoring for refrigerators and freezers; an alarm system may be required (e.g., blood bank refrigerator) or desired. • Establish a method to record temperature on a regular basis (e.g., visualize and document daily or observe an automated recording chart each shift); include action to take if reading is not in the acceptable range. • Schedule routine monitoring of refrigerator and freezer alarms where applicable. • Test accuracy of thermometers; calibration may be required (e.g., blood bank, tissue freezer) by using standard regulations and/or recommendations. • Schedule and perform regular preventive maintenance of all freezers and refrigerators; include air vents, gaskets, cooling coils, and fans. • Walk-in refrigeration units may experience a condensation point if the building dehumidification is inadequate, resulting in mold proliferation. • Provide training for personnel in recognizing and implementing safe maintenance of refrigerators to include appropriate cleaning methods. Reference: APIC Text, 4th edition, Chapter 112 - Maintenance and Engineering CBIC Core Competency: Preventing/Controlling the Transmission of Infectious Agents

A case of healthcare-associated Mycobacterium chelonae respiratory infection has been identified in a patient. Of the following exposures that the patient had in the facility, which should be investigated as a potential source of the infection? 1) Bronchoscopes 2) Dialysis 3) Hydrotherapy pool 4) Ice from tap water used during surgery A. 1 B. 1, 3 C. 1, 3, 4 D. 1, 2, 3, 4

B. 1, 3 Rationale: Mycobacterium chelonae belongs to the family of nontuberculous mycobacteria (NTM) classified in the rapidly growing mycobacteria (RGM), Runyon group IV. M chelonae are ubiquitous in the environment and have been isolated from both natural and potable freshwater sources, soil, contaminated solutions, and reptiles. The organism can grow in distilled and unsupplemented water. Likely sources of healthcare- related exposure may occur from bronchoscopes that were processed with tap water or from exposure to a hydrotherapy pool. Reference: CDC Guidelines for Environmental Infection Control in Health-Care Facilities, 2003 CBIC Core Competency: Environment of Care

Which of the following are used for sterilization of medical instruments? 1) Gravity-displacement steam sterilizer 2) Pasteurizer 3) Ethylene oxide sterilizer 4) Ultrasonic cleaner A. 1, 2 B. 1, 3 C. 1, 2, 3 D. 1, 2, 3, 4

B. 1, 3 Rationale: Sterilization of instruments results in the instruments being free of all microbes, including spores. This can be achieved through the use of various types of steam sterilizers; gas sterilizers including those that use ethylene oxide gas, ozone, and hydrogen peroxide gas or vapor; and liquid immersion in chemical sterilants. Reference: APIC Text, 4th edition, Chapter 106 - Sterile Processing

Which of the following need to be considered when updating the annual infection risk assessment? 1) An evaluation of the previous year's goals and objectives 2) An identification of risks based on geographic location, community, and population served 3) Risks related to the type of services that the facility provides 4) A broad assessment of all risks identified in the facility A. 1,2,4 B. 1,2,3 C. 2,3,4 D. 1,3,4

B. 1,2,3 1) An evaluation of the previous year's goals and objectives 2) An identification of risks based on geographic location, community, and population served 3) Risks related to the type of services that the facility provides Rationale: The risk assessment should be tailored to the risks in specific to a facility and must include identification of risks based on the facility's geographical location, community and population served as well as the types of services the facility offers. The annual risk assessment should take into account the previous year's goals and objectives and whether the facility met the targets associated with those goals.

Which of the following specimens can be refrigerated? 1) Urine 2) CSF 3) Stool 4) Sputum 5) Vaginal secretions 6) Ocular secretions A. 1,3,5 B. 1,3,4 C. 1,2,3 D. 2,5,6

B. 1,3,4

A caregiver has an unprotected exposure to TB. The TST should be administered at the time of the exposure and repeated after ________ to look for possible converters. A. 3 Weeks B. 12 Weeks C. 3 Months D. 6 Months

B. 12 Weeks

What is the temperature required for flash steam sterilization? A. 100C/212F B. 132C/270F C. 150C/302F D. 172C/342F

B. 132C/270F

Microorganisms are grown on culture media made of an agar base. Additive to media vary according to growth requirements of organism and/or the desire to select out a specific organism. Fastidious organisms require _____________ media, and _____________ media is used to inhibit normal commensals. 1. Differential 2. Enriched 3. Selective 4. Nutrient broth 5. Synthetic sheep blood agar A. 1,3 B. 2,3 C. 5,1 D. 3,4

B. 2,3 Fastidious organisms require Enriched media, and Selective media is used to inhibit normal commensals.

I design meeting is reviewing the new layout of the preparation and packing area in a sterile processing department. What important information should the IP bring to the meeting? 1. Minimum of 8 air exchange as per hour 2. Surgical pack assembly room should maintain positive airflow 3. Humidity issue between 30 and 60% 4. Return air registers should be at or near ceiling level. A. 1,2 B. 2,3 C. 3,4 D. 1,4

B. 2,3 Sterilization , preparation, packaging and sterile storage are considered clean areas and should have positive airflow ventilation. Soiled and decontamination areas should be under negative pressure. Each area should have a minimum of 10 air exchanges per hour. Humidity should be between 30 and 60%. Return air registers shall be at rear floor level, thereby facilitating the installation and effective maintenance of any filtering systems.

Which of the following statements are true of tests with a higher sensitivity than specificity? 1. The test will be more accurate at predicting who is ill 2. Patients require further testing 3. The test has a high Type II error rate 4. There will be a higher rate of false-positives A. 2,3 B. 2,4 C. 1,3 D. 1,2

B. 2,4 Patients require further testing There will be a higher rate of false-positives

On September 1, there were 30 surgical patients in the hospital. Two of these were postop patients with SSIs. A total of 75 surgeries were performed in September. Six additional SSIs occurred in patients who had surgery in September. What was the numerator for an incidence rate in September? A. 30 B. 6 C. 8 D. 75

B. 6 Rationale: An incidence rate is a measure of the frequency with which an event occurs in a population over a specified period of time. Incidence indicates the risk of disease in a population over a period of time. The numerator is the number of new cases of a disease during a specific time period. The denominator is the population at risk for the same time period. The incidence rate is equal to the numerator divided by the denominator and multiplied by a constant. Because there were six new cases after September 1, the numerator to calculate the incidence rate in this scenario is 6. Reference: APIC Text, 4th edition, Chapter 13 - Use of Statistics in Infection Prevention CBIC Core Competency: Surveillance and Epidemiologic Investigation

The incubation period for pertussis is A. 3-5 days B. 7-10 days C. 14 days D. 14-21 days

B. 7-10 days The incubation period for pertussis is 7-10 days with a range of 5-21 days. Classic pertussis is characterized by the 3 phases: 1. Catarrhal Phase: 7-10 incubation with a low grade fever, cough, and coryza 2. Paroxysmal Phase: Intense coughing spasms (whooping) 3. Convalescent Phase: Cough that can last for 2-8 additional weeks Pertussis is transmitted from person to person via large respiratory droplets generated by coughing and sneezing. Infants aged younger than 12 months and especially younger than 3 months are most likely to have severe pertussis, require hospitalization, and have respiratory and other complications of pertussis. Most deaths due to pertussis occur in infants less than 2 months of age. Tetanus toxoid, reduced diphtheria toxoid, and acellular pertussis (Tdap) vaccine is recommended before pregnancy, during pregnancy, or immediately postpartum.

Which of the following is an example of an effective performance measure? A. A measure that has been developed based on observation of practices in a facility. B. A measure that is based on a definition that can easily be understood and applied in a facility C. A measure that demonstrates a return on investment (ROI) D. A measure that takes into consideration customer satisfaction

B. A measure that is based on a definition that can easily be understood and applied in a facility Rationale: Performance measures should be evidence-based, well-defined, clinically important for patient populations, and broadly applicable in different types of facilities. Selection of performance measures will be based on both external and internal measurement requirements. Reference: APIC text, 4th edition, Chapter 17 - Performance

The director has requested that the IP summarize the results of an education program presented to five different groups within the institution. The director specifically requests that the method used not only indicate the overall mean score for each group, but also, aid a simple comparative analysis for all who participated. The best data display technique to summarize these fi would be: A. A line list B. A pie chart C. A bar chart D. A spreadsheet

B. A pie chart Rationale: A pie chart is a circular chart divided into sectors, illustrating proportion. In a pie chart, the arc length of each sector (and consequently its central angle and area), is proportional to the quantity it represents. A pie chart is best suited for visually representing the overall mean score and allowing viewers to rapidly compare one group to another. Reference: APIC Text, 4th edition, Chapter 13 - Use of Statistics in Infection Prevention

You have recently been hired as an IP at a 250-bed hospital. One of your first duties will be to develop an orientation program for all new employees. This program must be applicable to individuals assigned to nursing and ancillary departments. You realize that these employees will have different educational backgrounds and life experiences. Which of the following is a correctly written instructional objective? A. To provide information so the employee will feel more secure in the hospital environment. B. After completion of the program the employee will be able to list, with 90% accuracy, 5 infection prevention and control resources in the hospital. C. After completion of the program, the employee will understand why infection prevention and control is important. D. To provide information that will enable the employee to use infection prevention and control manual.

B. After completion of the program the employee will be able to list, with 90% accuracy, 5 infection prevention and control resources in the hospital. Instructional objectives will describe what the MEASURABLE outcome once the learner has completed the course.

Third-generation cephalosporins have supplanted clindamycin as the highest risk antimicrobials associated with CDI. What class of antibiotics do cephalosporins belong to? A. Fluoroquinolones B. Beta-lactam drugs C. Macrolides A. Aminoglycosides

B. Beta-lactam drugs Rationale: Fluoroquinolones = emerging trend for CDI

An employee has sustained a needlestick injury from a blood-contaminated needle. The source patient was Hepatitis B virus (HBV) positive, and the employee had completed one of the three vaccinations in the Hepatitis B series. Which of the following is the correct postexposure prophylaxis (PEP) for this caregiver? A. Complete Hepatitis B vaccine series B. Complete the Hepatitis B vaccine series and provide Hepatitis B immunoglobulin C. Provide Hepatitis B immunoglobulin and begin interferon therapy D. No PEP is needed

B. Complete the Hepatitis B vaccine series and provide Hepatitis B immunoglobulin Rationale: HBV is transmitted by percutaneous or mucosal exposure to infectious blood or body fluids. The risk of HBV seroconversion after a percutaneous injury ranges from 23-62% depending on the Hepatitis B e antigen (HBeAg) status of the source person. For exposed persons who are in the process of being vaccinated but have not completed the vaccination series, vaccination should be completed as scheduled, and Hepatitis B immune globulin (HBIG) should be added as indicated. Both HBIG and the Hepatitis B vaccine should be administered as soon as possible after exposure (preferably within 24 hours).

Strategic Planning includes all of the following EXCEPT: A. Action Planning B. Defining the common purpose for all departmental activities C. An analysis of the organization D. Forming conclusions about what an organization must do

B. Defining the common purpose for all departmental activities Rationale: Strategic planning is an organization's process of defining its strategy, or direction, and making decisions on allocating its resources to pursue this strategy. Steps integral to the process include: 1. Determine where you are 2. Identify what is important 3. Define what you must achieve 4. Determine who is accountable 5. Review

IP Managers are individuals who plan, organize, direct, control and coordinate activities in order to move the organization toward: A. Economic Stability B. Desired objectives C. Higher Profits D. Greater Social influence

B. Desired objectives Rationale: The overall role of a manager is to guide organizations towards accomplishing goals. The 6 basic functions of managers include: 1. Planning 2. Organizing 3. Staffing 4. Leading 5. Controlling 6. Motivating

The IP is preparing Data for entry into NHSN. Which of the following is correctly classified as clean-contaminated (class II)? A. Emergency splenectomy in a 10-year-old female with ideal pathic thrombocytopenic purpura (ITP) B. Elective thoracotomy with upper lobe ectomy in a 48-year-old smoker. C. Perforated diverticulum and a 50-year-old female. D. Closed reduction of the Smiths fracture in an 82-year-old male

B. Elective thoracotomy with upper lobe ectomy in a 48-year-old smoker. Clean contaminated or class II surgical wounds may involve entry into part of the body that normally contain flora, such as respiratory or urinary tract; however, in order to qualify as a class II, such procedures must be elective and do not violate aseptic technique nor show evidence of an infectious process. The elective thoracotomy with upper lobe resection involves the respiratory tract, a potential source of contamination. However, surgery was elective and did not show infection or break in technique, so is correctly classified as a clean contaminated

During educational in-service to the pediatric nurses, the IP asks how to manage a patient with cytomegalovirus (CMV). The correct precautions include: A. Assigning non-pregnant RN to care for CMV-infected children B. Following standard precautions C. Cohorting all children with CMV D. Using Droplet precautions

B. Following standard precautions CMV infects 50-80% of persons. Routine screening of patients for CMV is not recommended. Groups at high risk are infants infected in utero or during delivery, and immunocompromised persons. Standard precautions in healthcare settings are adequate for preventing transmission of CMV between patients and staff.

Which of the following would be a good tool to prepare for a Joint Commission visit? A. Root cause analysis B. Gap analysis C. Multivoting D. Plan, Do, Study, Act

B. Gap analysis Rationale: Business and quality professionals describe a gap analysis as a technique to determine the steps to take to move from a current state to a desired future state. A gap analysis begins with: 1) Listing characteristic factors, such as attributes, competencies or performance levels of the present situation - "What is" 2) Listing factors required to achieve the future objectives - "What should be" 3) Identifying the highlights or "gaps" that exist within the process and that must be filled to meet a goal or achieve standard compliance. Literature also refers to a gap analysis as a need-gap analysis, needs analysis, and needs assessment.

Which vaccine is contraindicated for pregnant healthcare workers? A. Hepatitis A (HAV) B. Human Papillomavirus (HPV) C. Hepatitis B (HBV) D. Herpes Simplex Virus (HSV)

B. Human Papillomavirus (HPV) If a woman is found to be pregnant after initiating the vaccination series, the remainder of the three-dose series should be delayed until completion of pregnancy.

You are reviewing the serology marker results of a healthcare worker. The Hepatitis B serology marker results are as follows: HBsAg = Negative (-) anti-HBc = Positive (+) anti-HBs= Positive (+) What is the Hepatitis B status of this healthcare worker? A. Susceptible B. Immune due to natural infection C. Immune due to HBV vaccination D. Acutely infected

B. Immune due to natural infection Rationale: Hepatitis B Surface Antigen (HBsAg) = The presence of HBsAg indicates that the person is infectious (regardless of acute or chronic). It is also used to make the vaccine. Total Hepatitis B Core Antibody (anti-HBc) = Appears at the onset of symptoms in acute HBV and persists for life. Its presence indicates previous or ongoing infection. Hepatitis B Surface Antigen Antibody (anti-HBs)= The presence of anti-HBs is generally interpreted as indicating recovery and immunity form Hepatitis B. Reference: CDC Interpretation of Hepatitis B Serologic Test Results table www.cdc.gov/hepatitis

A patient has been taught to use a clean technique for intermittent clean catheterizations. The best method to assess competency in the procedure is to: A. Ask the patient to describe the technique B. Observe a return demonstration C. Quiz patient on the technique D. Monitor the patient for urinary infections

B. Observe a return demonstration

Human factors engineering is the discipline of applying what is known about human capabilities and limitations to the design of A. Organizing, planning, leading, controlling B. Products, processes, systems, work environment C. Directing, goalsetting, decision-making, resource allocation D. Hardware, software and operating system interfaces

B. Products, processes, systems, work environments Human factors is the scientific discipline concerned with the understanding of interactions among humans and other elements of a system, and the profession that applies theory, principles, data and methods to design in order To optimize human well-being and overall system performance.

The IP has implemented a new educational program focusing on teaching safe injection practices to the staff nurses. Several months after the program is initiated, the IP is notified of a potential outbreak issue - a nurse used a medication vial for more than one patient. Which of the following quality tools would be most useful to investigate this incident? A. Value stream mapping (VSM) B. RCA C. SWOT D. FMEA

B. RCA

A physician orders a culture, ova and parasite specimen on a 10 y/o boy admitted with diarrhea. A liquid stool specimen is collected from the patient at 9pm. The specimen is refrigerated until 9am the next day when the physician calls and requests the lab to look for amoebic trophozoites. The best course of action is to: A. Perform a trichome stain on the original specimen B. Request a fresh specimen C. Perform a concentration on the original stool specimen D. Perform a saline wet mount on the original specimen

B. Request a fresh specimen

Which of the following processes should take place after an adverse event has occurred? A. Failure mode effect analysis (FMEA) B. Root cause analysis (RCA) C. Strength, Weaknesses, Opportunities, Threats analysis (SWOT) D. Infection Control Risk Assessment (ICRA)

B. Root cause analysis (RCA) Rationale: The RCA process takes a retrospective look at adverse outcomes and determines what happened, why it happened, and what an organization can do to prevent the situation from recurring. A thorough RCA determines: 1) Human and other factors 2) The process or system involved 3) Underlying cause and effects of the process 4) The risk and potential contributions to failure or adverse results Reference: APIC text, 4th edition, Chapter 16-Quality Concepts

Hand hygiene, clean exam gloves, and personal protective equipment (as appropriate) are sufficient for aseptic technique for which of the following procedures? A. Central Line dressing change B. Routine wound cleaning C. Tracheostomy care D. Lumbar puncture

B. Routine wound cleaning Routine wound cleaning can be carried out under aseptic conditions using good hand hygiene, clean exam gloves, and any necessary PPE such as a gown. Reference: APIC Text Online, Chapter 30 - Aseptic Technique

The infection prevention team has adopted the reduction of healthcare-associated MRSA as an annual goal. It has implemented improved processes and plans to monitor the effectiveness of these processes. Which of the following performance improvement tools will assist in identifying effectiveness? A. Affinity diagram B. Run chart C. Process flow chart D. Pareto chart

B. Run chart Rationale: Run charts are useful for identifying variations and trends, especially when assumptions for construction of control charts are not met. Run charts display observed data that can reveal trends or patterns over a specified period of time. They can be used with any type of data (discrete, continuous, etc.) and do not use any statistical calculations aside from measures of central tendency. They require at least 20 data points for reliability.

Micro organisms are grown on culture media made of an agar base. Additives to media vary according to growth requirements of organisms and or the desire to select out a specific organism. Fastidious organisms required enrichment media. Which media is used to inhibit normal commensals? A. Synthetic sheep blood agar B. Selective C. Enrichment D. Differential

B. Selective Select media are used for the growth of only selected micro organisms. For example, if a micro organism is resistant to a certain type of antibiotic, such as ampicillin or tetracycline, then the anabiotic can be added to the medium in order to prevent other cells, which do not possess the resistance, from growing. Media lacking in amino acids such as protein in conjunction with E. coli unable to synthesize it were commonly used by geneticist the for the emergence of genomics to map bacterial chromosomes. Selective growth media are also used in cell culture to ensure the survival or proliferation of cells with certain properties, such as antibiotic resistance or the ability to synthesize a certain metabolite. Normally, the presence of a specific gene or an allele of a gene conferred upon the cell the ability to grow in a selective medium. In such cases, the gene is termed a marker. Selective growth media for eukaryotic cells commonly contain neomycin to select cells that have been successfully transfected with a plasmid carrying the neomycin resistant gene as a marker. Gancyclovir is an exception to the rule as it is used to specifically kill cells that carry its respective marker, the herpes simplex virus thymidine kinase.

The optimal positioning of a patient to prevent ventilator associated pneumonia is A. Supine B. Semirecumbent C. Probe D. Sim's position

B. Semirecumbent Five key components for the VAP bundle: 1. Elevate the head of the bed to 45° when possible; otherwise, attempt to maintain the head of the bed at more than 30° 2. Evaluate readiness for extubation daily 3. Use endotracheal tube's with subglottic secretion drainage 4. Conduct oral care and decontamination with chlorhexidine 5. Initiate safe enternal nutrition within 24 to 48 hours of ICU admission

Who is at a higher risk of contracting Cytomegalovirus (CMV)? A. Unvaccinated Healthcare worker for CMV. B. Seronegative Healthcare worker with children at daycare. C. Healthcare worker who self identified to Employee Health as immunocompromised. D. Healthcare worker with a patient who tested positive for CMV one day prior.

B. Seronegative Healthcare worker with children at daycare Rationale: Major risks for primary infections are among seronegative women include the following: 1. Age <25 years 2. Multiple sexual partners 3. Exposure to young children (especially those who attend day care) at home and in the work place; for example, day care centers and schools HCW do not have an increased risk of acquisition of CMV infection The spread of CMV among the general population is facilitated by asymptomatic primary and recurrent infections, multiple sites of excretion, prolonged and intermittent execration, and excretion of virus despite the presence of specific immunity. Asymptomatic transmission further supports the need for adherence to Standard Precautions among all HCW when in contact with all patients.

Do you have identified cases of Stenotrophomonas maltophilia In the respiratory secretions of patients in the ICU. After conducting a preliminary review, which of the following sources as most likely? A. Showerheads B. Shallow, automatic sink with aerator C. New waterless baiting system e.g. warm pipes D. Contaminated respiratory equipment

B. Shallow, automatic sink with aerator Various study section on the aerators may be contaminated by Graham negative bacteria including P. aeruginsoa, S. maltophilia, B. cepacia, and A. calcoaceticus. Aerators contaminated with bacteria have been epidemiological linked to colonized or infected patients.

HAV infection during pregnancy can cause an increased risk of severe systemic infections. It can also cause: A. Severe birth defects in the fetus B. Spontaneous abortion and preterm delivery C. Material HAV transmission to the fetus D. Severe complications during delivery and is considered a high risk pregnancy

B. Spontaneous abortion and preterm delivery Peak infectivity occurs during the 2 weeks before the onset of jaundice or elevation of hepatic transaminases, when the viral concentration in the stool is the greatest. Hepatitis A is an inactivated vaccine and is therefore RECOMMENDED for pregnant healthcare workers.

Which of the following precautions should be used for a patient who is immunocompromised and suspected of having cryptococcal meningitis? A. Contact precautions for staff, family restricted from visiting other patients B. Standard Precautions for family and staff C. Mask worn when within 3 feet from bed D. Airborne Precautions for 24 hours after abx is started if the patient is improving

B. Standard Precautions for family and staff Cryptococcus is a fungus - does not spread from person-to-person.

A Gram stains shows clusters of Gram-positive cocci. The most likely pathogen is A. Streptococcus spp. B. Staphylococcus spp. C. Neisseria spp. D. Haemophilus spp.

B. Staphylococcus spp. Rationale: Streptococcus spp. = Pairs and chains of gram-positive cocci Staphylococcus spp. = Clusters of Gram-positive cocci Neisseria spp. = Gram-Negative diplococci Haemophilus spp. = Gram-Negative cocobacilli

Which of the following would be an appropriate method to evaluate the quality of an infection prevention program? A. The total number of areas where surveillance was carried out in the past year B. THe average amount of time that elapsed between receiving reports from the lab about patients with MDR infections and placing those patients in the appropriate isolation precautions C. The number of IPs in the program per the number of beds D. THe average amount of money spent on isolation gowns this year as compared to last year

B. THe average amount of time that elapsed between receiving reports from the lab about patients with MDR infections and placing those patients in the appropriate isolation precautions Rationale: THe quality of the IP program should be assessed routinely by evaluating customer satisfaction, appropriateness, efficacy, timeliness, availability, effectiveness, and efficiency.

Which of the following is the MOST important reason for having an Infection Prevention and Control Committee? A. The IPC is necessary to justify the IP's position B. The IPC is a vehicle for communication and consensus building C. The IPC function is required by The Joint Commission D. The IPC can replace the organization's safety committee

B. The IPC is a vehicle for communication and consensus building Rationale: The IPC function as

The endoscopy manager has asked for your guidance in finding a location to store sterile supplies. Your response is: A. The room must be positive pressure with a temperature of no greater than 65°F, with a relative humidity of less than 70 percent B. The room must be positive pressure with a temperature of approximately no greater than 75°F, with a relative humidity not to exceed 70 percent C. The room must be negative pressure with a temperature of 65°F, and a relative humidity of greater than 70 percent D. There are no specific requirements for storing sterile supplies

B. The room must be positive pressure with a temperature of approximately no greater than 75°F, with a relative humidity not to exceed 70 percent Rationale: The sterile storage area should be a limited access area with a controlled temperature (may be as high as 75°F) and relative humidity (30 to 60 percent in all work areas except sterile storage, where the relative humidity should not exceed 70 percent). Reference: APIC Text, 4th edition, Chapter 106 - Sterile Processing

The biological indicator that was included in a steam sterilization load of non-implantable instruments has shown a positive result, as have biological indicators used in two additional follow-up tests of the sterilizer. The chemical indictor in the original load was reactive, as were the chemical indicators used in the follow-up tests. The log of all the runs indicates that the run conditions (temperature and time) were appropriate for the instrument load and there were no abnormalities in steam supply or electrical supply. Which of the following should be done next: A. The load should be released for use in the facility B. The sterilizer should be tested again with paired biological indicators from two different manufacturers C. The items from the load should be immediately recalled and reprocessed D. All items that were processed in that sterilizer since the time of the last negative biological indicator should be recalled and reprocessed.

B. The sterilizer should be tested again with paired biological indicators from two different manufacturers Rationale: Biological indictors included in steam sterilization loads may show a positive result if the sterilization parameters are not met; if there is contamination of the growth medium in the indicator; or if the indicator was not manufactured properly. With steam sterilization, the instrument load does not need to be recalled for a single positive biological indicator test, with the exception of implantable objects. The log should be checked to ensure that the sterilizer was used correctly and maintenance should be contacted to determine if there was an interruption to steam or electrical supply. If there is no indication of abnormalities, then the sterilizer should be tested again in three consecutive cycles using paired biological indicators from different manufacturers to account for a possible defective biological indicator. Reference: CDC Guideline for Disinfection and Sterilization in Healthcare Facilities, 2008 Page: 76-78, 117 CBIC Core Competency: Cleaning, Sterilization, Disinfection, Asepsis

The type of leader who sets goals, articulates clear expectations of results and behaviors, and provides constructive feedback to keep the team on task is referred to as a: A. Charismatic leader B. Transactional leader C. Transformational leader D. Process-orientated leader

B. Transactional leader Rationale: Also known as managerial leadership, this focuses on the role of supervision, organization, and group performance. This style of leadership promotes compliance of subordinates through both rewards and punishments. References: APIC Text Online, Chapter 3 - Education and Training

The CDC has categorized bioterrorism agents according to priority. A disease that meets the criteria listed under Category B is: A. Anthrax B. Typhus C. Hantavirus D. Measles

B. Typhus Rationale: Agents classified by CDC as Category A are biological agents that have high potential for adverse public health impact, increased risk to national security, serious potential for large-scale dissemination, potential for public panic, social disruption, and high mortality rates. Category B agents have a moderate ease in disseminating, specific requirements for disease surveillance, moderate morbidity rates, and low mortality. Typhus (Rickettsia prowazekii) meets the Category B criteria. Reference: Bioterrorism Agents. In: Brooks K. Ready Reference for Microbes, 3rd edition. Washington, DC: Association for Professionals in Infection Control and Epidemiology, 2012. CBIC Core Competency: Preventing/Controlling the Transmission of Infectious Agents

Which virus is associated with Chickenpox, Shingles, and Herpes? A. Varicella B. Varicella-Zoster C. Coxsackie D. Herpes zoster

B. Varicella-Zoster Varicella (chickenpox) and herpes zoster are different manifestations of the same virus. - The primary infection is chickenpox when a varicella-zoster virus (VZV) first infects humans. - VZV remains latent for a variable period of time and, when it is reactivated, it presents as shingles or herpes zoster infection. The effects of the varicella virus on the fetus are unknown, pregnant women should not be vaccinated during pregnancy. The infection is highly contagious (90% transmission rate to susceptible household contacts) from 1 day before the outbreak of the rash and lasts until the lesions have dried completely.

What 5 bacteria are known to transform into endospores?

Bacillus antracis - anthrax Clostridium tetani - tetanus Clostridium defficile - c diff Clostridium perfringens - gas gangrene Clostridium botulinum - botulism

Patients with cell-mediated immunity dysfunction are susceptible to infections attributed to pathogenic intracellular bacteria. Examples of these organisms include: 1) Salmonella typhi 2) Bacteroides fragilis 3) Listeria monocytogenes 4) Staphylococcus aureus A. 3,4 B. 1,2 C 1,3 D. 2,3

C 1,3 Rationale: Facultative intracellular bacteria invade cells when it gives them selective advantage in the host. Bacteria that can enter and survive within eukaryotic cells are shielded from humoral antibodies and can be eliminated only by cellular immune response. However, these bacteria must possess specialized mechanisms to protect them from the harsh environment of the lysomal enzymes encountered within the cells. Examples include: Legionella pneumophila R. rickettsia Salmonella and Mycobacterium spp Invasive Escherichia coli Listeria monocytogenes Neisseria spp Shigella spp

The annual education budget for the Infection Prevention Department is $1,650.00. In October, the Infection Prevention Manager allocated 20% of the department education budget towards resources for the annual flu shot program. However, in November, the financial report indicates only 15% was spent. How much of the budget amount remains unspent? A. $330.00 B. $247.50 C. $82.50 D. $66.00

C. $82.50 Rationale: 20%-15%=5% 1650 x 0.05 = 82.5

Which of the following rules should be followed when collecting a stool sample for C. difficile testing? 1) Stool sample should be freshly passed within 1-2 hours 2) 10-20mL. of formed stool should be collected 3) Stool should be passed into a clean, dry container 4) Specimens should be obtained before antimicrobial agents have been administered A. 1, 2 B. 2, 3 C. 1, 3 D. 1, 4

C. 1, 3 Rationale: The accuracy of all tests depends on proper specimen handling and transport. The following rules should be followed when collecting samples for C. difficile testing: • Stool samples should be freshly passed within 1-2 hours • 10-20 mL of watery, soft, or unformed stool should be collected • Stool should be passed into a clean, dry container Reference: APIC Text, 4th edition, Chapter 72 - Clostridium difficile Infections and Colitis CBIC Core Competency: Identification of Infectious Disease Processes

An entry-level (novice) IP in your department has set a goal of advancing to achieve middle level (proficient) competency within the next year. Which of the following activities would be the most appropriate to include on her personal development plan for the year: 1) Nominating herself for the President-elect position of the local APIC chapter 2) Taking the certification infection control exam within 6 months 3) Requesting information about a Masters of Science in Epidemiology degree 4) Learning the basics of CAUTI surveillance A. 1 B. 1,2 C. 1,2,3 D. 1,2,3,4

C. 1,2,3 1) Nominating herself for the President-elect position of the local APIC chapter 2) Taking the certification infection control exam within 6 months 3) Requesting information about a Masters of Science in Epidemiology degree Rationale: APIC has created a competency model to help guide the advancement of infection preventionist in the field. The 3 levels of competency are: 1. Early-Level = Novice 2. Middle-level = Proficient 3. Advanced level = Expert The competency levels can be used to guide goal setting activities as part of the IPs personal development plan. Middle-level competencies include being a certified IC, considering and advanced degree in the field, and being active in the local APIC chapter by serving in a leadership position.

Which of the following are considered to reduce the chance of transmission of Neisseria meningitidis to laboratory personnel. 1. Surgical mask 2. Gown and Gloves 3. Biosafety cabinet (BS-2) 4. N95 mask and gloves 5. Immunization A. 1,2,3,4,5 B. 2,3,4,5 C. 1,2,3,5 D. 2,3,4

C. 1,2,3,5 Rationale: Transmission to laboratory workers is also a concern and reinforces the importance of masks, gowns, and gloves with manipulating secretions or potential aerosols, biosafety cabinet (BS-2) precautions for working with cultures of N. meningitidis, and immunization of microbiology laboratory personnel. In 2002, the CDC reported two cases of fatal laboratory-associated meningococcal disease that occurred in microbiologists who handled clinical specimens of N. meningitis serogroup C. This meningococcal disease occurring in the preceding 15 years through a Listserv survey. The overall fatality rate in this series was 50%.

Patients with cell-mediated immunity dysfunction are susceptible to infections attributed to pathogenic intracellular bacteria. Examples of these organisms include: 1. Salmonella typhi 2. Bacteroides fragilis 3. Listeria monocytogenes 4. Staphylococcus aureus A. 3,4 B. 1,2 C. 1,3 D. 2,3

C. 1,3

The IP is performing the annual evaluation of the infection prevention and control program. Components of tis document should include: 1) The achievements and activities of the program 2) Results from the latest accreditation survey 3) Stress the value of the program to the organization 4) Satisfy the legal requirements for reporting infections A. 1,2 B. 2,3 C. 1,3 D. 3,4

C. 1,3 Rationale: An annual evaluation of the infection prevention program is important to outline the achievements and activities of the program and describe support requirements. The value of the infection prevention program to the organization should be emphasized along with patient outcomes and cost savings.

The risk of Hepatitis B virus seroconversion after percutaneous injury ranges from _____ to _____ % depending on the Hepatitis B e antigen status of the source person. A. 6 to 15 B. 10 to 21 C. 23 to 62 D 25 to 65

C. 23 to 62 Rationale: The risk of Hepatitis B virus seroconversion after a percutaneous injury ranges from 23-62% depending on the Hepatitis B e antigen status of the source person. APIC text - Chapter 101- Occupational Exposure to Bloodborne Pathogens

If a patient is on droplet precautions and no private room or cohorting is available, what minimum spatial separation should be maintained between infected patient and non-infected patient? A. 6 feet B. 4 feet C. 3 feet D. 2 feet

C. 3 feet Rationale: Spatial separation of at least 3 feet and a Curtin between them is recommended.

Which of the following groups have a similar cell structure that is known as "prokaryotic" A. Virus B. Fungi C. Bacteria D. Parasites

C. Bacteria There are at least 17 groups of bacteria according to one classification scheme that is based in large part on nucleotide sequences of RNA. All of these groups have a similar cell structure that is known as "prokaryotic". Prokaryotic cell structure is microscopically characterized by lack of a visible nucleus, by the lack of a membranous nuclear envelope, and by lack of a membrane bound internal structure such as mitochondria.

Chickenpox pneumonia in pregnancy is more severe than in the nonpregnant adult, with a maternal mortality rate of ____________% compared with 11% in the nonpregnant adult. A. 21-37% B. 33-52% C. 41-46% D. 45-50%

C. 41-46% Chickenpox pneumonia is the most common complication and is expected to occur in 15-50% of adults when no antiviral treatment is given. Chickenpox pneumonia in pregnancy is more severe than in the nonpregnant adult, with a maternal mortality rate of 41-46% compared with 11% in the nonpregnant adult. Treatment with acyclovir is recommended for pregnant woman who develops chickenpox. Varicella-Zoster Immunoglobulin (VariZIG): is used in the US as PEP of varicella* and for whom varicella vaccine is contraindicated. - VariZIG is approved for administration as soon as possible following varicella-zoster virus exposure, ideally within 96 hours (4 days) for greatest effectiveness. - CDC recommends the use of VariZIG for pregnant women without evidence of immunity. Fetal death is the result of severe maternal varicella infection, usually accompanied by pneumonia. It has not been shown that the virus causes fetal death or first-trimester wastage.

Airborne precautions apply to airborne particles that are less than or equal to: A. 10 micrometers B. 8 micrometers C. 5 micrometers D. 3 micrometers

C. 5 micrometers Rationale: Airborne precautions apply to infections that are spread through airborne particles (evaporated droplets or dust) that are <5 micrometers in size as particles this small may stay suspended in the air for extended periods of time and can be inhaled or carried on the person from one patient to another. Patients on airborne precautions must be maintained in a negative pressure room. The patient must remain inside the room with the door closed.

What is the acceptable upper limit for relative humidity in a facility to prevent fungal growth? A. 40 percent B. 50 percent C. 60 percent D. 70 percent

C. 60 percent Rationale: Building materials contain fungal spores that have the potential to germinate in the correct conditions. Fungi require high relative humidity for growth, with most species needing relative humidity above 70 percent. Healthcare facilities should maintain relative humidity below 60 percent to prevent the growth of fungus and to provide a comfortable patient care environment. Reference: CDC Guidelines for Environmental Infection Control in Health-Care Facilities, 2003 CBIC Core Competency: Environment of Care

The hospital is at 100% occupancy in the IP is consulted regarding cohorting of patients. Which of the following pairs would be the safest to cohort? A. A new post operative knee replacement patient and a patient with an infected decubitus ulcer that was recently debriefed. B. A patient with AIDS and a patient receiving chemotherapy C. A newly admitted patient with influenza and a patient with pneumonia who is originally admitted with influenza four days ago. D. A premature neonate in a full-term healthy baby born the same day.

C. A newly admitted patient with influenza and a patient with pneumonia who is originally admitted with influenza four days ago. The first option is incorrect as a potential exists for the new knee to become infected from the decubitus ulcer. The second choice is incorrect as the patient with AIDS could have an opportunistic infection which could infect the immuno suppressed patient. The last choice is incorrect as a premature neonate are more vulnerable to organisms that the healthy baby might be colonized with.

Which of the following patient should be assigned to an airborne infection isolation room? A. A patient with a tracheostomy who has bronchitis. B. A patient with Guillain-Barré syndrome who is on a ventilator. C. A patient with an undetermined cause of pneumonia who has AIDS. D. A patient with community acquired pneumonia who is having copious respiratory secretions.

C. A patient with an undetermined cause of pneumonia who has AIDS. People with aids are at high risk for opportunistic infection such as TB. Worldwide, TB is the most common opportunistic infection affecting HIV-seropositive individuals, and it remains the most common cause of death in patients with AIDS.

A preliminary microbiology report states that a patient's blood culture grew aerobic, Gram-negative bacilli. Which of the following is the most likely genus and species of the organism: A. Enterococcus faecalis B. Bacteroides fragilis C. Acinetobacter baumanii D. Neisseria meningitides

C. Acinetobacter baumanii Rationale: Acinetobacter baumannii is a Gram-negative bacterium. It is typically short, almost round, and rod- shaped (coccobacillus). It can be an opportunistic pathogen in humans, affecting people with compromised immune systems and is becoming increasingly important as an HAI. It has also been isolated from soil and water samples in the environment. Reference: APIC Text, 4th edition, Chapter 77 - Environmental Gram-negative Bacilli CBIC Core Competency: Identification of Infectious Disease Processes

You are reviewing the serology marker results of a healthcare worker. The Hepatitis B serology marker results are as follows: HBsAg = Positive (+) anti-HBc = Positive (+) IgM anti-HBc = Positive (+) anti-HBs= Negative (-) What is the Hepatitis B status of this healthcare worker? A. Susceptible B. Immune due to natural infection C. Acutely infected D. Chronically infected

C. Acutely infected Rationale: Hepatitis B Surface Antigen (HBsAg) = The presence of HBsAg indicates that the person is infectious (regardless of acute or chronic). It is also used to make the vaccine. Total Hepatitis B Core Antibody (anti-HBc) = Appears at the onset of symptoms in acute HBV and persists for life. Its presence indicates previous or ongoing infection. IgM Antibody to Hepatitis B core Antigen (IgM anti-HBc) = Positivity indicates recent infection with Hepatitis B virus (<6 months). Its presence indicates an acute infection. Hepatitis B Surface Antigen Antibody (anti-HBs)= The presence of anti-HBs is generally interpreted as indicating recovery and immunity form Hepatitis B. Reference: CDC Interpretation of Hepatitis B Serologic Test Results table www.cdc.gov/hepatitis

A pregnant healthcare worker is concerned because she has been assigned to take care of a patient who has a cytomegalovirus (CMV) infection. How should an IP respond to this concern? A. Reassign her to another patient B. Place the patient on Contact Precautions while the healthcare worker cares for him C. Advise her that following Standard Precautions while caring for the patient will prevent transmission D. Advise her that she is likely already infected with CMV and should not worry about transmission

C. Advise her that following Standard Precautions while caring for the patient will prevent transmission Rationale: Standard precautions are sufficient. CMV is transmitted through saliva, urine, blood products, and organs. The CDC recommends using Standard precautions when caring for patients with CMV, with no additional precautions recommended for pregnant HCP.

Which of the following indicators is most accurate for assessing whether surgical instruments have been processed correctly in a steam sterilization cycle: A. Autoclave tape B. Chemical Indicator C. Biological Indicator D. The cycle indicator on the steam sterilizer

C. Biological Indicator Rationale: A biological indicator is the only method of assuring that a steam sterilizer has reached the correct conditions to ensure sterility of products. Use of a biological indicator provides reliable, qualitative evidence that all sporeforming organisms have been inactivated during the sterilization cycle. Reference: APIC Text, 4th edition, Chapter 106 - Sterile Processing

All of the following are examples of risk-adjusted stratification except: A. CLABSI rates by birth weight in the NICU B. Needlestick injuries by profession C. CATUI rate for the ICU D. CLABSI rates by the type of line

C. CATUI rate for the ICU Rationale: Stratification is a form of risk adjustment that involves classifying data into subgroups based on one or more characteristics, variables, or other categories. For example, a measure's population might be stratified by gender before calculating rates, resulting in a separate rates for females and males. In infection prevention, it is common to stratify infants by birth weight when assessing infection s and infection risk or to stratify sharps injuries by time of day, role, and unit. Each subgroup becomes a separate denominator (population of interest), with the numerator event of interest the same for the subgroups; separate rates are then calculated for each subgroup. Reference: APIC Text, 4th edition, Chapter 17 - Performance Measures

After reviewing the quarterly report, the manager of the adult ICU contacts the IP for assistance to create a plan to reduce central line infections. Which of the following should the IP recommend: A. Wait for next report to see if the rate has decreased B. Create an intravascular Team C. Develop a multidisipanary team to review and implement best practices D. Send a referral to Medical Affairs for peer review

C. Develop a multidisipanary team to review and implement best practices

Which of the following is not an infection prevention objective of an occupational health program? A. Contain costs by preventing infectious disease that result in absenteeism and disability. B. Provide care to personnel for work-related illness or exposures. C. Educate patients about the principles of infection prevention D. Collaborate with the IP department in monitoring and investigating potentially harmful infectious exposures and outbreaks.

C. Educate patients about the principles of infection prevention The five IP Objectives of an Occupational Health Program are: 1. Educate HCP about principles of infection prevention and their individual responsibility for infection prevention. 2. Collaborate with the IP department in monitoring and investigating potentially harmful infectious exposures and outbreaks. 3. Provide care to personnel for work-related illnesses or exposures. 4. Identify work-related infection risks and institute appropriate preventative measures. 5. Contain costs by preventing infectious diseases that result in absenteeism and disability

An example of using stratified random sample of surgery patients for measuring performance is selecting: A. The sample through random computer selection B. All surgical patients C. Every third surgical patient D. All patients of the four surgeons with the most patients

C. Every third surgical patient Rationale: An example of using stratified random sample of surgery patient for measuring performance is selecting every 3rd surgical patient because the sample is done of a predictable basis. Other means of stratification may be by age, gender, or other characteristics. While a small sample size may skew the data, a huge sample size may generate so much data that analysis is more difficult and time consuming, so with a large population, some type of sampling should be considered.

An infection Preventionist would like to compare best practices from the APIC guide to preventing central line associated bloodstream infections with current facility processes to determine what steps should be taking to help reduce CLABSI events. Which of the following tools would best help the IP to achieve this goal? A. Root cause analysis B. Strategic plan C. Gap analysis D. Failure mode effects analysis

C. Gap analysis A gap analysis can be defined as the determination of the difference between current knowledge/practices (What we are doing) And current evidence-based practices (what we should be doing). Gaps can occur in knowledge, skills or practice.

The bacterium most likely to be transmitted from mother to infant during labor and cause neonatal sepsis is: A. Escherichia coli B. Staphylococcus aureus C. Group B Streptococcus D. Group A Streptococcus

C. Group B Streptococcus Rationale: In Group B Streptococcus (GBS) neonatal infections, heavy maternal colonization is associated with an increased risk for preterm labor, which in turn is a significant risk factor for neonatal infection. "Group B is for Baby" Intrauterine infection of the fetus therefore likely occurs via ascending spread of GBS from the vagina of a pregnant, asymptomatically colonized woman and subsequent rupture of membranes before 37 weeks' gestation. Reference: APIC Text, 4th edition, Chapter 94 - Streptococci CBIC Core Competency: Identification of Infectious Disease Processes

You are reviewing the serology marker results of a healthcare worker. The Hepatitis B serology marker results are as follows: HBsAg = Negative (-) anti-HBc = Negative (-) anti-HBs= Positive (+) What is the Hepatitis B status of this healthcare worker? A. Susceptible B. Immune due to natural infection C. Immune due to HBV vaccination D. Acutely infected

C. Immune due to HBV vaccination Rationale: Hepatitis B Surface Antigen (HBsAg) = The presence of HBsAg indicates that the person is infectious (regardless of acute or chronic). It is also used to make the vaccine. Total Hepatitis B Core Antibody (anti-HBc) = Appears at the onset of symptoms in acute HBV and persists for life. Its presence indicates previous or ongoing infection. Hepatitis B Surface Antigen Antibody (anti-HBs)= The presence of anti-HBs is generally interpreted as indicating recovery and immunity form Hepatitis B. Reference: CDC Interpretation of Hepatitis B Serologic Test Results table www.cdc.gov/hepatitis

The healthcare facility has established a goal of improving infection prevention competency with hand hygiene among all staff. Which of the following education and trining approaches should the infection preventionist (IP) recommend as a priority? A. Analysis of human factors that my present unrecognized obstacles for compliance. B. Attending local/state health department educational programs on hand hygiene. C. Implementation of a mentoring program based on peer-to-peer instruction and coaching. D. Intensified disciplinary actions for employees who do not follow hand hygiene procedures.

C. Implementation of a mentoring program based on peer-to-peer instruction and coaching. HH is among the simplest and most effective preventative measures to reduce healthcare-associated infections. However, compliance with hand hygiene among healthcare workers is consistently suboptimal. The CDC's HH guideline describes the importance of role models. Numerous studies have suggested that healthcare personnel HH compliance is influenced significantly by the behavior of other healthcare personnel. The IP should emphasize the importance of role modeling to set high standards and to contribute to a culture of safety within a healthcare environment.

The Director of Infection Prevention and Control is leading a process improvement project to decrease the rates of CLABSIs in one of the hospital's ICUs. The multidisipanary team has discussed multiple process improvement strategies to decrease these blood stream infections. In developing the final improvement plan which of the choices below is most likely to help decrease the rates of these infections? A. Performing a gap analysis each moth. B. Performing a Failure Mode Effect Analysis immediately C. Incorporating the use of a CLABSI bundle and checklist to ensure that all aspects of the plan are followed D. Perform a SWOT analysis.

C. Incorporating the use of a CLABSI bundle and checklist to ensure that all aspects of the plan are followed.

The process of evaluating learner response to individual test questions in order to determine the quality and accuracy of those questions is known as: A. Validity testing B. Correlation C. Item analysis D. Risk adjustment

C. Item analysis Rationale: Item analysis is the process of examining student's test scores in order to assess the quality of the individual test questions as well as the test itself. Item analysis is useful for improving the quality of the test and is valuable for increasing the instructors' skill in test construction and identifying areas that need improvement or greater emphasis. Reference: APIC Text, 4th edition, Chapter 3 - Education and Training

A 38-year-old woman being treated for breast cancer has a WBC count of 2.3. This is an improvement over the counts seen during the past two weeks. Her physician has been aggressive in treating every potential infection she has had. She is currently on an antibiotic for a bloodstream infection from staphylococcus epidermidis. She now has a fever of 100.4°F. What action should be taken? A. Her intervenous side should be inspected frequently. B. None because she may have a neutropenic fever and no real infection. C. More blood cultures because she may be developing a fungal septicemia. D. Antibiotics stopped because she may have a "drug fever"

C. More blood cultures because she may be developing a fungal septicemia. Sepsis (staphylococcus and fungi) is the leading cause of mortality in neutropenic cancer patients. Early initiation of affective causative therapy as well as intensive adjunctive therapy is mandatory to improve outcome. Blood cultures as a part of the usual microbiological work up remain the gold standard for the diagnosis of bacteremia.

A nurse is concerned that she has been exposed to a patient with possible meningitis because she assisted with the patient's lumbar puncture in the ED and did not wear a mask. The patient's CSF Gram-Stain shows Gram-positive cocci in pairs and chains. Does the nurse need to receive post-exposure prophylaxis? A. Yes, she should receive PEP because the Gram-stain is indicative of Listeria monocytogenes. B. No, she does not need PEP because the Gram-stain is indicative of fungal meningitis. C. No, she does not need PEP because the Gram-stain is indicative of Streptococcus pneumoniae D. Yes, she should receive PEP because the Gram-stain is indicative of Neisseria meningitis.

C. No, she does not need PEP because the Gram-stain is indicative of Streptococcus pneumoniae Listeria monocytogenes = rod Neisseria meningitis = Gram Negative Fungus = no gram-stains

Managing infection prevention outcomes, analyzing variance trends, and evaluating corrective actions taken to reduce infection risks are components of: A. Case management B. Utilizing review C. Performance improvement D. Medical review

C. Performance improvement Rationale: Performance improvement is measuring the output of a particular process or procedure, than modifying the process or procedure to increase the output, increase efficiency, or increase the effectiveness of the process. Performance improvement is an ongoing cycle that focuses on patient clinical outcomes and customer satisfaction and service. Measuring performance determines program effectiveness and efficiency and whether proactive approaches or retrospective analysis of high-risk processes can further improve the infection prevention program. Reference: APIC Text, 4th edition, Chapter 16 - Quality Concepts

The IP receives a call from the FDA with an official request for private health information (PHI) about a patient who was admitted to the facility with botulism. How should the IP respond to this call? A. Tell the FDA officer that she cannot share PHI with the FDA due to HIPPA regulations B. Ask the FDA officer to contact the local health department to obtain information about this patient C. Provide the FDA office with the minimum amount of information necessary related to the patient D. Transfer the call to the Risk Management Department

C. Provide the FDA officer with the minimum amount of information necessary related to the patient

What instructor actions would be most useful when a change in on-the-job behavior is anticipated as an outcome of a training exercise? A. Alert the learner when the job performance does not meet expectations B. Conduct one-on-one interviews with the learners after the exercise C. Review program content and learner reaction with the learner's supervisor D. Use exit questionnaires to further gather information about the overall success of the training exercise

C. Review program content and learner reaction with the learner's supervisor The instructor should work with the appropriate supervisor to determine whether the learning objectives were met. It is the supervisor's responsibility to advise the learner if job expectations are not being met. Exit questionnaires and one-on-one interviews are good evaluation strategies, but working with the supervisor is essential when the focus of learning objectives is on-the-job behavior. Reference: APIC Text, 4th ed., Chapter 3 - Education and Training

What action is indicated when the IP is asked to help determine if a worker has experienced occupational acquisition of an infectious agent or disease in order to receive workers' compensation benefit? A. Provide enough information to prove or disprove the employee's claim B. Notify the facility's attorney immediately C. Review workers' compensation system in place D. Perform a root cause analysis (RCA) to investigate

C. Review workers' compensation system in place Rational: Disease that results from occupational exposure usually is eligible for compensation if the occupational exposure is the sole cause of disease; the occupational exposure is one of several causes of the disease; the occupational exposure hastens the onset of disability. The burden of proving that disease was occupationally acquired lies with the workers. The Management and Occupational Health provide the necessary follow-ups. IPs should be familiar with the workers' compensation system in place within their country.

An IP has completed the investigation of an increase in postdischarge SSIs following ambulatory surgical procedures. He identified some corrective measures that will require monitoring. Which of the following actions should the Infection Prevention team take? A. Monitor the cleaning of the surgical suites B. Ensure the effectiveness of the ventilation system C. Revise the facility's surveillance plan D. Ensure that patient care equipment is adequately cleaned

C. Revise the facility's surveillance plan Rationale: Surveillance activities should support a system that can identify risk factors for infection and other adverse advents, implement risk-reduction measures, and monitor the effectiveness of interventions. The items noted in answers A, B, and D are rarely related to the development of SSIs. The facility's surveillance program should be revised to reflect the current risks and risk reduction measures. Reference: APIC Text, 4th edition, Chapter 11 - Surveillance CBIC Core Competency: Surveillance and Epidemiologic Investigation

Fastidious organisms require Enrichment media. Which media is used to inhibit normal commensals? A. Differential B. Enriched C. Selective D. Synthetic sheep blood agar

C. Selective

A 19 y/o Female UCF student presents to the ED at ORMC with a high fever, and respiratory issues. The patient became hypoxic and required immediate intubation prior to being transferred to the ICU. Her CXR revealed air-space disease, most likely being pneumonia. A bronchoscopy is performed, and the Gram-stain reveals a Gram-negative diplococci. The attending calls you to assist both the employees who cared for this patient and her contacts. Those in need of prophylaxis following this exposure to this patient are: A. The ICU and ER staff, the college students in her dormitory and her family B. No special prophylaxis is needed C. The EMT who suctioned the patient, the physician who intubated the patient, and the patient's boyfriend D. The EMTs, the ER staff on duty, the ICU personnel and the radiography technician

C. The EMT who suctioned the patient, the physician who intubated the patient, and the patient's boyfriend

In a gram stain procedure, Gram-positive bacteria stain purple because: A. They have lipopolysaccharide layer in their cell wall that is decolorized with alcohol B. Their cell walls contain long-chain fatty acids that take up crystal violet easily C. They have a thick peptidoglycan cell wall that retains the primary stain during the alcohol decolorization D. Gram-staining is simple staining so the only stain used is crystal violet

C. They have a thick peptidoglycan cell wall that retains the primary stain during the alcohol decolorization lipopolysaccharide = gram (-) = Red Long-chain fatty acids = acid fast bacteria Simple staining = safranin as well

The occupational health RN has requested the IPs assistance in reporting the RN needlestick rate annually. Which formula should be used? A. Total number of needlesticks reported by RNing, divided by the average daily census B. Total number of needlesticks reported by RNing, divided by the needle devices used by RNing C. Total number of needlesticks reported by RNing, divided by the number of full-time RN employed during the year D. Total number of needlesticks reported by RNing, divided by the number of injections given by RNs

C. Total number of needlesticks reported by RNing, divided by the number of full-time RN employed during the year Rationale: The rates of needlestick injuries can be identified by occupational category. For example, total number of needlestick reported by RNing in 1 year, divided by the number of full-time equivalent RN employed in that year, equals the rate of needlesticks per full-time equivalent RN per year. = (# of Needlesticks by RNing) / (# FTE RNs employed that year)

You are the Infection Preventionist Director at a local hospital during an active pandemic. During the hospital Incident Command (IC) meeting you and your laboratory partners are directed to collaborate in establishing a testing strategy for patients being admitted to the hospital. Which is the best testing strategy for you to suggest? A. Utilize a test with a higher sensitivity and place patients with a positive result in isolation. B. Utilize a test with a higher specificity and place patients with a positive result in isolation. C. Utilize a test with a higher sensitivity, patients with a negative result do not need isolation, patients with a positive result should be retested with a test that has a higher specificity to confirm the positive result. D. Utilize a test with a higher specificity, patients with a negative result do not need isolation, patients with a positive result should be retested with a test that has a higher sensitivity to confirm the positive result.

C. Utilize a test with a higher sensitivity, patients with a negative result do not need isolation, patients with a positive result should be retested with a test that has a higher specificity to confirm the positive result. Rationale: Tests with a high sensitivity have a higher probability of a false positive and should be followed up with a test with a high specificity to confirm the positive result and rule out a potential false positive from the initial test.

What are the definitions of the CDC Category Rankings? Category IA. - Category IB. - Category IC. - Category II. - No recommendation. -

Category IA. Strongly recommended for implementation and strongly supported by well-designed experimental, clinical, or epidemiologic studies. Category IB. Strongly recommended for implementation and supported by some experimental, clinical, or epidemiologic studies, and by a strong theoretical rationale. Category IC. Required by state or federal regulations. Because of state differences, readers should not assume that the absence of an IC recommendation implies the absence of state regulations. Category II. Suggested for implementation and supported by suggestive clinical or epidemiologic studies or by a theoretical rationale. No recommendation. Unresolved issue. These include practices for which insufficient evidence or no consensus exists regarding efficacy.

causal association

Causal association is said to exist when the evidence indicates that a factor clearly increases the probability of the occurrence of a disease and reduction of this factor decreases the frequency of the disease.

Continuous data

Continuous data contains information that can be measured on a continuum scale and can have numeric value between the minimum and maximum value (a continuum). Examples: age, serum cholesterol level, temperature, infection rates

Which of the following specimens can remain at room temperature after collection if transport to the lab will be delayed? A. Sputum B. Urine C. Stool D. Cerebral spinal fluid (CSF)

D. Cerebral spinal fluid (CSF) - only for 1 hour

Please select which elements are included in an occupational health program: 1. Surveillance 2. Education 3. Immunizations 4. Isolation Precautions 5. Injury Prevention and Response A. 2,3,4,5 B. 1,2,3,4,5 C. 2,3,5 D. 1,2,3,5

D. 1,2,3,5 APIC chapter - Occupational Health

A patient is admitted to the ED with a diagnosis of severe sepsis. Which of the following elements of the sepsis bundle must be completed within three hours of presentation? 1. Measure lactate level 2. Apply vasopressors 3. Administer 30 ml/kg crystalloid for hypotension or lactate >=4mmol/L 4. Measure central venous oxygen saturation (ScvO2) A. 1,2 B. 2,3 C. 3,4 D. 1,3

D. 1,3 The severe sepsis three hour resuscitation bundle contains the following elements, to be completed within 3 hours of the time of presentation with severe sepsis: Measure lactate level Obtain blood cultures prior to administration of antibiotics Administer broad-spectrum anabiotic's Administer 30 ml/kg crystalloid for hypotension or lactate >=4mmol/L The 6- hour septic shock bundle contains the following elements, to be completed within 6 hours of the time of presentation with severe sepsis: Apply vasopressors (for hypotension that does not respond to initial fluid resuscitation to maintain a mean arterial pressure (MAP) >=65 mm Hg) In the event of persistent arterial hypertension despite volume resuscitation (septic shock) or initial lactate >=4 mmol/L (36 mg/dL): Measure central venous pressure Measure of central venous oxygen saturation Remeasure lactate if initial lactate was elevated

Standard Precautions apply to which of the following? 1. Blood 2. Sweat 3. Sputum 4. Non-intact skin A. 1,2 B. 1,3 C. 1,2,3 D. 1,3,4

D. 1,3,4 Rationale: Standard precautions should be used when caring for all patients as a minimum standard. Standard precautions apply to blood and all other bodily fluids (except sweat), mucous membranes, and non-intact skin.

Which of the following must be documented when using soaking solutions for cleaning instruments? 1) Monitoring the temperature of the solution 2) Monitoring the room temperature 3) Monitoring the humidity of the room 4) The time the instruments soaked A. 1,2 B. 2,4 C. 3,4 D. 1,4

D. 1,4 Monitoring the temperature of the solution and The time the instruments soaked

You are revising the disaster plan with the emergency management committee of your hospital. What should be the primary focus of this plan? 1. Preparing for a surge of causalities in case of a disaster. 2. How to identify risks for disaster. 3. How to prevent disasters from occurring. 4. How to reduce disaster impact on patients and staff. A. 1,2 B. 2,3 C. 3,4 D. 1,4

D. 1,4 The joint commission requires hospitals to plan how emergencies will be managed; must be documented in the emergency operations plan (EOP). The goals of a hospital preparedness/disaster plan are to: Enhance surge capacity response Ensure ability to maintain services in a sustained event. Collaboration and integration of plans with all emergency response partners Options two and three are outside the scope of a hospital EMC plan

An appropriate agent to select when performing high level disinfection would be A. Phenolic B. Quaternary ammonium compound C. Chlorine based product D. 2% glutaraldehyde

D. 2% glutarldehyde Glutarldehyde based agents are used to disinfect medical equipment that cannot be subject to steam sterilization, specifically heat sensitive, lens device is typically requiring high level disinfection between patient uses.

According to AAMI ST79, which of the following are among the recommendations for testing of mechanical cleaning equipment in order to verify adequate cleaning? 1) Verification should be carried out monthly 2) Verification should be carried out upon installation 3) Verification should be carried out after major repairs 4) Verification should be carried out when changing cleaning chemistry A. 1, 2, 3, 4 B. 1, 2, 4 C. 1, 2, 3 D. 2, 3, 4

D. 2, 3, 4 Rationale: Mechanical cleaning equipment uses automated cleaning and rinsing to remove soiling from items. According to AAMI ST79, verification of mechanical cleaners should be carried out weekly, and preferably daily. It should also be carried out upon installation of the equipment, after major repairs, and all cycles should be tested when changing cleaning chemistry. Reference: APIC Text, 4th edition, Chapter 106 - Sterile Processing

An IP wishes to monitor process outcomes in the ICU. Which of the following would be appropriate measures to incorporate in her performance measurement plan? 1) Rate of CAUTIs in a given month 2) Compliance with central line insertion bundle 3) Patient satisfaction 4) Administration of preoperative antibiotics within 1 hour prior to incision A. 1,2 B. 2,3 C. 3,4 D. 2,4

D. 2,4 Rationale: A process measure focuses on a process of the steps in a process that leads to a specific outcome. Process measures are commonly used to evaluate compliance with desired care or support practices or to monitor variation in these practices. Reference: APIC Text Online, Chapter 17 - Performance measures

Which of the following are acceptable methods for follow-up testing among healthcare personnel with unprotected exposure to TB? 1) QuantiFERON-TB Gold testing (QFT-G) of sputum at the time of exposure and 12 weeks after exposure. 2) QFT-G testing of blood at the time of exposure and 12 weeks after exposure. 3) TST via tine tests at the time of exposure and 12 weeks post exposure. 4) TST via the intradermal method at the time of exposure and 12 weeks after exposure. 5) Chest radiograph for personnel with prior positive TST or QFT-G results 6) Chest radiograph for symptomatic personnel with positive TST or QFT-G results A. 1,3,6 B. 2,3,5 C. 1,4,6 D. 2,4,6

D. 2,4,6 Rationale: QFT-G is a blood assay. Intradermal rather tine testing methods should be used for TST. Testing should be administered at the time of exposure and repeated at 12 weeks postexposure to look for possible converters. Chest radiographs are performed only on those with prior positive screening results who are currently symptomatic.

What is the appropriate temperature for vaccines that require refrigeration? A. 46F to 55F B. 25F to 35F C. 25F to 45F D. 35F to 46F

D. 35F to 46F Rationale: Most routinely recommended vaccines should be stored in refrigerator between 35F and 46F, with a desired average temperature of 40F. Exposure to temperatures outside this range may result in reduced vaccine potency and increased risk of vaccine-preventable diseases.

Negative pressure rooms should have how many air exchanges per hour? A. 20-34 B. 17-20 C. 12-16 D. 6-12

D. 6-12

Which of the following should be used as a quality improvement measure for infection prevention education programs? A. The frequency of classes offered through the year. B. The average number of attendees per class offered. C. Summary of pre-/post-test scores for each class. D. Analysis of program evaluation scores for all classes.

D. Analysis of program evaluation scores for all classes. Rationale: Quality improvement (QI) consists of systematic and continuous actions that lead to measurable improvement in healthcare services and the health status of targeted patient groups. Data is the cornerstone of QI. It is used to describe how well current systems are working and what happens when changes are applied and to document successful performance. Both quantitative and qualitative methods of data collection are helpful in QI efforts. Whatever evaluation methodology is used, the data must be gathered, tabulated, and analyzed to assess impact and make recommendations for curriculum revision before the next presentation. References: APIC Text, 4th edition, Chapter 3 - Education and Training; APIC Text, 4th edition, Chapter 16 - Quality Concepts

A critical review of the message section of a published study should include all the following except: A. Is the time period of the study appropriate? B. If controls are used, are they comparable to the cases? C. Is the study design described in enough detail and applicable to the purpose of the study? D. are the potential limitation and weaknesses describe?

D. Are the potential limitations and weaknesses described? Limitations and weaknesses of the study should be described in detail in the discussion section.

The first step in preparing of any educational program is: A. Investigating media/projection equipment available B. Identifying budgetary resources for educational program C. Incorporating the evaluation process in the preparations strategies D. Assessing the educational needs of the learning population

D. Assessing the educational needs of the learning population Learning needs assessment is a crucial first stage in the educational process. Learning is more likely to lead to change in practice when needs assessment has been conducted, the education is linked to practice, personal incentive drives the educational effort, and there is some reinforcement of the learning Reference: APIC Text, 4th edition, Chapter 3 Education and Learning

When contaminated human tissue is suspected as the cause of an outbreak, agencies that should be notified include: A. OSHA B. FDA only C. OSHA and FDA D. CDC and FDA

D. CDC and FDA Rationale: When contaminated human tissue or any defective product, medical device, or medication is suspected as the cause of an outbreak, agencies that should be notified include the FDA and CDC. Depending on the type of outbreak and state requirements, state and local public health departments may need to be notified as well. Notifications are important in outbreak investigations and should be done as soon as possible when a healthcare-associated infection or outbreak occurs.

Which surveillance study measures quality improvement related to outcome? A. Measuring the rates of compliance to a needleless IV system B. Observing handwashing in a critical care unit C. Monitoring compliance with isolation measures D. Calculating a ventilator-associated pneumonia rate for the ICU

D. Calculating a ventilator-associated pneumonia rate for the ICU Rationale: Performance measures focus on outcomes or processes that are used for internal improvement purposes, intra- or inter-organizational comparisons, and by various external entities for making decisions about care. An outcome measure is a measure that indicates the result of the performance (or nonperformance) of a function(s) or process(es). Outcome measures in infection prevention usually describe undesirable events, such as SSI, CLABSI, or CAUTI rates. Reference: APIC Text Online, Chapter 17 - Performance measures

Which of the following specimens can remain at room temperature after collection if transport to the lab will be delayed? A. Sputum B. Urine C. Stool D. Cerebral spinal fluid

D. Cerebral spinal fluid Rationale: Cerebral spinal fluid (CSF) should be transported to the laboratory immediately after collection if possible. If this is not possible, then it should be maintained at room temperature and transported within the ONE hour.

You are reviewing the serology marker results of a healthcare worker. The Hepatitis B serology marker results are as follows: HBsAg = Positive (+) anti-HBc = Positive (+) IgM anti-HBc = Negative (-) anti-HBs= Negative (-) What is the Hepatitis B status of this healthcare worker? A. Susceptible B. Immune due to natural infection C. Acutely infected D. Chronically infected

D. Chronically infected Rationale: Hepatitis B Surface Antigen (HBsAg) = The presence of HBsAg indicates that the person is infectious (regardless of acute or chronic). It is also used to make the vaccine. Total Hepatitis B Core Antibody (anti-HBc) = Appears at the onset of symptoms in acute HBV and persists for life. Its presence indicates previous or ongoing infection. IgM Antibody to Hepatitis B core Antigen (IgM anti-HBc) = Positivity indicates recent infection with Hepatitis B virus (<6 months). Its presence indicates an acute infection. Hepatitis B Surface Antigen Antibody (anti-HBs)= The presence of anti-HBs is generally interpreted as indicating recovery and immunity form Hepatitis B. Reference: CDC Interpretation of Hepatitis B Serologic Test Results table www.cdc.gov/hepatitis

With statistical process control (SPC), which of the following would be classified as common cause variation? A. New onset of Norovirus infections B. Consistent increase in incidence of catheter-associated infections over a 3-month period C. Sudden increase in rate of MRSA infection over a 3-month period D. Consistent slight variations in rates of MDRO infections each month for one year

D. Consistent slight variations in rates of MDRO infections each month for one year Rationale: According to statistical process control (SPC) process are either subject to common cause variation. Common cause variation is inherent to the process, such as consistent slight variations in rates of MDRO infections each month for one year. The variation is not related to any specific change in process or action. Special cause variation is associated with changes or problems with a process, such as a sudden onset of infections and consistent increases in incidence of infection, which may indicate inadequate prevention measures.

You receive an incident report that states a pediatric nurse received an exposure. The report is incomplete. It sates the nurse was scratched by a "needle" in a patient's room, while she was spiking an IV bag. You cannot tell at what point the exposure occurred or type of "needle" involved. Your first action should be: A. Determine if correct personal protective equipment was used or improper technique used. B. Determine if the source patient is infectious or high risk for blood-borne infectious disease. C. Determine who the source patient was and what his/her/their medical diagnosis was. D. Determine if a blood or body fluid exposure actually occurred.

D. Determine if a blood or body fluid exposure actually occurred. Rationale: Transmission of bloodborne pathogens from patient to healthcare workers is an important occupational hazard. The risk of bloodborne pathogen transmission following occupation exposure depends on a variety of factors including source patient factors, the type of injury and quantity of blood/body fluid transferred to the HCW during the exposure and the HCW's immune status. Before any further action should be taken the IP should collect further data on the incident to verify the presence of an occupational exposure.

A pregnant environmental services worker, who is nonimmune to varicella, enters the room of a patient with confirmed varicella (chickenpox) before an isolation sign is posted. She spends 6 minutes in the room with the patient (who is not wearing a mask). This exposure happened on January 11th. After giving the employee varicella-zoster immune globulin (VZIG), it is determined that the employee should be excluded from work. What day can she return to work? A. January 31 B. February 1 C. February 8 D. February 9

D. February 9 Rationale: The usual incubation period and exclusion from work is 10-21 days. However, if a caregiver receives the VZIG, they are excluded from work for 28 days, the caregiver would be able to return on the 29th day after the exposure.

The purpose of root cause analysis is to: A. Determine which individual made an error so that the employee may be disciplined or terminated. B. Review the basic processes that are in place and then turn that review over to a unit-specific team so that they can determine how they should modify their practices. C. Provide a process that requires little time or training but allows employees to identify culpability after an adverse event. D. Include participants from diverse areas of the organization to delve into the cause of an error or systems failure and identify changes in practice and/or policy that will prevent repeat of that error.

D. Include participants from diverse areas of the organization to delve into the cause of an error or systems failure and identify changes in practice and/or policy that will prevent repeat of that error.

Once the purpose of an educational program is established, the instructor must determine the specific actions the learner will perform. These actions are known as: A. Course guidelines B. Outcomes C. Educational Goals D. Instructional Objectives

D. Instructional Objectives An instructional objective describes an intended outcome. A usefully stated objective is stated in behavioral, or performance, terms that describe what the learner will be doing when demonstrating his/her achievement of the objective. An instructional objective must describe what the learner will be doing when demonstrating that he/she has reached the objective; describe the important conditions under which the learner will demonstrate his/her competence; and, indicate how the learner will be evaluated, or what constitutes acceptable performance. Reference: APIC Text, 4th edition, Chapter 3 Education and Learning

In 1997, the Joint Commission on Accreditation of Healthcare Organization (TJC) mandated the use of root cause analysis to: A. Document instances of medical malpractice B. Predict the occurrence of an incident C. Improve Staffing issues D. Investigate sentinel events in accredited hospitals

D. Investigate sentinel events in accredited hospitals A sentinel event is defined by the TJC as an event that results in loss of life, limb, or mental stability from an adverse event within the hospital.

All of the following are features of a well-written methods section except: A. clear criteria for defining cases and controls B. Methods of quality assurance C. Time period of the study D. Questions the researcher will answer

D. Questions the researcher will answer The methods section of a research paper provides the information by which a study's validity is judged. The methods section structure should: 1. describe the materials used in the study, 2. explain how the materials were prepared for the study, describe the research protocol, 3. explain how measurements were made and what calculations were performed, 4. and state which statistical tests were done to analyze the data. Reference: APIC Text, 5th edition, Chapter 20 Research study desig

One disadvantage of liquid sterilization is: A. Liquid sterilants are highly toxic and items must be aerated before use. B. It is a high heat process so it may not be used on heat-labile items. C. It is not an appropriate process for critical items. D. Items cannot be wrapped during the sterilization process so sterility cannot be maintained during storage.

D. Items cannot be wrapped during the sterilization process so sterility cannot be maintained during storage. Rationale: Liquid sterilization uses glutaraldehyde, hydrogen peroxide, or peracetic acid to sterilize items that are critical and are heat-sensitive. This is a liquid immersion process so items cannot be wrapped during the sterilization process. In addition, items may need to be rinsed after the process so sterility is not maintained in the rinsing, drying, and storage phases. Therefore, this method of sterilization is used only when an item cannot be sterilized in any other manner. Reference: APIC Text, 4th edition, Chapter 31 - Cleaning, Disinfection and Sterilization

The director of critical care has expressed a concern that there seems to be an increase in contaminated blood culture results in the ICU. An acceptable contamination level for blood cultures is: A. Less than 10 percent B. Greater than 3 percent C. Greater than 5 percent D. Less than 3 percent

D. Less than 3 percent Rationale: Bacteremia is a significant cause of morbidity and mortality in hospitalized patients. Accurate and timely identification of the causative organism is imperative. Blood cultures are considered the "gold standard" in the diagnosis and treatment of bacteremia. However, the prognostic value of blood cultures is limited by contamination. Contamination, or false positive blood cultures, occurs when organisms that are not present in the blood are grown in culture. Blood culture contamination rates of less than 3 percent are desired. Reference: The Infection Preventionist's Guide to the Lab, Chapter: 3, Page: 39 CBIC Core Competency: Identification of Infectious Disease Processes

An example of a selective media that inhibits Gram-positive bacteria is: A. Sabouraud agar B. Chocolate agar C. Typticase Soy agar D. MacConkey's agar

D. MacConkey's agar

The most common organism associated with pneumonia in school-aged children and young adults is: A. Neisseria meningitidis B. Streptococcus pneumoniae C. Staphylococcus aureus D. Mycoplasma pneumoniae

D. Mycoplasma pneumoniae Rationale: Mycoplasma is uncommon under the age of 5 but is the leading cause of pneumonia in school-aged children and young adults. It can occur during any season and occurs throughout the world. Reference: Mycoplasma pneumoniae and Other Mycoplasma Species Infections. In: Pickering LK, ed. Red Book: 2012 Report of the Committee on Infectious Diseases, 29th edition. Elk Grove Village, IL: American Academy of Pediatrics, 2012. CBIC Core Competency: Identification of Infectious Disease Processes

The IP is reviewing the chart of a patient with a sputum culture positive for pathogens. Which of the following findings indicates that the specimen had been properly collected from a patient with possible bacterial pneumonia? A. Presence of Blood B. Variety of both gram-positive and gram-negative bacteria. C. Many epithelial cells and few neutrophils. D. Numerous neutrophils and few if any epithelial cells.

D. Numerous neutrophils and few if any epithelial cells. Useful sputum culture results rely heavily on good sample collection. If examination of a Gram stain of the sample reveals that it contains a significant number of normal cells that line the mouth (squamous epithelium cells), then the sample is not generally considered adequate for culture and a recollection of the sample may be required. If the sample contains a majority of neutrophils that indicates a body's response to an infection, then it is considered to be an adequate sample for culturing. APIC Text ch 24

Which US agency requires a respiratory program for HCP? A. Food and Drug Administration (FDA) B. The Joint Commission (TJC) C. Centers for Disease Control and Prevention (CDC) D. Occupational Safety and Health Administration (OSHA)

D. Occupational Safety and Health Administration (OSHA) Rationale: The OSHA respiratory protection standard requires that the employer implement a respiratory protection program with a qualified administrator as the person who oversees the program, including evaluating the effectiveness of the program. The standard also requires that each worker assigned to wear a respirator receive a fit test before wearing the respirator in the workplace and perform a seal check with each use.

A caregiver has reached out to Occupational health with concerns about caring for patients in isolation. Which disease should the pregnant caregiver be excluded from A. Varicella-Zoster B. Meningitis C. TB D. Parvovirus B19

D. Parvovirus B19 Rationale: Patients who maybe shedding certian infectious agents for which there is no vaccine may cause the pregnant HCP to be concerned regarding their safety and the safety of the fetus. These may include patents with: 1. Cytomegalovirus 2. Parvovirus B19 3. Herpes simplex virus 4. Syphilis 5. Rubeola 6. Rubella 7. Varicella Although respiratory syncytial virus (RSV) infection does not have an adverse effect on pregnant women, exposure to the ribavirin aerosol used for treatment of RSV is contraindicated during pregnancy based on theoretical, but unproven, adverse effects during pregnancy.

If an IP would like to describe the beliefs of nurses who would be expected to care for victims of a pandemic influenza, which research design would be conducive to understanding how nurses perceive this phenomenon? A. Cohort B. Case-control C. Mixed methods D. Qualitative

D. Qualitative Qualitative research is primarily exploratory research. It is used to gain an understanding of underlying reasons, opinions, and motivations. It provides insight into the problem or helps to develop ideas or hypothesis for potential quantitative research. It may also be used to uncover trends in thoughts and opinions.

Aseptic technique is defined as: A. No touch technique B. A process used in the operating room C. An absence of organisms D. The process for keeping away disease-producing organisms

D. The process for keeping away disease-producing organisms Rationale: Aseptic techniques, defined as the process for keeping away disease-producing micro-organisms, may be used in any clinical setting. It is implemented to protect the patient by minimizing contamination to reduce the risk for infection. Reference: APIC Text, 4th edition, Chapter 30 - Aseptic Technique

Which of the Five Tools of Patient Safety is the study of a process to achieve "Failure-Free" operation over time to reduce process defects and improve system safety? A. Human Factors Engineering B. Human Factors Analysis C. Resiliency D. Reliability Science

D. Reliability Science Rationale: Reliability Science is the study of a process to achieve "Failure-Free" operation over time to reduce process defects and improve system safety Human Factors Engineering (HFE) = Goal is to create designs that are safe, comfortable and effective for human use Human Factors Analysis = Study of elements involving a human-machine interface with the intent of improving working condition or operations. Ergonomics = study of people at work and then design tasks, jobs, information, tools, equipment, facilities, and the working environment so that people can be safe, effective, productive, and comfortable. Resiliency = the intrinsic ability of a system to adjust and sustain operations during periods of stress or after an event.

A hospital has admitted an immunocompetent patient with localized herpes zoster. During unit rounds, the IP notes a sign on the door that says "NO PREGNANT WOMEN." How should the IP best respond to this? A. Leave the sign on the door because the patient poses a high risk of disease transmission to pregnant women. B. Remove the sign from the door but ask the charge RN to not assign the patient to any pregnant HCP and to prevent pregnant visitors from entering the room. C. Remove the sign from the door and place the patient in both Contact and Airborne precautions. D. Remove the sign from the door and ensure that all lesions are completely covered; Standard precautions are sufficient to prevent the spread of the virus.

D. Remove the sign from the door and ensure that all lesions are completely covered; Standard precautions are sufficient to prevent the spread of the virus. Rationale: according to CDC recommendations, following standard precautions and completely covering lesions are sufficient to prevent transmission of localized zoster to susceptible visitors and personnel. IF a woman has a history of varicella-zoster infection or vaccination, her antibodies will protect her fetus from infection, so there is no risk for her to enter the room or care for the patient. Infections with the zoster virus during pregnancy can be harmful to the fetus; however, all HCP should be immune to zoster through a history of illness or through immunization upon employment. If unvaccinated or susceptible HCP is at risk for severe disease and for whom vaccination is contraindicated (eg pregnant HCP) is exposed, the CDC recommends that that personnel receive VZIG after exposure. Reference: APIC Text - Herpes Virus chapter and the Immunization of Healthcare Personnel chapter

Decontamination is the process by which an item is: A. Cleaned of all soil and germs B. Rendered free from all pathogens and infectious organisms C. Sterilized and ready for use D. Rendered safe for handling without protective attire

D. Rendered safe for handling without protective attire Decontamination according to OSHA, is defined as "the use of physical or chemical means to remove, inactivate, or destroy bloodborne pathogens on a surface or item to the point where they are no longer capable of transmitting infectious particles and the surface or item is rendered safe for handling, use, or disposal" [29 CFR 1910.1030]. In health-care facilities, the term generally refers to all pathogenic organisms. Reference: Guideline for Disinfection and Sterilization in Healthcare Facilities, 2008 Rutala WA, Weber DJ, Healthcare Infection Control Practices Advisory Committee

The Director of the Operating Room (OR) requests that the OR surfaces be routinely environmentally cultured. The IPs best response should be: A. A schedule for routine culturing of the OR should be arranged so that each room is cultured at a set interval. B. Routine culturing of the OR should be done in the absence of any epidemiologic investigations in that area. C. Routine culturing should not be done because it is too expensive. D. Routine culturing should not be considered unless an epidemiologic investigation is being conducted.

D. Routine culturing should not be considered unless an epidemiologic investigation is being conducted.

A pediatric patient is admitted to the inpatient unit on droplet precautions. The patient is experiencing a facial rash that resembles a "slapped cheek" and a reticular pattern or "Lace-like" rash on the arms. The RN on the pediatric unit calls Employee Health and Infection Prevention with questions about work restrictions due to being within their 2nd trimester of pregnancy. What is the best response to this RN as the IP? A. Since the patient is in Droplet isolation, your mask will protect you from any concerns related to potential exposure. B. The RN should be removed from the patient's primary care team until 4 days after the measles rash appeared. C. Out of an abundance of caution, the RN should be excluded from working on the pediatric floor until the pregnancy is completed for the safety of the fetus. D. The RN should be removed from the patient's primary care team until Parvovirus B19 is ruled-out.

D. The RN should be cautioned and removed from the patient's primary care team until Parvovirus B19 is ruled-out. Parvovirus B19 is the causative agent of erythema infectiosum, aka fifth disease. This agent is of concern to pregnant HCP because B19 can cause infection of fetal red blood cell precursors and lead to severe anemia and high-output cardiac failure in the fetus, hydrops fetalis, and fetal death. 5% of infections in the first 20 weeks of pregnancy will result in fetal death and often early miscarriages.

The need to include employee education for infection prevention is included in all of the following programs components except: A. The facility infection prevention risk assessment B. The facility's infection prevention program plan C. The annual facility budget and allocation of resources D. The facility's 5-year strategic plan

D. The facility's 5-year strategic plan Rationale: Strategic plans determine the direction an organization will go in the future and what the organization must do in order to reach the goal, mission, or vision. Strategic planning involves several important steps: (1) an analysis of the organization, (2) forming conclusions about what an organization must do as a result of issues facing the organization, and (3) action planning. Action plans determine what tactic the organization will use to accomplish goals, who will take responsibility to carry out the action, the timeline of action, and resources and evaluation criteria. A facility's strategic plan does not include department-specific goal. References: APIC Text, 4th edition, Chapter 3 - Education and Training; APIC Text, 4th edition, Chapter 16 - Quality Concepts

Which of the following student learning objectives falls within the "Evaluate" domain of Bloom's Taxonomy Cognitive Levels? A. The student will compare and contrast the available cleaning agents. B. The student will define what a cleaning agent is. C. The student will describe the types of cleaning agents available in the facility. D. The student will recommend appropriate environmental cleaning agents.

D. The student will recommend appropriate environmental cleaning agents. Bloom's Taxonomy is used to promote higher levels of thinking in learners. The six levels of Bloom's taxonomy are: analyze, evaluate, create, apply, understand, and remember. Action verbs used to describe learning objectives that fall within the "evaluate" domain of Bloom's taxonomy include: recommend, evaluate, and appraise. Reference: APIC Text, 4th edition, Chapter 3 Education and Learning

The IP is reviewing the immunization records of healthcare personnel at their facility and discovers that employees born before 1957 do not have any record of receiving MMR vaccine. What should the IP recommend to the Human Resources Director regarding employees born before 1957? A. They are considered immune and do not require follow-up B. They should receive two doses of the vaccine 4 weeks apart C. They are only required to provide proof of immunity to measles D. They are required to provide proof of immunity to measles, mumps, and rubella

D. They are required to provide proof of immunity to measles, mumps, and rubella Rationale: All persons working in healthcare facilities should be immune to measles, mumps, and rubella. It is reasonable to require proof of immunization. Proof of immunity consists of documented vaccination with one or more doses of live vaccine on or after the first birthday, laboratory evidence of immunity to the disease, or history of these diseases based on a healthcare worker diagnosis. Although birth before 1957 is considered acceptable evidence of measles, rubella, and mumps immunity, ACIP recommends that healthcare facilities consider vaccinating unvaccinated personnel born before 1957 who do not have laboratory evidence of measles, rubella, and mumps immunity; laboratory confirmation of disease; or vaccination with two appropriately spaced doses of MMR vaccine for measles and mumps and one dose of MMR vaccine for rubella.

A technician in the OR has reported an infestation of fruit flies in the sterile instrument storage room. The sterile packs were temporarily removed and pest management was called in to eradicate the flies. The OR staff want to know whether they can use the sterile packs. Which of the following is the best response? A. The OR staff can use the packs without any reprocessing because there are no flies visible on the outside of the packs. B. The OR staff can use the packs without any reprocessing if they open a test pack and don't find any flies inside it. C. The OR staff don't need to repack the items but they do need to run the packs through the sterilizer again. D. They need to completely reprocess all of the items that were in the affected room.

D. They need to completely reprocess all of the items that were in the affected room. Rationale: The presence of any type of vermin can affect the integrity of the packaging and contaminate the contents. Fruit flies are small enough to crawl into the sterile packs and potentially contaminate the instruments. They could also become trapped in the packs and leave debris on the instruments. The instruments should be unwrapped, visible inspected, cleaned if necessary, and reprocessed. Reference: APIC Text, 4th edition, Chapter 106 - Sterile Processing

All are the key components of a Strong Safety Culture, except: A. Generative, uneasy about risk, constantly seeking out best practices, always looking for where the next mistake is going to happen, and then working to prevent it. B. Fosters a learning organization, where staff members share information about mistakes and errors in order to prevent them from reoccurring C. Emphasizes reciprocal accountability, everyone holds each other accountable D. Utilizes a multidisipanary team to conduct RCAs to identify mistakes and or gaps within healthcare processes and compares to industry best practice before implementation.

D. Utilizes a multidisipanary team to conduct RCAs to identify mistakes and or gaps within healthcare processes and compares to industry best practice before implementation. Rationale: an RCA is used to identify the breakdown in a process that is already implemented. A gap analysis is used to compare current practice with industry best practice.

You are developing an education program to provide annual blood borne pathogen training to healthcare personnel in your facility. There are approximately 7,500 employees who need annual training, and your team of 5 IPs has to provide and document this training while still carrying out all other department duties. What is the best method of delivery for this training? A. Group lecture sessions B. Educational cart C. Role play training sessions D. Web-based training sessions

D. Web-based training sessions Although each of the delivery methods listed has benefits, the best method in this case is web-based training. It is easy to deliver to a large number of people, flexible so that people with varied schedules can complete the training at a time that is convenient for them, and easy to track by having employees register for training through an online system. Assessments can be built into web-based training so that learners can evaluate their understanding of the training while it is in progress and demonstrate their mastery of the material at the end of the training session. Reference: APIC text, 4th edition, Chapter 3 - Education and Training

What is ordinal data?

Data that can be placed in a sequence or meaningful order, or rank between options. Examples: - Income level = Low, middle, high - Level of agreement = agree, neutral, disagree As you can see in these examples, all the options are still categories, but there is an ordering or ranking difference between the options. You can't numerically measure the differences between the options (because they are categories, after all), but you can order and/or logically rank them. So, you can view ordinal as a slightly more sophisticated level of measurement than nominal.

Bloom's Taxonomy: Remembering

Define, duplicate, list, memorize, recall, repeat, reproduce state. Example: "I recall it is Eyeshadow"

Instructional Objectives

Describe learner outcomes in MEASURABLE terms and use action verbs such as discuss, describe, demonstrate, compare, or evaluate.

Descriptive statistics

Descriptive statistics refers to data analysis that helps describe, show, or summarize the data in a meaningful way such that patterns may emerge from the data. Descriptive statistics do not allow us to make any conclusions beyond the data analyze or reach conclusions regarding any hypothesis we might have made. They are simply a way to describe the data.

What is the difference between Detergents and Disinfectants?

Detergents: - More environmentally friendly, and produce less offensive odors - cost less than disinfectants - Do not effectively remove microorganisms from surfaces Disinfectants: - Disinfectants are divided into low, intermediate, and High levels - Have different kill capabilities based on levels

The Strategic Plan

Determines the direction an organization will go in the future and what the organization must do in order to reach the goal, mission, or vision. Steps: 1. An analysis of the organization 2. Forming conclusions about what an organization must do as a result of issues facing the organization. 3. Action planning

Quaternary Ammonium Compounds (Quats)

Disinfectant that is considered nontoxic, odorless, and fast-acting.

Decontamination

Disinfected and cleaned and therefore rendered safe for handling without PPE.

True or False The average risk of seroconversion after a percutaneous injury involving blood infected with HIV is approximately 30%.

False HIV conversion after percutaneous injury involving blood is 0.3%

True or False The ICRA is a written plan describe how the release of potentially infectious agents from the construction activates will be controlled.

False The 2006 FGI guidelines added the requirement for written infection control risk mitigation recommendations (ICRMR) describing how the transmission of biological contaminants from construction zones will be continued and controlled

True or False: Personnel who have laryngeal or pulmonary TB are excluded from work until they are receiving adequate therapy, the cough has resolved, and there have been two consecutive sputum smears negative for acid-fast bacilli.

False There needs to be 3 consecutive sputum smears negative for AFB.

True of False: The generally accepted respiratory protection that is used to protect personnel from a person with suspected or confirmed TB is a particulate N95 respirator, a requirement identified in CDCs respiratory protection standard.

False This is a requirement from OSHA, not the CDC

True or False Water from flower vases has been identified as a disease risk for immunocompetent patients because potential pathogens have been isolated from such water sources.

False Water from flower vases has been identified as a disease risk for IMMUNOCOMPROMISED patients because potential pathogens have been isolated from such water sources.

Data Collection Techniques for Qualitative Data

Focus Groups - explore beliefs of participants, perceptions and concerns. Participant Observation - collecting data of the participants in the natural environment without impacting/initiating the environment. Interviews - ask open ended questions. Ask how to describe the process? Field Notes - body language. how things are stored.

Evaluation

Formative evaluation (internal) Summative evaluation (external)

What is the difference between Sodium Hypochlorite and Bleach?

Hypochlorite is an ion composed of chlorine and oxygen with the chemical formula ClO−. Being unstable in the pure form, hypochlorite is most commonly used for bleaching, disinfectation, and water treatment purposes in its salt form, sodium hypochlorite. Bleach should be stored between 2-8°C (50-70°F). a 1:10 bleach solution has a shelf life of 24 hours. Bleach must be stored separately from corrosives, soaps, detergents or other cleaning products. Minimum Personal Protective Equipment (PPE) that MUST be worn when preparing or handling a sodium hypochlorite solution: Nitrile Rubber Gloves; and Approved Safety Glasses; and Respiratory protection (NIOSH Approved) should be used any time there is the potential for exposure to vapor and/or dust and a fume hood cannot be used; and Lab coat. If sodium hypochlorite comes in contact with other cleaners containing ammonia or chlorine compounds, fatal levels for ammonia gas or chlorine gas can be produced. Never mix bleach with other chemicals. For site decontamination of spills of blood or other potentially infectious materials (OPIM), implement the following procedures. Use protective gloves and other PPE (e.g., when sharps are involved use forceps to pick up sharps, and discard these items in a puncture-resistant container) appropriate for this task. Disinfect areas contaminated with blood spills using an EPA-registered tuberculocidal agent, a registered germicide on the EPA Lists D and E (i.e., products with specific label claims for HIV or HBV or freshly diluted hypochlorite solution. Category II, IC. 1) * If sodium hypochlorite solutions are selected use a 1:100 dilution (e.g., 1:100 dilution of a 5.25-6.15% sodium hypochlorite provides 525-615 ppm available chlorine) to decontaminate nonporous surfaces after a small spill (e.g., <10 mL) of either blood or OPIM. If a spill involves large amounts (e.g., >10 mL) of blood or OPIM, or involves a culture spill in the laboratory, use a 1:10 dilution for the first application of hypochlorite solution before cleaning in order to reduce the risk of infection during the cleaning process in the event of a sharp injury. Follow this decontamination process with a terminal disinfection, using a 1:100 dilution of sodium hypochlorite. Category IB, IC.63, 215, 557

Foodborne illness: Campylobacter jejuni

Incubation Period = 2-5 days Signs & Symptoms = Diarrhea, cramps, fever, and vomiting; diarrhea may be bloody. Duration = 2-10 days Food Sources = Raw and undercooked poultry, unpasteurized milk, contaminated water

Foodborne illness: Listeria monocytogenes

Incubation Period = 9-48 hrs for gastrointestinal symptoms, 2-6 weeks for invasive disease Signs & Symptoms = Fever, muscle aches, and nausea or diarrhea. Pregnant women may have mild flu-like illness, and infection can lead to premature delivery or stillbirth. The elderly or immunocompromised patients may develop bacteremia or meningitis Duration = Variable Food Sources = Unpasteurized milk, soft cheeses made with unpasteurized milk, ready-to-eat deli meats

Macrolides

Inhibit protein synthesis; mostly bacteriostatic (arrest growth), therefore used for less serious infections Examples: erythromycin, clarithromycin, azithromycin Commonly used for: Chlamydia

Isolation Initiation and Duration: Rash with Fever

Isolation Type: Airborne Respirator & Contact Duration: Until physician diagnosis or measles and chickenpox have been ruled out

Isolation Initiation and Duration: Lice (prediculosis)

Isolation Type: Contact Duration: 24 hours after initiation of effective therapy AND no visible infestation/eggs

Isolation Initiation and Duration: Scabies

Isolation Type: Contact Duration: 24 hours after initiation of effective therapy AND no visible infestation/eggs

Isolation Initiation and Duration: Conjunctivitis

Isolation Type: Contact Duration: Duration of illness - symptom resolution

Isolation Initiation and Duration: MDR-GNR (Multidrug-resistant Gram-Negative Rod)

Isolation Type: Contact Duration: Patient to remain in precautions (No discontinuation policy at this time)

Isolation Initiation and Duration: Shigella

Isolation Type: Contact Duration: PeaceHealth is more stringent than CDC guidance due to literature regarding spread and the small number of organisms required for transmission. Follow Contact Precautions for duration of illness.

Isolation Initiation and Duration: Wound drainage, major

Isolation Type: Contact Duration: Until drainage stops or can be adequately contained with a dressing

Isolation Initiation and Duration: CRE (Carbapenem-resistant Enterobacteriaceae) Non-CP (Carbapenemase) producer

Isolation Type: Contact Duration: Patient to remain in precautions (No discontinuation policy at this time)

Isolation Initiation and Duration: Rotavirus

Isolation Type: Contact Enteric Duration: Until 48 hours of no symptoms (vomiting/diarrhea) and until environment has been terminally cleaned

Isolation Initiation and Duration: Diarrhea of Unknown Cause

Isolation Type: Contact Enteric Duration: Until physician diagnosis, laboratory results, or resolution of symptoms without intervention

Isolation Initiation and Duration: MRSA (Methicillin-resistant Staphylococcus aureus)

Isolation Type: Contact or Standard (depending on State Laws) Duration: MARCH 2020 UPDATE: Suspended to conserve PPE as allowed by the Washington Department of Health. This supersedes all other guidance. Only required for lab positive in current admission.

Isolation Initiation and Duration: Adenovirus (pneumonia)

Isolation Type: Contact/Droplet Duration: 7 days after illness onset or until 24 hours after the resolution of fever and respiratory symptoms, whichever is longer. *In immunocompromised hosts, extended duration of droplet and Contact precautions due to prolonged shedding of virus.

Isolation Initiation and Duration: Coronavirus (other than MERS, SARS, or COVID-19)

Isolation Type: Droplet Duration: 7 days after illness onset or until 24 hours after resolution of fever and respiratory symptoms, whichever is longer

Obligate Aerobes

Only aerobic growth, oxygen required

Cleaning and Disinfecting Environmental Surfaces in Healthcare Facilities: CDC Disinfectant Detergent Solution Recommendations

Prepare disinfecting (or detergent) solutions as needed and replace these with fresh solution frequently (e.g., replace floor mopping solution every three patient rooms, change no less often than at 60-minute intervals), according to the facility's policy. Category IB.

What are the three types of CJD cases?

Sporadic, Familial, and Iatrogenic

Sterilization- high temperature

Steam and dry heat

When disease transmission from direct contact with waterborne microorganisms is considered, what pathogen(s) should you suspect?

The 3 most commonly reported pathogens identified are: 1. Pseudomonas 2. Atypical (non-TB) Mycobacteria 3. Legionella

p-value

The P value or calculated probability is the estimated probability of rejecting the null hypothesis (Ho) of a study question when that hypothesis is true. Small p-value (<0.05) = indicates strong evidence against the null hypothesis, so you reject the null hypothesis. " if the P is low, the Ho has got to go" Large p-value (>0.05) = indicates weak evidence against the null hypothesis, so you failed to reject the null hypothesis. p-value close to cutoff (0.05) = consider to be marginal (could go either way).

Sensitivity

The ability of a test to detect ALL TRUE cases of the disease. = (Number of true-positive results) / (the number of true-positive + false-negative results) High sensitivity = High False Positives. High sensitivity = TRUE NEGATIVES. You will not have any false negative tests.

Minimum inhibitory concentration (MIC)

The lowest concentration of an anti-microbial drug that will inhibit the visible growth of a micro organism after overnight incubation.

Which waterborne pathogens are most often related to outbreaks of contaminated equipment?

The two pathogens that are most often related to contaminated equipment outbreaks are: 1. Pseudomonas 2. Atypical (non-TB) Mycobacteria

Fishbone or Ishikawa Diagram

This is an efficient way of problem solving by isolating specific components and deconstructing positive or negative consequences through an investigatory lens. Often used after a RCA is completed.

Calcofluor White Stain

Used to stain fungus, yeast, molds

Steam sterilization (autoclave)

Uses steam under pressure to obtain high temperature of 250-254F with exposure times of 20-40 minutes defending on the item being sterilized. Kills bacterial spores and all other micro organisms. Can only be used for items that are tolerant of high temperatures.

Which agency or agencies has regulatory authority over some aspects of healthcare facility design, construction, and renovation related to Infection Prevention and Control? 1) Occupational Health and Safety Administration (OSHA) 2) National Institute for Occupational Safety and Health (NIOSH) 3) National Institutes of Health (NIH) 4) Centers for Disease Control and Prevention (CDC) a. 1 b. 1, 2 c. 2, 3, 4 d. 1, 2, 3, 4

a. 1 Rationale: OSHA is the only agency listed above that has regulatory authority over any aspect of healthcare facility design, construction, and renovation. OSHA has set regulations for engineering controls related to bloodborne pathogens and to isolation of suspected or confirmed cases of Tuberculosis in an airborne isolation room. Reference: Occupational Health and Safety Administration (OSHA) https://www.osha.gov CBIC Core Competency: Environment of Care

A patient is receiving eye drops to treat conjunctivitis. The drops are only available in a multi-dose vials. Which of the following is/are acceptable practices for this use of this medication? 1) This vial of medication may not be used on multiple patients 2) This vial of medication may be used on multiple patients if it has not come into contact with tears or the conjunctiva 3) This vial of medication may be used on multiple patients if it is used within 28 days of being opened 4) This vial of medication should be stored away from the patient care area a. 1 b. 1, 4 c. 2, 3 d. 2, 3, 4

a. 1 Rationale: Multidose eye drops or creams are not acceptable for reuse on other patients if they have been used on a patient with an infectious disease of the eye. If the patient does not have an infectious disease of the eye, then the multidose vial may be used for other patients provided that: - Tt has not come into contact with the conjunctiva or tears of a patient. - This medication must be used within 28 days of being opened or per the manufacturer's recommendations. - Multidose medications that will be used on more than one patient must be stored away from the patient care area. Reference: APIC Text, 4th edition, Chapter 64 - Ambulatory Surgery Centers CBIC Core Competency: Preventing/Controlling the Transmission of Infectious Agents

Frequency histograms and polygons are most useful for variables of what level of measurement? 1) Interval scale 2) Ratio scale 3) Nominal scale 4) Ordinal scale a. 1, 2 b. 1, 3 c. 2, 3 d. 3, 4

a. 1, 2 Rationale: A histogram is a graphic of a frequency distribution in which one bar is used for each time interval, and there is no space between the intervals (see Figure PE3-1). It is used to portray the (grouped) frequency distribution of a variable at the interval or ratio level of measurement. A frequency polygon (see Figure PE3-2) is similar to a line graph, but each coordinate point is represented by a point displayed on the graph with straight lines connecting them. A frequency polygon will provide the same data information as a histogram. References: APIC Text, 4th edition, Chapter 13 - Use of Statistics in Infection Prevention; APIC Text, 4th edition, Chapter 10 - General Principles of Epidemiology CBIC Core Competency: Surveillance and Epidemiologic Investigation

Using Improved Hydrogen Peroxide offers many advantages. Those advantages are: 1) Rapid action 2) Low toxicity 3) Effective against spores 4) Low cost a. 1, 2 b. 2, 3 c. 2, 4 d. 1, 4

a. 1, 2 Rationale: A major advantage of improved hydrogen peroxide is its rapid action, high effectiveness, and low toxicity. A disadvantage is it is more expensive than a quaternary ammonium compound in ready-to-use containers. Improved hydrogen peroxide is considered safe for humans and equipment, and benign for the environment. In fact, improved hydrogen peroxide has the lowest EPA toxicity category based on its oral, inhalation, and dermal toxicity, which means that it is practically nontoxic and is not an irritant. Reference: APIC Text, 4th edition, Chapter 31 - Cleaning, Disinfection and Sterilization CBIC Core Competency: Cleaning, Sterilization, Disinfection, Asepsis

A patient with a positive HBsAg must be dialyzed. Which of the following are the best measures to prevent the transmission of disease? 1) Perform treatment in a separate room or area 2) Use a dedicated machine and equipment 3) Perform the treatment at the end of the day 4) Schedule the treatment on a separate day with other positive HBsAg patients a. 1, 2 b. 2, 4 c. 3, 4 d. 1, 3

a. 1, 2 Rationale: The CDC recommends separating HBsAg patients by room or area and using a separate, dedicated machine and equipment to reduce the risk of transmission of HBV in the dialysis setting. Patients who are known to be positive for HBsAg should be excluded from reprocessing programs because of the risk of transmission to susceptible reuse personnel. The incidence of HBsAg has been found to be higher in dialysis units that do not follow recommendations. Reference: APIC Text, 4th edition, Chapter 39 - Dialysis; Centers for Disease Control and Prevention (CDC). Recommendations for Preventing Transmission of Infections Among Chronic Hemodialysis Patients. MMWR 2001 April 27;50(RR05):1-43. CBIC Core Competency: Preventing/Controlling the Transmission of Infectious Agents

While rounding in an ambulatory care center, an IP discovers that healthcare personnel (HCP) have been using single-dose vials (SDVs) of lidocaine for multiple patients. She informs the clinic manager that the practice must end immediately. Which of the following statements about SDVs should the IP include in her explanation to the clinic manager? 1) SDVs lack antimicrobial preservatives 2) Inappropriate use of SDVs can lead to contamination 3) A needleless access device (spike) must be applied when reusing an SDV 4) All medications from an SDV must be prepared in a pharmacy a. 1, 2 b. 2, 3 c. 3, 4 d. 2, 4

a. 1, 2 Rationale: The CDC's guidelines call for medications labeled as "single dose" or "single-use" to be used for only one patient. This practice protects patients from life-threatening infections that occur when medications get contaminated from unsafe use. Vials labeled by the manufacturer as "single dose" or "single-use" should only be used for a single patient. These medications typically lack antimicrobial preservatives and can become contaminated and serve as a source of infection when they are used inappropriately. Reference: APIC Text, 4th edition, Chapter 48 - Ambulatory Care CBIC Core Competency: Preventing/Controlling the Transmission of Infectious Agents

The following factors should be considered when preparing the environment for an educational program: 1) Providing an atmosphere of mutual respect and support 2) Establishing a comfortable environment conducive to learning 3) Encouraging interaction by arranging desks in straight rows 4) Having a person control and troubleshoot the environment as needed a. 1, 2, 4 b. 2, 3, 4 c. 1, 3, 4 d. 1, 2, 3

a. 1, 2, 4 Option 3 is not conducive to promoting an educational environment because desks in straight rows do not promote engaging discussions between classmates. Rationale: One of the most important roles of the educator is to provide an atmosphere of mutual respect, as well as one that is friendly, informal, and supportive. Eye contact, addressing students by name, listening without interrupting, and acknowledging the validity of problems or opinions expressed are characteristics of an effective educator. The educator must also take steps to create an environment that is comfortable and conducive to learning. The learning space should be private and congenial with careful consideration to seating, room temperature, and lighting. There should be a contact person at the learning site to troubleshoot any facility or technical problems should they occur. The traditional classroom setup with straight rows of desks does not promote interaction. Reference: APIC Text, 4th edition, Chapter 3 - Education and Training CBIC Core Competency: Education and Research

The IP exits the elevator and finds a major renovation area with non-intact barriers near the operating room and ICUs. Dust and debris are evident in the corridor. A meeting is immediately arranged between the hospital engineering director and the contractor. The IP stresses the following: 1. An airtight barrier must be installed from floor to ceiling and taped closed with duct tape 2. All debris hauled out of the renovation area must be in tightly closed containers 3. All dust tracked outside the area must be removed immediately, either by damp mopping or HEPA-filtered vacuum 4. All ventilation ducts must be blocked within the construction area a. 1,2,3 b. 1,3,4 c. 1,2,4 d. 2,3,4

a. 1,2,3

Seventy-five patients were admitted to the Medical-Surgical ICU. Forty were on the surgical service and 35 were on the medical service. Fifteen patients developed a healthcare-associated MRSA infection. Nine of the patients with MRSA infection were on the surgical service. There were 230 patient days in the ICU for the surgical patients in January, and 325 patient days for medical patients. What was the overall MRSA attack rate? a. 20 percent b. 2 percent c. 53 percent d. 5 percent

a. 20 percent Rationale: An attack rate is a special form of incidence rate. It is not truly a rate, but a proportion. It is the proportion of persons at risk who become infected over an entire period of exposure or a measure of the risk or probability of becoming a case. It is usually expressed as a percentage and is used almost exclusively for epidemics or outbreaks of disease where a specific population is exposed to a disease for a limited time. The attack rate equals the number of new cases of disease (for a specified time period) divided by the population at risk for the same time period multiplied by 100. Attack rate is the same as incidence rate, except that attack rates are always expressed as cases per 100 populations or as a percentage. The attack rate for this scenario is calculated as follows: 15 ÷ 75 × 100= 20 percent. Reference: APIC Text, 4th edition, Chapter 13 - Use of Statistics in Infection Prevention CBIC Core Competency: Surveillance and Epidemiologic Investigation

Of the following sharp object injury examples, which would have the highest risk of transmission of bloodborne pathogens (assuming that all patient-related risk factors are identical for each example)? a. A nurse is stuck with an intravenous (IV) catheter stylet after withdrawing the stylet from the catheter b. A medical resident is stuck with a suture needle that had been used to suture a head wound c. A nurse is stuck with the needle from a syringe that had been used to give an intramuscular injection d. A surgeon sustains a superficial skin injury from a used disposable scalpel

a. A nurse is stuck with an intravenous (IV) catheter stylet after withdrawing the stylet from the catheter Rationale: IV catheter stylets are involved in only about 3 percent of sharp object injuries, but they have the highest risk of transmission of bloodborne pathogens because they are hollow-bore needles that can be filled with blood. This results in greater exposure to bloodborne pathogens. References: APIC Text, 4th edition, Chapter 105 - Minimizing Exposure to Blood and Body Fluids; Centers for Disease Control and Prevention (CDC). Updated U.S. Public Health Service guidelines for the management of occupational exposure to HBV, HCV, and HIV and recommendations for postexposure prophylaxis. MMWR 2001 June 29;50(RR11):1-42. CBIC Core Competency: Employee/Occupational Health

Which of the following terms refers to patient harm that is the result of treatment by the healthcare system rather than from the health condition of the patient? a. Adverse event b. Dire consequence c. Unanticipated event d. Sentinel event

a. Adverse event Rationale: An adverse event is an unintended consequence of healthcare or services that results in a negative patient outcome (e.g., infection or physical or psychological injury). Incidents such as patient falls or improper administration of medications are also considered adverse events even if there is no permanent effect on the patient. References: APIC Text, 4th edition, Chapter 18 - Patient Safety; APIC Text, 4th edition, Chapter 16 - Quality Concepts

A patient has been diagnosed with Legionnaires' disease was possibly acquired during his stay in the hospital. What is the recommended first response to this incident? a. All water sites in the healthcare facility to which the patient was exposed should be culture for L pneumophila b. The patient should be placed on respiratory isolation until 24 hours after treatment has started c. The water system should be superheated and flushed immediately d. All patients should immediately be provided with bottled water for drinking

a. All water sites in the healthcare facility to which the patient was exposed should be culture for L pneumophila

Which of the following is the primary method to prevent influenza? a. Annual vaccination b. Hand washing c. Droplet Precautions d. Promotion of respiratory hygiene/cough etiquette

a. Annual vaccination Rationale: Vaccination is the primary method for preventing influenza and its complications. Recommendations for vaccine and antiviral drug use are published regularly by the ACIP. All persons aged 6 months and older should be vaccinated annually unless specific medical contraindications exist. References: APIC Text, 4th edition, Chapter 82 - Influenza; Appendix C. In: Brooks K. Ready Reference for Microbes, 3rd edition. Washington, DC: Association for Professionals in Infection Control and Epidemiology, 2012. CBIC Core Competency: Preventing/Controlling the Transmission of Infectious Agents

During outbreaks, the CDC recommends that high-touch surfaces in patient rooms be cleaned and disinfected: a. At least three times a day b. Hourly c. At least twice a day d. Only when soiled

a. At least three times a day Rationale: During outbreaks, the environment, especially high-touch surfaces, is heavily contaminated with microorganisms. High-touch surfaces include commodes, toilets, toilet handles, faucets, bathroom rails, bed rails, telephones, computers, and food preparation areas. The CDC recommends that these surfaces be cleaned and disinfected at least three times a day and that low-touch surfaces be cleaned at least twice a day. Reference: APIC Text, 4th edition, Chapter 107 - Environmental Services CBIC Core Competency: Environment of Care

Which of the following educational tools is most effective to bridge the learning gap between theory and actual practice? a. Case study b. Lecture c. Computer-based training d. Game

a. Case study Rationale: Case studies can be used as a training method to help bridge the learning gap between theory and actual practice. The method builds on a variety of learner skills: analytical, critical, and interactive. Learners explore multiple solutions and enhance creativity and problem-solving approaches often using a discussion-based format. Reference: APIC Text, 4th edition, Chapter 3 - Education and Training CBIC Core Competency: Education and Research

An IP is writing instructional objectives for a learning module on catheter-associated bloodstream infections. He wants to focus on the cognitive level of "Analysis" from Bloom's taxonomy. Which of the following choices contains the verbs he should use to describe his learning objectives? a. Compare, discriminate, and differentiate b. Define, identify, and select c. Discuss, explain, and paraphrase d. Judge, summarize, and recommend

a. Compare, discriminate, and differentiate Rationale: Bloom's taxonomy was developed in the 1950s and is still used today to categorize ways of learning and thinking in a hierarchical structure. A revised model was developed in the 1990s to better fit educational practices of the 21st century. The "Analysis" level refers to the process of breaking material into constituent parts, determining how the parts relate to one another and to an overall structure or purpose through differentiating, organizing, and attributing. Applicable verbs include analyze, appraise, break down, calculate, categorize, classify, compare, contrast, criticize, derive, diagram, differentiate, discriminate, distinguish, examine, experiment, identify, illustrate, infer, interpret, model, outline, point out, question, relate, select, separate, subdivide, test. 1. Remembering - state, name, list 2. Understanding - interpret, compare, describe, provide example 3. Applying - Show, Illustrate, Classify 4. Analyzing - Explain, Investigate, Compare 5. Evaluating - Judge, Recommend, Decide 6. Creating - Plan, Design, Construct Reference: APIC Text, 4th edition, Chapter 3 - Education and Training CBIC Core Competency: Education and Research

The IP is helping develop a large-scale project to address SSIs. Which of the following tools will the IP utilize that helps secure expert judgment prior to launching the project? a. Delphi technique b. Peer review c. Fisher exact test d. Meta-analysis

a. Delphi technique Rationale: The Delphi technique is a structured communication method that solicits opinions from a panel of experts who answer questionnaires in two or more rounds. After each round, the responses are summarized and redistributed for discussion. The experts are encouraged to revise their earlier answers in light of the replies of other members of their panel. Common trends are identified, outliers are examined and a consensus is reached. Delphi is based on the principle that forecasts (or decisions) from a structured group of individuals are more accurate than those from unstructured groups. References: APIC Text, 4th edition, Chapter 5 - Infection Prevention and Behavioral Interventions; APIC Text, 4th edition, Chapter 3 - Education and Training CBIC Core Competency: Management and Communication

The OR notifies the IP that a patient is scheduled for a brain biopsy. After reviewing the results of diagnostic tests, possible diagnosis, and the reason for biopsy, the IP assesses the patient to be high risk for Creutzfeldt-Jakob disease (CJD). The next step is to: a. Determine the instruments to be used and the processing to be done b. Notify the staff to quarantine the patient after the procedure c. Advise the OR staff to disinfect the instruments in the OR d. Incinerate all equipment used to perform the biopsy

a. Determine the instruments to be used and the processing to be done Rationale: The OR, Sterile Processing Department, and the IP should meet to review the policy and recommendations for disinfecting and sterilization of the instruments as outlined by the World Health Organization (WHO) and the CDC. Reference: APIC Text, 4th edition, Chapter 73 - Creutzfeldt-Jakob Disease

Which of the following clinical uses would not be indicated for Gram stain results? a. Early identification of antibiotic resistance b. Initial direction for empiric antibiotic treatment c. Quality of the specimen d. Need for Isolation Precautions

a. Early identification of antibiotic resistance Rationale: Gram staining of a specimen may help to determine the quality of a specimen, initial direction for therapy (empiric antibiotics), or the need for Isolation Precautions (e.g., Gram-negative diplococci in CSF, suggesting meningococci). It does not identify antibiotic resistance. Reference: APIC Text, 4th edition, Chapter 24 - Microbiology Basics CBIC Core Competency: Identification of Infectious Disease Processes

The water temperature range recommended by the CDC for prevention of growth of Legionella bacteria is: a. Hot water at 124F and above, and cold water at 68F b. Hot water at 108F and above, and cold water at 77F c. Hot water at 110F and above, and cold water at 80F d. Hot water at 77F and above, and cold water at 50F

a. Hot water at 124F and above, and cold water at 68F

The Director of Infection Prevention and Control has been asked to participate in the organization's strategic planning. Which of the following might be a strategic goal for the Infection Prevention and Control program? a. Implement an electronic surveillance system in the next 3 years b. Fill the vacant IP position in the department within 45 days c. Participate more actively in the organization's Value Analysis Committee d. Share key HAI reports with senior managers every month

a. Implement an electronic surveillance system in the next 3 years Rationale: Strategic planning is an organization's process of defining its strategy, or direction, and making decisions on allocating its resources to pursue this strategy. The process includes setting goals, determining actions to achieve the goals, and mobilizing resources to execute the actions. Strategic goals are planned objectives that a department or organization strives to achieve. Answer A is an example of a clear measurable goal that is focused on the future and provides direction for the department. References: APIC Text, 4th edition, Chapter 1 - Infection Prevention and Control Programs; APIC Text, 4th edition, Chapter 16 - Quality Concepts; APIC Text, 4th edition, Chapter 6 - Healthcare Informatics and Information Technology

The viral load in a patient with HIV: a. Increases during the antiretroviral phase, decreases during asymptomatic HIV infection, then increases as the patient progresses to acquired immune deficiency syndrome (AIDS) b. Remains low during the antiretroviral phase, increases during asymptomatic HIV infection, then decreases as the patient progresses to AIDS c. Remains low during the antiretroviral phase, increases during asymptomatic HIV infection, then increases as the patient progresses to AIDS d. Increases during the antiretroviral phase, decreases during the asymptomatic HIV infection and continues to decrease as the patient progresses to AIDS

a. Increases during the antiretroviral phase, decreases during asymptomatic HIV infection, then increases as the patient progresses to acquired immune deficiency syndrome (AIDS) Rationale: During the acute retroviral syndrome there is a high level of HIV viremia with plasma HIV RNA titers of 105 copies per mL. Therefore, when a patient presents with signs and symptoms compatible with acute retroviral syndrome, the laboratory diagnosis is based on the determination of HIV RNA titers or viral load, not determination of HIV serology. Reference: APIC Text, 4th edition, Chapter 81 - HIV/AIDS CBIC Core Competency: Preventing/Controlling the Transmission of Infectious Agents

Which of the following is an advantage of orthophthalaldehyde (OPA) over glutaraldehyde? a. OPA does not require activation b. OPA is compatible with many more materials than glutaraldehyde c. It does not stain skin d. No personal protective equipment is needed to handle OPA

a. OPA does not require activation Rationale: There are several advantages of OPA over glutaraldehyde: 1. OPA requires no activation, 2. Is stable over a range of pH values, 3. Does not irritate the eyes and nose, 4. Does not have a strong odor. Both OPA and glutaraldehyde are compatible with many types of materials. Reference: CDC Guideline for Disinfection and Sterilization in Healthcare Facilities, 2008, Page 48-49 CBIC Core Competency: Environment of Care

Which would be the most objective method to determine the need for education about hand washing for a group of employees on a unit? a. Observational studies b. Personal interviews c. Focus group discussions d. Test development

a. Observational studies Rationale: Methods that can be used to determine the educational needs of the learner population include the following: • Learner self-assessment: The learner develops a self-achievement model and compares the current situation to the standard. • Focus group discussion: Learning needs are assessed in small groups with members assisting each other to clarify needs. • Interest-finder surveys: These are data-gathering tools, such as checklists or questionnaires. • Test development: Tests can be used as diagnostic tools to identify areas of learning deficiencies. • Personal interviews: The educator consults with random or selected individuals to determine learning needs. • Job analysis and performance reviews: These methods provide specific, precise information about work and performance. • Observational studies: Direct observation of personnel working can be performed by quality management analysts or IPs (e.g., a hand-washing study in critical care units). • Review of internal reports: Incident reports, occupational injury and illness reports, and performance improvement studies can be reviewed to determine the specific learning needs of healthcare providers Reference: APIC Text, 4th edition, Chapter 3 - Education and Training CBIC Core Competency: Education and Research

The IP is called to the day care center for a possible outbreak of Hepatitis A. The Public Health Nurse is assisting her in investigating the outbreak. Prophylactic administration of immunoglobulin to the day-care workers and noninfected children would be an example of: a. Passive immunity b. Active immunity c. Herd immunity d. Nonspecific immunity

a. Passive immunity Rationale: Hepatitis A, caused by infection with HAV, has an incubation period of approximately 28 days (range: 15 to 50 days). HAV replicates in the liver and is shed in high concentrations in feces from 2 weeks before to 1 week after the onset of clinical illness. HAV infection produces a self-limited disease that does not result in chronic infection or chronic liver disease. Current CDC guidelines recommend that persons who have recently been exposed to HAV and who have not been vaccinated previously be administered a single dose of single-antigen Hepatitis A vaccine or Immunoglobulin (IG) (0.02 mL/kg) as soon as possible, within 2 weeks after exposure. IG is an example of passive immunity, which is provided when a person is given antibodies to a disease rather than producing them through his or her own immune system. Reference: APIC Text, 4th edition, Chapter 103 - Immunization of Healthcare Personnel CBIC Core Competency: Surveillance and Epidemiologic Investigation

A biological indicator from one of the sterilizers in Sterile Processing turns positive. The first action should be: a. Retrieve unused items from the load b. Conduct surveillance of involved patients c. Evaluate the sterilizer d. Evaluate staff education of using the sterilizers

a. Retrieve unused items from the load Rationale: Recalling unused items would prevent further possibility of infection in patients and, therefore, should be done first. A written policy and procedure is needed to address who, when, and how to recall reprocessed items. Evidence of sterilization failures (e.g., positive biological indicators) is the most common reason for a recall. In addition, the policy may include compliance components of the Safe Medical Device Act if failure is noted in the reuse of reprocessed items. At a minimum, a log of items within the sterilized load should be reviewed, unused items in the load should be retrieved for reprocessing, functional evaluation of the involved sterilizer should be completed, and surveillance of involved patients should be initiated. Reference: APIC Text, 4th edition, Chapter 106 - Sterile Processing CBIC Core Competency: Cleaning, Sterilization, Disinfection, Asepsis

Unused alcohol swabs that are placed on top of a dialysis machine in an active dialysis unit: a. Should be discarded b. May be returned to the common area c. Must be cleaned before being used on another patient d. Should only be used on patients with that dialysis machine

a. Should be discarded Rationale: There has been transmission of Hepatitis B and Hepatitis C in dialysis settings, which necessitates rigorous infection control practices. Any medications or patient care supplies such as tape, alcohol swabs, and syringes that come into contact with a dialysis machine should be discarded. Other items should be cleaned if possible before being returned to a common area or used on another patient. If cleaning is not possible, then those items should also be discarded. Reference: APIC Text, 4th edition, Chapter 39 - Dialysis CBIC Core Competency: Cleaning, Sterilization, Disinfection, Asepsis

Which of the following recommendations should be made to reduce the risk of infection from sinks in patient care areas? a. Sink basins should be deep enough to prevent splashing of water onto nearby patient care items b. Sinks faucets should be located such that the flow of water hits the drain directly c. Sinks should be placed within 2 feet of the point of care to encourage frequent hand hygiene d. Aerators should be installed on faucets to minimize the amount of splash in the sink

a. Sink basins should be deep enough to prevent splashing of water onto nearby patient care items

There is a flu outbreak in a community, and the IP at the local hospital is interested in accurately identifying infected patients as quickly as possible so that they can be placed on Isolation Precautions. The IP has a choice between two rapid flu tests. Test A has a sensitivity of 98% and a specificity of 85%. Test B has a sensitivity of 92% and a specificity of 89%. Which test will be most accurate in correctly identifying patients who are infected with flu? a. Test A, because it has higher sensitivity b. Test A, because it has a higher predictive value negative c. Test B, because it has higher specificity d. Test B, because it has a higher predictive value negative

a. Test A, because it has higher sensitivity Rationale: Test A is the best choice. It will correctly identify 98 percent of people who have flu because it has 98 percent sensitivity. The specificity of Test A is 85 percent, so 15 percent of patients who do not have flu will be identified as positive (15 percent false positives [FPs]). In this case the IP is most interested in isolating infected patients, so the false-positive rate is an acceptable trade-off in this example. Reference: APIC Text, 4th edition, Chapter 13 - Use of Statistics in Infection Prevention CBIC Core Competency: Identification of Infectious Disease Processes

The IP is designing a new hand hygiene intervention for HCP based on the Health Belief Model. According to that model, which of the following is/are examples of "modifying factors" in the hand hygiene intervention? a. The ages and genders of the people for whom the intervention is being designed b. Signs posted in the unit that remind HCP to perform hand hygiene c. The perception of how much time it will take HCP to perform hand hygiene compared to the potential for hand hygiene to prevent infection in patients d. The belief among the target audience that they will be able to comply with hand hygiene 100 percent of the time

a. The ages and genders of the people for whom the intervention in being designed Rationale: The health belief model (HBM) is the oldest theory specifically developed to understand and predict health-associated behavior. An IP can use this model as a theoretical framework to motivate and influence infection prevention behaviors of HCP. The HBM includes key components of perceived susceptibility, perceived severity, perceived benefits, and perceived barriers. According to the HBM, HCP would adhere to hand hygiene if they believed that they were susceptible to infection if they neglected to wash their hands. Education on infection prevention measures may influence HCP's perceived risk of contracting and spreading infection. HCP may perceive severity by understanding the serious consequences of infection caused by poor hand hygiene compliance, such as prolonged hospital stay, expensive medical cost, and increased morbidity. HCP may perceive benefits regarding the effectiveness of hand hygiene practice when it comes to decreasing infection among patients and thereby decreasing HCP's heavy workload. Reference: APIC Text, 4th edition, Chapter 5 - Infection Prevention and Behavioral Interventions CBIC Core Competency: Education and Research

Failure mode and effects analysis (FMEA) is used to examine adverse events and identify what went wrong and what might prevent it from happening again. Which statement best describes the mode element of FMEA? a. The way of operating or using a system or process, or a way or manner in which a thing is done b. The results or consequences of an action c. The detailed examination of the elements or structure of something—perhaps a process, substance, or situation d. Lack of success, nonperformance, nonoccurrence, or breaking down or ceasing to function

a. The way of operating or using a system or process, or a way or manner in which a thing is done Rationale: The FMEA tool is a proactive, preventive approach to identify potential failures and opportunities for error. The mode is described as the way of operating or using a system or process, or a way or manner in which a thing is done. A mode is the way or manner in which something, such as a failure, can happen. Combining the words "failure" and "mode," a failure mode is the manner by which something can fail. A failure mode generally describes the way the failure occurs and its impact on a process. Any step in a process can fail, and each failure may have many failure mode. Reference: APIC Text, 4th edition, Chapter 18 - Patient Safety

An IP is reviewing the below data from a cohort study that examined the relationship between VRE colonization and long-term care. What was the incidence density of VRE colonization in this study? Participant #- Person/time observed - Colonized with VRE? #1 = 6 months Yes #2= 1 Year Yes #3= 3 months No #4= 1.5 years Yes #5= 9 months No a. Three cases/4 person-years b. Five cases/4 person-years c. Three cases/3 person-years d. Two cases/1 person-year

a. Three cases/4 person-years Rationale: Incidence density is the total number of new cases over the period of time that each participant was observed during the study period. In this table there are 3 new cases of VRE over the study period, so the numerator for the incidence density calculation is 3. Participants were observed for a total of four person-years (add up the number of months of observation of all study participants) so the denominator is 4 person-years. Reference: APIC Text, 4th edition, Chapter 13 - Use of Statistics in Infection Prevention CBIC Core Competency: Surveillance and Epidemiologic Investigation

The reservoir of microbes of pathogens present in potable water and its delivery network include: 1) Staphlococci 2) Pseudomonas spp. 3) Nontuberculous Mycobacteria 4) Arthrobacter spp. a. 1, 2 b. 2, 3 c. 1, 4 d. 2, 4

b. 2, 3 Rationale: Disruption of water utility systems during construction or renovation can disturb the biofilm present in water delivery pipes and pose a threat to patients. Microbes present in potable water and its delivery network include gram-negative bacteria, e.g., Legionellae and Pseudomonas spp, nontuberculous Mycobacteria, protozoa, and fungi. Reference: APIC Text, 4th edition, Chapter 117 - Construction and Renovation CBIC Core Competency: Environment of Care

According to the Centers for Disease Control and Prevention (CDC) and the Advisory Committee on Immunization Practices (ACIP), which of the following groups should receive the Hepatitis B vaccine? 1) Unvaccinated adults receiving chemotherapy 2) Residents and staff of facilities for developmentally delayed disabled persons 3) People with more than one sexual partner 4) Unvaccinated adults <60 years of age with diabetes mellitus a. 1, 2, 3 b. 2, 3, 4 c. 1, 3, 4 d. 1, 2, 4

b. 2, 3, 4 Rationale: Hepatitis B vaccination is the most effective measure to prevent HBV infection and its consequences, including cirrhosis of the liver, liver cancer, liver failure, and death. In adults, ongoing HBV transmission occurs primarily among unvaccinated persons with behavioral risks for HBV transmission, such as heterosexuals with multiple sex partners and men who have sex with men. Developmentally disabled persons in residential and nonresidential facilities also have had high rates of HBV infection, but the prevalence of infection has declined since the implementation of routine Hepatitis B vaccination in these settings. However, since Hepatitis B surface antigen-positive persons reside in such facilities, clients and staff continue to be at risk for infection. In response to multiple outbreaks of HBV among persons receiving assisted blood glucose monitoring, it is now recommended that all previously unvaccinated adults aged 19 through 59 years with diabetes mellitus (type 1 and type 2) be vaccinated against Hepatitis B as soon as possible after a diagnosis of diabetes is made. References: Centers for Disease Control and Prevention. Use of Hepatitis B Vaccination for Adults with Diabetes Mellitus: Recommendations of the Advisory Committee on Immunization Practices (ACIP). MMWR 2011 Dec 23; 60(50);1709-1711. Mast EE, Weinbaum CM, Fiore AE, et al. A Comprehensive Immunization Strategy to Eliminate Transmission of Hepatitis B Virus Infection in the United States: Recommendations of the Advisory Committee on Immunization Practices (ACIP) Part II: Immunization of Adults. MMWR Rec Rep 2006 Dec 8;55(RR-16) 1-33. CBIC Core Competency: Preventing/Controlling the Transmission of Infectious Agents

Which of the following is an accurate statement regarding the antibiogram for Staphylococcus isolates shown in the table? 1) 50 percent of Staphylococcus aureus isolates were resistant to Vancomycin 2) 12 percent of MRSA isolates were resistant to Trimeth/Sulfa 3) Clindamycin should not be used for coagulase-negative Staphylococcus infections in this facility because 100 percent of isolates were resistant to it 4) There were no cases of Vancomycin-resistant Staphylococcus aureus in this facility a. 2 b. 2, 4 c. 1, 2, 4 d. 1, 2, 3, 4

b. 2, 4 Rationale: An antibiogram provides the percentage of samples for a given organism that were sensitive to certain antibiotics and can be unit-specific or reflect hospital-wide isolates. There should be at least 30 diagnostic isolates included in an antibiogram, with only the first isolate from each patient included. The greater the number of isolates, the more accurate the sensitivity results for the given organism. Therefore the antibiogram might not accurately reflect all resistant microbes isolated in the facility. Reference: APIC Text, 4th edition, Chapter 26 - Antimicrobials and Resistance CBIC Core Competency: Identification of Infectious Disease Processes

When a test has a higher specificity than sensitivity, it means the test: a. Will be more accurate when predicting who is ill b. A negative result will be more accurate than a positive c. A positive result will be more accurate than a negative d. It should only be done as a secondary testing procedure to rule out disease

b. A negative result will be more accurate than a positive Rationale: Sensitivity (also called the true positive rate) measures the proportion of actual positives that are correctly identified as such (e.g., the percentage of sick people who are correctly identified as having the condition). Specificity (sometimes called the true negative rate) measures the proportion of negatives that are correctly identified as such (e.g., the percentage of healthy people who are correctly identified as not having the condition). Specificity = TN/(TN+FP) Therefore, a test with 100 percent specificity correctly identifies all patients without the disease. Reference: APIC Text, 4th edition, Chapter 13 - Use of Statistics in Infection Prevention CBIC Core Competency: Surveillance and Epidemiologic Investigation

Infection of short-term intravenous devices is associated with all of the following except: a. Percutaneous transmission during insertion b. Colonization of the catheter hub c. Extraluminal transmission d. Contamination of intravenous (IV) fluids

b. Colonization of the catheter hub Rationale: With short-term intravenous devices (IVDs) (in place fewer than 10 days), such as peripheral IV catheters, arterial catheters, and noncuffed, nontunneled CVCs, most device-associated bloodstream infections (BSIs) are of - Cutaneous origin, - Insertion site, - Gain access extraluminally, occasionally intraluminally. In contrast, contamination of the catheter hub and luminal fluid is the predominant mode of BSI with long-term IVDs (e.g., in place more than 10 days), such as cuffed Hickman- and Broviac-type catheters, subcutaneous central ports, and PICCs. Reference: APIC Text, 4th edition, Chapter 34 - Intravascular Device Infection CBIC Core Competency: Preventing/Controlling the Transmission of Infectious Agents

Based on current CDC guidelines, which of the following modes of transmission is most responsible for contamination of a central venous catheter? a. Airborne b. Direct contact c. Indirect contact d. Droplet spread

b. Direct contact Rationale: According to the CDC Guidelines for the Prevention of Intravascular Catheter-Related Infections, there are four recognized routes for contamination of catheters: 1) migration of skin organisms at the insertion site into the cutaneous catheter tract and along the surface of the catheter with colonization of the catheter tip; this is the most common route of infection for short-term catheters; 2) direct contamination of the catheter or catheter hub by contact with hands or contaminated fluids or devices; 3) less commonly, catheters might become hematogenously seeded from another focus of infection; and 4) rarely, infusate contamination might lead to catheter-related bloodstream infections. Reference: APIC Text, 4th edition, Chapter 34 - Intravascular Device Infections CBIC Core Competency: Preventing/Controlling the Transmission of Infectious Agents

A new EVS employee has been asked to clean up a large blood spill on the floor in the OR. How should he proceed? a. He should mix an EPA-registered disinfectant with water in a bucket and mop up the spill b. He should place absorbent material over the spill and pour the correct dilution of disinfectant over the material for the recommended contact time c. He should pour undiluted bleach directly on the blood spill, wait 10 minutes, and then mop up the spill d. He should place absorbent material over the spill, dispose of the material after absorption, and then mop the floor with an EPA-registered disinfectant

b. He should place absorbent material over the spill and pour the correct dilution of disinfectant over the material for the recommended contact time Rationale: Cleaning of a large blood spill may be a risk for acquisition of bloodborne pathogens, so immediate inactivation of any pathogens before cleaning is important to reduce this risk. Absorbent material may be placed over the spill to contain it and the correct dilution of an EPA-registered disinfectant can be poured on the absorbent material to reduce the bioburden. After this, the absorbent materials can be gathered up and disposed of and the area can be cleaned.

Which of the following has a low risk of transmission from sexual contact? a. Hepatitis B b. Hepatitis C c. HIV d. Syphilis

b. Hepatitis C Rationale: HCV is inefficiently transmitted by sexual intercourse (prevalence ranging from 1.3 percent in North America to 27 percent in Asia in longterm partners), though homosexual men, persons with multiple sexual partners and HIV patients have higher rates of seroprevalence for HCV than monogamous heterosexuals. The average risk for vertical transmission is 6 percent overall and 17percent in mothers with HIV, which appears to be related to viral titer. No difference in transmission is noted whether the child is breast- or bottle-fed. Reference: APIC Text, 4th edition, Chapter 97 - Viral Hepatitis CBIC Core Competency: Preventing/Controlling the Transmission of Infectious Agents

The antibiogram is usually prepared by: a. Infection Prevention and Control Department b. Laboratory c. Pharmacy d. Information Technology Department

b. Laboratory Rationale: Many hospital laboratories routinely perform antimicrobial susceptibility testing on bacterial pathogens. Cumulative susceptibility testing results are often organized into a summary table, or antibiogram, which may be used by clinicians, pharmacists, infection control personnel, and microbiologists as a reference guide to community or hospital-specific resistance patterns. Antibiograms lend information that can be used to raise awareness of resistance problems, support the use of optimal empiric therapy, and identify opportunities to reduce inappropriate antibiotic usage and to ascertain success of such efforts. Antibiograms are generally prepared by the laboratory according to the Clinical Laboratory Standards Institute guidelines. References: APIC Text, 4th edition, Chapter 26 - Antimicrobials and Resistance; Appendix D. In: Brooks K. Ready Reference for Microbes, 3rd edition. Washington, DC: Association for Professionals in Infection Control and Epidemiology, 2012.

An outbreak of Aspergillosis is suspected after several oncology patients are identified with positive cultures. The IP suspects a heating, ventilation, and air conditioning (HVAC) malfunction and begins an outbreak investigation. The IP has been asked to make recommendations for environmental cultures. All of the following need to be considered except: a. Identifying the purpose of culturing and appropriate methods beforehand b. Meeting with the facility legal staff to discuss notification of the patients c. Anticipating decisions and planned actions to results of culturing before undertaking the process d. Determining whether there are existing standards to interpret the results

b. Meeting with the facility legal staff to discuss notification of the patients

A hospital is beginning a major construction project. The IP has been asked to join the planning team to assist with the development of the ICRA for the project. What is the purpose of the ICRA? a. Develop and oversee the construction project schedule b. Minimize infectious hazards for patients and healthcare personnel c. Provide direction for level 3 and 4 projects only d. Coordinate systems start-ups

b. Minimize infectious hazards for patients and healthcare personnel An ICRA must guide a strategic, proactive design to mitigate environmental sources of microbes, prevent infectious hazards through architectural design (eg handwashing and hand hygiene stations, isolation rooms, materials selection for surfaces and furnishings), and provide control measures that mitigate potential contamination during actual construction or renovation (eg dust, barriers, pressure differentials, protection of air handlers)

Five cases of prosthetic valve endocarditis caused by Staphylococcus epidermidis are observed in one hospital. Of the following available methods, which is best for determining whether all five isolates were derived from a single source? a. Serotyping b. Pulsed-field gel electrophoresis c. Antimicrobial susceptibility testing d. Bacteriophage typing

b. Pulsed-field gel electrophoresis Rationale: Pulsed-field gel electrophoresis (PFGE) begins with the lysis of organisms and digestion of their chromosomal DNA with restriction enzymes. The fragments are separated into a pattern of discrete bands by switching the direction of the electrical current. This pattern serves as a "bar code" of the bacterial chromosome that can be used to assess the relatedness of different clinical isolates. This test method may be used with any organism from which chromosomal DNA can be properly isolated; it has been used with a wide variety of bacterial pathogens to assess epidemiological interrelationships. PFGE is probably the most widely used method for molecular epidemiology and is generally considered to be the gold standard for most clinically important organisms. Reference: APIC Text, 4th edition, Chapter 24 - Microbiology Basics CBIC Core Competency: Identification of Infectious Disease Processes

A patient is admitted to rule out TB. All of the following would be noted as a risk factor for transmission of infection to the staff except: a. Improper N95 respirator use by the staff b. Surgical mask placed on the patient during transport c. Acid-fast bacilli (AFB) smear positive sputum culture from the patient d. Patient placed in a regular exam room in the emergency room

b. Surgical mask placed on the patient during transport Rationale: The risk of infection transmission is related to the number of organisms that are aerosolized. Increased risk of infection is associated with patients with: - cavitary disease, - smear-positive sputum, - or a cough. - if patients are placed in poorly or improperly ventilated rooms. The emergency room (ER) and clinic areas should have plans for appropriately isolating patients with suspected TB who are being seen as outpatients. These plans should include details on placing the patient in a separate area from other patients and placing a surgical mask on him or her until appropriate isolation can be arranged. Reference: APIC Text, 4th edition, Chapter 95 - Tuberculosis and Other Mycobacteria CBIC Core Competency: Preventing/Controlling the Transmission of Infectious Agents

The IP hears that one of the nursing units in his facility will soon be renovated. After verifying this information, he contacts the newly hired Director of Design and Construction to explain how important it is for the IP to be included in the planning and design of all renovation projects. Which of the following is the most compelling argument? a. The IP supplies necessary maintenance for critical utility systems that delver ventilation and water to patient care areas b. The IP provides essential input into preventing hazardous risks to patients, HCP, and visitors during design and construction projects c. The IP will ensure compliance with various compliance-, regulatory standard-, and guideline-setting agencies d. The IP is responsible for facilitating the transport and approval for disposal of waste materials

b. The IP provides essential input into preventing hazardous risks to patients, HCP, and visitors during design and construction projects

There is a shortage of influenza vaccine, and the IP has been asked to help prioritize the administration of the vaccine among HCP in the facility. Which of the following employee groups would be among the highest priority for immunization? a. The admissions clerk in the ED b. The Lung Transplant Coordinator c. A nurse in labor and delivery d. The CEO of the hospital

b. The Lung Transplant Coordinator Rationale: Of all HCP listed, this employee would get priority because they work postoperatively with immunocompromised patients who are at high risk for influenza infection. Any HCP who works with high-risk patients should be vaccinated against influenza. Reference: APIC Text, 4th edition, Chapter 103 - Immunization of Healthcare Personnel CBIC Core Competency: Employee/Occupational Health

There was contamination of a healthcare facility's potable water system with sewer water and there is high concern for the presence of Cryptosporidium. Which of the following should you recommend for corrective decontamination of the system? a. Use chlorination to disinfect the system b. Use high-temperature flushing to disinfect the system c. Use fluoridation to flush the system d. Use ultraviolet light to flush the system

b. Use high-temperature flushing to disinfect the system Rationale: If there is contamination of the potable water system with sewer water in a facility then it is necessary to flush the system. A variety of methods can be used to accomplish this and they may be used in combination, if necessary. As Cryptosporidium is highly resistant to chlorine and there is suspected contamination with this protozoa in this scenario, you would have to minimally recommend the use of high-temperature flushing to disinfect the system. Reference: CDC Guidelines for Environmental Infection Control in Health-Care Facilities, 2003 CBIC Core Competency: Environment of Care

A patient is admitted with fever, rash, headache, abdominal pain, vomiting, and muscle pain. The IP is aware that Rocky Mountain spotted fever is endemic in the area. The patient reports a recent camping trip with exposure to ticks. Which of the laboratory tests listed below should be ordered to detect and differentiate the appropriate antibodies in the serum? a. Blood culture b. Weil-Felix agglutination c. Sedimentation rate d. Cold agglutinin

b. Weil-Felix agglutination Rationale: Rocky Mountain spotted fever (RMSF) is a tickborne disease caused by the bacterium Rickettsia rickettsii. This organism is a cause of potentially fatal human illness in North and South America and is transmitted to humans by the bite of infected tick species. Weil-Felix agglutination is a test performed to differentiate rickettsial antibodies in the serum. This test can be useful in diagnosing RMSF. Reference: APIC Text, 4th edition, Chapter 25 - Laboratory Testing and Diagnostics CBIC Core Competency: Identification of Infectious Disease Processes

Positive Deviance

behavior that over conforms to social expectations Working from the bottom up solution the caregivers have the answer to solve the problem very sustainable approach to work with the front line caregivers to solve problems. Example: working with front line caregivers on where to place the HH stations that best suits them. The community must own the entire process. 4 steps: 1. Define 2. Determine 3. Discover 4. Design

Which of the following organisms are most likely to be the cause of outbreaks of healthcare-associated infections due to improper reprocessing of bronchoscopes? 1) Mycobacterium tuberculosis 2) Carbapenem-resistant Enterobacteriaceae 3) Pseudomonas aeruginosa 4) Staphylococcus aureus a. 1, 2, 3, 4 b. 1, 3, 4 c. 1, 3 d. 2, 4

c. 1, 3 Rationale: Improperly processed bronchoscopes have been associated with outbreaks of infections in healthcare settings. These incidents have been due to inadequate cleaning of the scope, incorrect selection of disinfecting agents, and lapses in following the manufacturer's protocol for cleaning. The organisms that have been most highly associated with improperly processed bronchoscopes are Mycobacterium tuberculosis and Pseudomonas aeruginosa. Reference: CDC Guideline for Disinfection and Sterilization in Healthcare Facilities, 2008, Page 15 CBIC Core Competency: Cleaning, Sterilization, Disinfection, Asepsis

Which of the following statements is true regarding an asymptomatic employee with a newly positive tuberculin skin test (TST) of 10 mm induration in a medium to high risk setting? 1) The employee has latent tuberculosis (TB) infection 2) The employee is capable of transmitting TB to others 3) The employee is not infectious 4) The employee has TB disease a. 3, 4 b. 2, 4 c. 1, 3 d. 1, 2

c. 1, 3 Rationale: Latent tuberculosis infection (LTBI) is the presence of M. tuberculosis organisms (tubercle bacilli) without symptoms or radiographic or bacteriologic evidence of TB. Approximately 90 to 95 percent of those infected are able to mount an immune response that halts the progression from LTBI to TB. Persons with LTBI are asymptomatic (they have no symptoms of TB) and are not infectious. Reference: Jensen PA, Lambert LA, Iademarco MF, et al. Guidelines for Preventing the Transmission of Mycobacterium tuberculosis in Health-Care Settings, 2005. MMWR 2005 December 30. 41(RR-17). 1-141. Available at: http://www.cdc.gov/mmwr/preview/mmwrhtml/rr5417a1.htm?s_cid=rr5417a1_e. CBIC Core Competency: Identification of Infectious Disease Processes

Which of the following statements are true regarding consent to immunization? 1) Some states allow personal belief exemptions 2) Federal regulations require Informed consent 3) All states allow medical exemptions for persons with medical contraindications to vaccination 4) Vaccine recipients must receive Vaccine Information Statements (VISs) a. 1, 2, 3 b. 2, 3, 4 c. 1, 3, 4 d. 1, 2, 4

c. 1, 3, 4 Rationale: HCP are required by the National Childhood Vaccine Injury Act (NCVIA) to provide a copy of the Vaccine Information Statement to either the adult recipient or to the child's parent/legal representative. All states allow medical exemptions for persons who have medical contraindication to vaccination. Most states allow religious exemptions and some allow philosophical/personal belief exemptions. There is no federal requirement for informed consent relating to immunization. Reference: APIC Text, 4th edition, Chapter 100 - Occupational Health CBIC Core Competency: Employee/Occupational Health

Green cleaning is a new approach to environmental cleaning that aims at reducing harm to human health and the environment while maintaining or improving the hygiene of the healthcare environment. Which of the following green strategies would be appropriate to implement? 1) Replacing floor strippers and finishes that contain heavy metals and asthmagens 2) Substituting disinfectant room cleaners with less toxic detergents 3) Employing carcinogen-free carpet cleaners 4) Introducing high-filtration vacuum cleaners a. 1, 2, 4 b. 2, 3, 4 c. 1, 3, 4 d. 1, 2, 4

c. 1, 3, 4 Rationale: The effective use of disinfectants is part of a multibarrier strategy to prevent healthcare-associated infections. CDC Guidelines recommend that noncritical medical equipment surfaces should be disinfected with an EPA-registered low- or intermediate-level disinfectant. Current certification programs for green cleaning products and equipment do not cover EPA-registered disinfectants. Reference: APIC Text, 4th edition, Chapter 107- Environmental Services CBIC Core Competency: Environment of Care

The CDC lists 18 drug-resistant threats to the United States. These threats are categorized based on level of concern: urgent, serious, and concerning. Which of the following belong to the "Urgent" threat category? 1) Neisseria gonorrhea 2) Vancomycin-resistant Staphylococcus aureus 3) Drug-resistant Tuberculosis 4) Clostridium difficile a. 1, 2 b. 2, 3 c. 1, 4 d. 3, 4

c. 1, 4 Rationale: Infections classified as urgent threats include: - Carbapenem-resistant Enterobacteriaceae (CRE), - Drug-resistant Neisseria gonorrhea, - Clostridium difficile These are high-consequence threats due to the significant risks identified across several criteria. These threats have the potential to become widespread and require urgent public health attention to identify infections and limit transmission. Reference: CDC, Antibiotic Resistance Threats in the United States, 2013. http://www.cdc.gov/drugresistance/threat-report-2013 CBIC Core Competency: Identification of Infectious Disease Processes

Poor planning during a construction project can lead to an increase risk of infection related to: 1) Construction delays 2) Compromised air quality 3) Contaminated water 4) Increase in construction-related traffic a. 1, 3 b. 2, 4 c. 2, 3 d. 1, 4

c. 2, 3 Rationale: Insufficient planning can lead to compromised air quality and potential for continued environmental contamination from fungi (e.g., Aspergillus spp.) or water contaminated with water-associated microorganisms (e.g., Legionella spp.) during construction or renovation. Reference: APIC Text, 4th edition, Chapter 116 - Construction and Renovation CBIC Core Competency: Environment of Care

Which short-term central lines are associated with a higher risk of infection? 1) Peripherally-inserted central venous catheters (PICC) 2) Femoral lines 3) Subclavian lines 4) Jugular lines a. 1, 2 b. 2, 3 c. 2, 4 d. 1, 4

c. 2, 4 Rationale: The CDC recommends using the subclavian site rather than a jugular or a femoral site in adult patients to minimize infection risk for nontunneled central venous catheter placement. References: O'Grady NP, Alexander M, Burns LA, et al. Guidelines for the Prevention of Intravascular Catheter-Related Infections, 2011. Centers for Disease Control and Prevention website. 2011. Available at: http://www.cdc.gov/hicpac/pdf/guidelines/bsi-guidelines-2011.pdf; APIC Text, 4th edition, Chapter 34 - Intravascular Device Infections CBIC Core Competency: Preventing/Controlling the Transmission of Infectious Agents

Poor planning during a construction project can lead to an increased risk of infection related to: 1. Construction delays 2. Compromised air quality 3. Contaminated water 4. Increase in construction-related traffic a. 3 b. 2,4 c. 2,3 d. 1,4

c. 2,3 Insufficient planning can lead to compromised air quality and the potential for continued environmental contamination

The planning committee for your new healthcare facility would like to include a water feature just inside the entrance of the new building. Which of the following recommendations should you make to ensure that this does not put patients, staff, and visitors at risk of infection: 1. The water feature must include underwater lighting to allow personnel to visualize whether it is being properly maintained 2. The water feature should be designed so that water does not remain stagnant in any part of the feature, and should include built-in methods of filtering or disinfecting the water 3. The water feature should be designed to prevent aerosolizing of water 4. The water feature must have routine maintenance a. 2 b. 2,4 c. 2,3,4 d. 1,2,3,4

c. 2,3,4

Which of the following are the key to preventing water related problems in healthcare facilities? 1. Planning for remediation in the event of a disaster 2. Proactive risk assessment 3. Thoughtful design 4. Routine preventative maintenance a. 1,3,4 b. 1,2,3,4 c. 2,3,4 d. 2,4

c. 2,3,4 Although planning must necessarily provide for remediation in the event of a disaster - natural or manmade - the key to prevention is proactive risk assessment, thoughtful design, and routine preventative maintenance

Diluted bleach solutions stored in an open container must be changed every: a. 30 days b. 28 days c. 24 hours d. 12 hours

c. 24 hours

Under what circumstances should human immunodeficiency virus (HIV)-infected HCP be placed under work restrictions? 1) When viral burden measurements of <5 × 102 GE/mL for HIV infection and techniques for eliminating transmission risks cannot be identified 2) No work restrictions necessary if Standard Precautions are followed 3) When required by state or regional regulations 4) Whenever performing exposure-prone, noninvasive procedures 5) After counsel from an expert review panel is sought a. 2, 5 b. 1, 3 c. 3, 5 d. 4, 5

c. 3, 5 Rationale: The CDC recommends that exposure-prone invasive procedures should not be performed by HCP with HIV until counsel from an expert review panel is sought. Risks during noninvasive procedures would be minimized with the use of Standard Precautions, but this may not be true for high-risk invasive procedures. An expert review panel can make these determinations and recommendations. State or regional recommendations must be followed. In addition to CDC, the Society for Healthcare Epidemiology of America and the Association for Professionals in Infection Control and Epidemiology provide guidance on this issue. Reference: APIC Text, 4th edition, Chapter 100 - Occupational Health CBIC Core Competency: Employee/Occupational Health

The lead IP is invited to participate in an employee health process improvement project. The goal of the project is to improve influenza vaccination rates among employees. A multidisciplinary team is formed with representation from front-line employees. After conducting a root cause analysis (RCA), the team develops a proposal for improvement to present to the facility leadership team. Which of the following elements should be included as part of the proposal to most clearly communicate to the leadership team the factors contributing to the facilities' current vaccination rate among employees? a. A timeline for implementing improvement activities b. A Pareto chart c. A fishbone diagram with an explanation d. A strategic plan

c. A fishbone diagram with an explanation Rationale: The RCA process takes a retrospective look at adverse outcomes and determines what happened, why it happened, and what an organization can do to prevent the situation from recurring in the future. The product of the RCA is an action plan that identifies the strategies that the organization intends to implement to improve safety. A thorough RCA demonstrates the credibility of the recommended process to the facility leadership team. When used during the RCA process, a fishbone diagram (also called an Ishikawa diagram) will help identify and visually display both the elements involved in the improvement project and the areas of responsibility and accountability (see Figure PE3-3). Used in conjunction with the RCA, the fishbone diagram with an accompanying explanation presents a clear picture of the both improvement project and the rationale behind it.

A number of research studies have examined the relationship between nurse staffing levels and the risk of HAIs in the hospital setting. Which of the following statements has not been supported by the literature? a. Patients in an intensive care unit (ICU) with lower levels of nurse staffing had an increased risk for ventilator-associated pneumonia b. The use of nonpermanent staff significantly increases a patient's infection risk c. A specific evidence-based nurse staffing level benchmark has been determined that is associated with decreased risk for HAI d. There is a relationship between adequate numbers of direct care providers (nurses) and the likelihood that CDC guidelines will be followed

c. A specific evidence-based nurse staffing level benchmark has been determined that is associated with decreased risk for HAI Rationale: Hospitals with low nurse staffing levels tend to have higher rates of poor patient outcomes such as pneumonia, shock, cardiac arrest, and urinary tract infections. Furthermore, a number of researchers have found the level and/or the use of nonpermanent staff also significantly increases a patient's infection risk. Despite these data, determination of a specific evidence-based nurse staffing level benchmark that is associated with decreased risk for HAI has not been determined. Reference: APIC Text, 4th edition, Chapter 9 - Staffing

There has been a major bioterrorism event in the area, as is evidenced by a large number of patients being admitted to local healthcare facilities with fever >101.1° F and respiratory symptoms that are progressing rapidly. Health officials are currently unaware of the causative organism. What type of Isolation Precautions are warranted in this situation? a. Standard Precautions b. Droplet Precautions c. Airborne Precautions d. Contact and Airborne Precautions

c. Airborne Precautions Rationale: During the beginning of an infectious disease disaster when the agent may not have been identified or when there is not enough evidence regarding the disease transmission route, IPs need to base infection prevention decisions on syndromes and symptomatology. This is referred to as syndrome-based isolation/control measures. General guidelines include implementing Airborne Precautions if the patient is severely ill with rapidly progressing respiratory symptoms and airborne spread disease is suspected (i.e., severe acute respiratory syndrome or avian influenza). Reference: APIC Text, 4th edition, Chapter 120 - Infectious Disease Disasters: Bioterrorism, Emerging Infections, and Pandemics CBIC Core Competency: Preventing/Controlling the Transmission

What key strategy should the IP use to meet the administrative challenges of outpatient settings that are geographically dispersed? a. Standardized measurement and definitions for HAIs consistent with those used for inpatient settings b. HAI comparison with external benchmarks for all sites c. Development of communication networks between sites, hospitals, and health departments d. Standardized infection prevention policies and procedures

c. Development of communication networks between sites, hospitals, and health departments Rationale: The establishment of communication networks and collaboration at all levels of the outpatient sites is essential to implement infection prevention practices more effectively and efficiently. For outpatient sites, the use of measurement and operational definitions for HAIs can be challenging due to the lack of standardized definitions. There are limited external benchmarks for HAIs in ambulatory care, and HAI rates are often internally compared over time to establish a benchmark. Infection prevention policies and procedures should be specific to each setting. Reference: APIC Text, 4th edition, Chapter 48 - Ambulatory Care CBIC Core Competency: Management and Communication

The Hemodialysis Department at a hospital completes a monthly water culture testing. The results are over the limit for colony-forming units (CFU)/mL. Which of the following actions should be done first? a. Notify the physician b. Initiate daily water culture testing c. Disinfect the system d. Notify the Infection Control Department

c. Disinfect the system Rationale: According to the Conditions for Coverage document from Centers for Medicare & Medicaid Services (CMS), product water used to prepare dialysate or concentrates from powder at a dialysis facility, or to process dialyzers for reuse, shall contain a total viable microbial count lower than 200 CFU/mL and an endotoxin concentration lower than 2 EU/mL. Measures must be performed promptly when results exceed the action level or the maximum allowable level. Dialysis may continue when bacteria/endotoxin is found to be at the action level, but retesting and/or disinfection of the system should be performed promptly. Promptly has been defined by CMS regulation as within 48 hours of receiving the report. Reference: APIC Text, 4th edition, Chapter 39 - Dialysis CBIC Core Competency: Preventing/Controlling the Transmission of Infectious Agents

The IP is consulted prior to the construction of a new unit in her hospital. The design that is most concerning to her is: a. AII rooms (AIIR) that have negative air pressure b. Protective environment rooms that have positive air pressure c. Dual-purpose rooms that can alternate between negative and positive air pressure d. All rooms with neutral air pressure

c. Dual-purpose rooms that can alternate between negative and positive air pressure Rationale: Isolation rooms can serve two purposes. The first is to provide appropriate isolation for patients infected with pathogens that are transmitted by the airborne route (e.g., M. tuberculosis, varicella-zoster virus, rubeola [measles] virus). The major goal in this situation is to prevent the transmission of pathogens from an infected patient to other patients, staff, or visitors. This is generally achieved by maintaining AIIRs. The second purpose is to provide a PE for severely immunosuppressed patients. AIIRs have negative air pressure, and PE rooms have positive air pressure with respect to adjacent areas. Correct direction of airflow and properly balanced air pressure in AIIR or PE areas are essential elements to consider during construction and renovation. The Facility Guidelines Institute's (FGI) Guidelines do not support "reversible" airflow rooms based on the complexity of pressure relationships, concerns for serious patient and HCP outcomes if errors are made, and labor intensity needed for preventive maintenance. Reference: APIC Text, 4th edition, Chapter 116 - Construction and Renovation CBIC Core Competency: Environment of Care

Which of the following statements best describes human factors? a. Ability to identify the many and various factors that impact upon a complex situation or event b. Prevention of errors and adverse effects to patients associated with healthcare use c. Environmental, organizational, and job elements and human and individual characteristics that influence behavior at work in a way that can affect health and safety d. The attitudes, beliefs, perceptions, and vales that employees share in relation to safety

c. Environmental, organizational, and job elements and human and individual characteristics that influence behavior at work in a way that can affect health and safety Rationale: Human factors examine the relationship between people, the tools and equipment they use in the workplace, and the systems with which they interact. The goal of human factors is to minimize errors by focusing on improving efficiency, creativity, productivity, and job satisfaction. The application of human factors knowledge to healthcare can help design processes to improve patient safety. Reference: APIC Text, 4th edition, Chapter 18 - Patient Safety

Which of the following statements is correct regarding the efficacy of a disinfectant product on a microbe? a. For all disinfectants, the greater the concentration of the disinfectant, the shorter the contact time is for effective cidal activity b. For all disinfectants, the activity of the disinfectant increases as temperature increases c. For all disinfectants, the greater the number of microbes on an object or surface, the greater the amount of time that's needed for a disinfectant to have effective cidal activity d. For all disinfectants, the existence of a biofilm does not affect cidal activity

c. For all disinfectants, the greater the number of microbes on an object or surface, the greater the amount of time that's needed for a disinfectant to have effective cidal activity Rationale: Without exception, a higher bioburden of organisms on an object or surface will require higher contact time for a disinfectant to kill the organisms. Prior cleaning of a surface or instrument will reduce the bioburden, which will decrease the necessary contact time for the disinfection process. Reference: CDC Guideline for Disinfection and Sterilization in Healthcare Facilities, 2008, Page: 33-35 CBIC Core Competency: Environment of Care

A newly hired Infection Prevention Manager is addressing program deficits that occurred during the months the facility sought to fill the vacant position. The new manager must focus on many specific asks, including working closely with others to clarify roles and responsibilities. This type of management approach is known as: a. Charismatic b. Situational c. Functional d. Motivational

c. Functional Rationale: Functional management is the most common type of organizational management. A functional manager is a person who has management authority over an organizational unit—such as a department— within a business, company, or other organization. Functional managers have ongoing responsibilities and are not usually directly affiliated with project teams, other than ensuring that goals and objectives are aligned with the organization's overall strategy and vision. References: APIC Text, 4th edition, Chapter 1 - Infection Prevention and Control Programs; APIC Text, 4th edition, Chapter 2 - Competency and Certification of the Infection Preventionist

The IP is selecting a process measure in order to monitor and evaluate the quality of care. An example of a process measure is: a. SSI after a hip replacement b. Conjunctivitis c. Hepatitis B immunity rates d. Tuberculin skin test (TST) conversions

c. Hepatitis B immunity rates Rationale: Process measures are commonly used to evaluate compliance with desired care or support practices or to monitor variation in these practices. They may also be used when the outcome to be measured is rare or difficult to measure (e.g., infections after endoscopy) or when there is difficulty acquiring the data (e.g., contacting the discharged and relocated patient after surgery). Process measures are also helpful in evaluating the effectiveness of an educational effort as a measure of behavior (e.g., compliance with aseptic technique for dressing change) or performance of basic infection prevention procedures, such as hand hygiene. Hepatitis B immunity rates are an example of a process indicator. The other answers are examples of outcome indicators. Reference: APIC Text, 4th edition, Chapter 11 - Surveillance CBIC Core Competency: Surveillance and Epidemiologic Investigation

A patient is concerned that there might be microbes in the facility that are resistant to the environmental disinfection products in use. How do you respond to this concern? a. Inform her that while microbes can develop resistance to antimicrobials, they cannot become resistant to disinfectants b. Assure her that the disinfection products in use are ones that microbes have not developed resistance to yet c. Inform her that while reduced susceptibility of microbes to disinfectants can occur, the antimicrobial level of disinfectants used is still sufficient to inactivate those microbes d. Assure her that your facility uses special environmental disinfection procedures for all antimicrobial resistant organisms

c. Inform her that while reduced susceptibility of microbes to disinfectants can occur, the antimicrobial level of disinfectants used is still sufficient to inactivate those microbes Rationale: As with resistance to antibiotics, microbes can develop altered sensitivity to environmental disinfectant products though spontaneous mutation in the bacterial genome, transmission of transposable resistance genes to the chromosome from a plasmid and vice versa, and transfer of resistance genes on plasmids between microbes. However, decreased sensitivity or increased tolerance to environmental disinfectant products does not render the microbes resistant to these products and the concentrations of product used in the healthcare facility remain cidal for even less sensitive organisms. Reference: CDC Guideline for Disinfection and Sterilization in Healthcare Facilities, 2008, Page 28 CBIC Core Competency: Environment of Care

patient is admitted to the hospital with severe communityacquired pneumonia (CAP). His symptoms include hemoptysis and a multilobar infiltrate seen on chest radiograph. The patient is recovering from a recent influenza virus infection. What organism should be suspected? a. Moraxella catarrhalis b. Streptococcus pneumoniae c. MRSA d. Haemophilus influenzae

c. MRSA Rationale: Community-associated MRSA (CA-MRSA) strains are primarily associated with skin and soft tissue infections; however, they are increasingly causing more invasive infections, including severe CAP. CA-MRSA should be suspected in patients with severe CAP plus hemoptysis, multilobar or cavitary infiltrate seen on chest radiograph, or neutropenia. Reference: APIC Text, 4th edition, Chapter 36 - Pneumonia CBIC Core Competency: Identification of Infectious Disease Processes

What recommendation should an IP give regarding mask use by a staff member who has documented immunity to varicella (chickenpox) while caring for a patient who is on Airborne Precautions for varicella (chickenpox)? a. Wear an N95 respirator whenever entering the room b. Wear a regular surgical mask whenever entering the room c. No mask is needed d. The patient should don a surgical mask

c. No mask is needed Rationale: According to the HICPAC 2007 Guideline for Isolation Precautions, there are no recommendations for HCP who are immune to measles and chickenpox (varicella) to wear respiratory PPE. There are also no recommendations for susceptible HCP to wear a surgical mask versus an N95 respirator when caring for patients with measles or chickenpox (varicella). References: APIC Text, 4th edition, Chapter 29 - Isolation Precautions (Transmission-based Precautions); Siegel J, Rhinehart E, Jackson M, et al., and the Healthcare Infection Control Practices Advisory Committee (HICPAC). 2007 Guideline for Isolation Precautions: Preventing Transmission of Infectious Agents in Health Care Settings. Am J Infect Control 2007;35(10 Suppl 2): S65- S164. Available at: www.cdc.gov/infectioncontrol/guidelines/isolation/index.html CBIC Core Competency: Preventing/Controlling the Transmission of Infectious Agents

On July 7, a 30-year-old male is admitted to the medical ICU with a 2-day history of acute gastroenteritis symptoms. The IP suspects the patient is infected with norovirus. Which of the following statements regarding the epidemiology of norovirus infection supports this? a. Most norovirus outbreaks are caused by genotype GII.2. b. Severe cases of norovirus are most common in adults aged 25-45. c. Noroviruses are the most common cause of epidemic gastroenteritis worldwide. d. Norovirus outbreaks occur most commonly in the summer months.

c. Noroviruses are the most common cause of epidemic gastroenteritis worldwide. Noroviruses are the most common cause of gastroenteritis worldwide and the leading cause of foodborne outbreaks in the United States. 80% of reported outbreaks occur during November - April Most NoV outbreaks are attributed to genotype GII.4. Severe disease associated with NoV occurs most frequently among older adults, young children, and immunocompromised patients.

The greatest concern in an emergency situation involving a patient with nontraditional body piercing is: a. Bleeding from areas of high vascularity b. Hematogeneous spread of bacteria to other sites c. Not being familiar with the opening mechanism of the jewelry to be able to remove it d. Causing distress to the patient

c. Not being familiar with the opening mechanism of the jewelry to be able to remove it Rationale: It is becoming more common for HCP to provide care for patients with body jewelry, and it is sometimes unclear whether removal of the jewelry is necessary. Removal of these items is sometimes required for radiological purposes and removal is not usually difficult if the patient is able to assist with the opening mechanism. In an emergency situation, attempts at removal may cause unnecessary trauma to the site. Many HCP are unaware of the procedures for removing body piercing. In a survey of 28 accident and emergency doctors, only six were able to accurately describe the opening mechanisms of all three commonly used types of jewelry. Reference: APIC Text, 4th edition, Chapter 123 - Body Piercing, Tattoos, and Electrolysis CBIC Core Competency: Preventing/Controlling the Transmission of Infectious Agents

A patient is admitted with measles and placed on Airborne Isolation. How many days after symptom onset would the characteristic blotchy red rash appear? a. On days 1-2 b. On days 7-10 c. On days 3-7 d. On days 21-25

c. On days 3-7 Rationale: Measles symptoms generally appear in two stages: Stage 1: In the first stage, which lasts 2 to 4 days, the individual may have a - runny nose, - cough, and - a slight fever. while the fever gradually rises each day, often peaking as high as 103° to 105°F. - The eyes may become reddened and sensitive to light, - Koplik spots (small bluish-white spots surrounded by a reddish area) may also appear on the gums and inside of the cheeks. Stage 2: The second stage begins on the 3rd-7th day and consists of - a red blotchy rash lasting 5 to 6 days. The rash usually begins on the face and then spreads downward and outward, reaching the hands and feet. The rash fades in the same order that it appeared, from head to extremities. Reference: Measles. In: Heymann D. Control of Communicable Diseases Manual, 19th edition. Washington, DC: American Public Health Association, 2008. CBIC Core Competency: Identification of Infectious Disease Processes

This process destroys all forms of microbial life: a. High-level disinfection b. Cleaning c. Sterilization d. Antisepsis

c. Sterilization Rationale: Sterilization is the term for the process where all microbial life including spores is destroyed. Sterilization may be carried out using steam, hydrogen peroxide gas, ethylene oxide gas, and liquid sterilants. Reference: CDC Guideline for Disinfection and Sterilization in Healthcare Facilities, 2008, Page 8-9 CBIC Core Competency: Cleaning, Sterilization, Disinfection, Asepsis

Breaches in patient safety are being studied to determine the cause and effect of human error. Human factors engineering is a tool that: a. Studies processes to achieve "failure-free" operation over time to reduce defects and improve system safety b. Studies people at work, then designs tasks and the working environment so that people can be safe, effective, and productive c. Studies human characteristics and is concerned with design of tools, machines, and systems that take into account human capabilities d. Studies the elements involved with human-machine interface to improve working conditions

c. Studies human characteristics and is concerned with design of tools, machines, and systems that take into account human capabilities Rationale: Human factors engineering (HFE) involves research in human psychological, social, physical, and biological characteristics and is concerned with design of tools, machines, and systems that take into account human capabilities, limitations, and characteristics. The goal is to create designs that are safe, comfortable, and effective for humans to use. Reference: APIC Text, 4th edition, Chapter 18 - Patient Safety CBIC Core Competency: Management and Communication

A used endoscope was placed in a carrying case before being cleaned and reprocessed. Which of the following actions should be taken? a. The endoscope should be cleaned and reprocessed and placed back in the carrying case b. The carrying case should be retained and only utilized for endoscopes that have not yet been cleaned or reprocessed c. The carrying case should be discarded d. The carrying case should be retained and only used for endoscopes that have been wrapped after cleaning and disinfection

c. The carrying case should be discarded Rationale: Of all medical instruments, endoscopes have the highest association with healthcare outbreaks. Endoscope carrying cases should not be used to store clean or dirty endoscopes within a healthcare facility. If an unprocessed endoscope is placed in a carrying case then the case should be discarded because of the potential for a clean endoscope to become contaminated through contact with the case. Reference: CDC Guideline for Disinfection and Sterilization in Healthcare Facilities, 2008 Page: 17, 88 CBIC Core Competency: Cleaning, Sterilization, Disinfection, Asepsis

Infection prevention challenges related to body piercings and tattooing include all of the following except: a. There have not been uniform licensure requirements for body art studios b. Reporting of infections to the Public Health Department is limited to bloodborne diseases c. The popularity of tattooing and body piercing has made it more socially acceptable d. Inspectors may not receive adequate training to conduct site inspections

c. The popularity of tattooing and body piercing has made it more socially acceptable Rationale: Tattooing and body piercing activities can transmit infectious diseases. It is difficult to describe the epidemiology of disease transmission after tattooing and body piercing procedures. There have not been uniform licensure requirements for body art studios or for individuals performing the procedures, so the reporting of infections to a public health agency is generally limited to bloodborne diseases identified by the healthcare system. The licensure varies greatly by state. Regulations may address the age of the client, client histories, sterilization processes, single-use items, apprenticeships/training, and bloodborne pathogens training. Some states leave the regulations up to local ordinances and some have no regulations whatsoever. The enforcement of the various regulations is dependent on the funding of the programs, which is also variable. Many local health departments may inspect the business sites, but it is questionable whether inspectors receive adequate training about the appropriate infection prevention practices they should monitor in these nontraditional settings. Also, the frequency by which the sites are inspected is variable. Sometimes it takes an adverse event for a site to be inspected. There are many opportunities for contamination, particularly for tattooing. Many of the practices being adopted by piercing studios worldwide are based on a common sense approach, as well as guidelines offered by professional organizations such as the Association of Professional Piercers or the Piercing Association of the United Kingdom. Reference: APIC Text, 4th edition, Chapter 123 - Body Piercing, Tattoos, and Electrolysis CBIC Core Competency: Preventing/Controlling the Transmission of Infectious Agents

In a case-control study of the association between exposure to long-term nursing care and the outcome of VRE colonization, the authors report an odds ratio of 2.3. How should this odds ratio be interpreted? a. The study is flawed because a relative risk (RR) should have been calculated b. The risk of being placed in long-term nursing care is 2.3 times higher for people with VRE than without VRE c. VRE cases have 2.3 times the odds of having had exposure to long-term nursing care than non-VRE cases d. There is no association between VRE colonization and exposure to long-term nursing care

c. VRE cases have 2.3 times the odds of having had exposure to long-term nursing care than non-VRE cases Rationale: A case-control study groups participants by their disease status (in this example, VRE-colonized cases or VRE-noncolonized controls) and looks retrospectively to determine whether they had an exposure of interest. The odds ratio that is calculated from a case-control study is the odds of the cases having had the exposure divided by the odds of the controls having had the exposure. Although this is often interpreted the same way as an RR, a case-control study does not give information about risk because incidence rates are not calculated in this study design. The odds ratio is not the risk of developing the disease given the exposure, but rather the ratio of the odds of having had the exposure given the disease status. Reference: APIC Text, 4th edition, Chapter 13 - Use of Statistics in Infection Prevention CBIC Core Competency: Education and Research

A patient is admitted with watery diarrhea and abdominal cramps. The food history interview reveals that the patient consumed raw oysters recently. Which of the following organisms is the most likely cause? a. Yersinia b. Clostridium difficile c. Vibrio d. Campylobacter

c. Vibrio Rationale: Vibrio gastroenteritis is caused by eating undercooked or raw seafood, such as fish and shellfish. Reference: APIC Text, 4th edition, Chapter 83 - Foodborne Illnesses CBIC Core Competency: Identification of Infectious Disease Processes

immunoglobulins are available for which of the following? 1) Tetanus 2) Rubella 3) Pertussis 4) Varicella-zoster a. 1, 2, 3 b. 2, 3, 4 c. 1, 3, 4 d. 1, 2, 4

d. 1, 2, 4 Rationale: Immunoglobulins are special concentrated antibody preparations that provide immediate short-term protection against disease for individuals who are at high risk of experiencing severe disease or of developing serious complications from the disease. Human normal immunoglobulin preparations for Hepatitis A, measles, polio, and rubella, and specific immunoglobulin preparations for Hepatitis B, rabies, and varicella-zoster for intramuscular use, are available. References: APIC Text, 4th edition, Chapter 100 - Occupational Health; Centers for Disease Control and Prevention (CDC). Vaccines and Immunizations. CDC website. 2010. Available at: http://www.cdc.gov/vaccines/ CBIC Core Competency: Employee/Occupational Health

You are presenting an in-service to the Environmental Services group in your facility. Which of the following recommendations should you make related to mopping of patient rooms? 1) Standard mop heads should be disinfected with a disinfectant weekly 2) Floors should be mopped using the "S" stroke 3) Mop water should be changed after two hours of use. 4) The mops should never be redipped into the mopping solution. a. 1, 2 b. 2, 3 c. 3, 4 d. 2, 4

d. 2, 4 Rationale: Cleaning of floors and carpets should start at the back of the room and move toward the door. Floors should be mopped using the "S" stroke, catching dirt and debris on the leading edge of the mop. Cleaning items (e.g., detergent and/or disinfectant solutions, water buckets, cleaning cloths, dusters, toilet brushes, and mops) must be changed routinely and after they are used to clean blood spills or highly contaminated areas such as isolation rooms or operating rooms. Using contaminated cloths and mops will result in cross- contamination of surfaces, equipment, and hands. CDC recommends changing floor mopping solutions every three rooms and at least every 60 minutes. Used mops and cleaning cloths should never be returned to containers of cleaning solution. They should be laundered or discarded after use. Reference:APIC Text, 4th edition, Chapter 107 - Environmental Services CBIC Core Competency: Cleaning, Sterilization, Disinfection, Asepsis

The existence of an outbreak is most often determined by: a. The identification of more than 10 new cases per week b. An incidence rate that is more than two standard deviations higher than the previous year c. Detection of a cluster of organisms within more than one population. d. An incidence of disease that is clearly in excess of that expected

d. An incidence of disease that is clearly in excess of that expected Rationale: Outbreaks in healthcare should be suspected when HAIs or adverse events occur above the background rate or when an unusual microbe or adverse event is recognized. Healthcare-associated outbreaks often have multiple causes, but almost all are due to one or more of the following: - Lapses in infection prevention or clinical practices - Colonization or infection of HCP, - Defects in or contamination of a product or device, either at the time of production (intrinsic contamination) or during use (extrinsic contamination). Outbreaks in healthcare settings may also be caused by visitors who have, or are harboring, an infectious disease (e.g., influenza or chickenpox). Reference: APIC Text, 4th edition, Chapter 11 - Surveillance CBIC Core Competency: Surveillance and Epidemiologic Investigation

All of the following may be indications of heating, ventilation, and air conditioning (HVAC) malfunctions except: a. An increase in the postoperative surgical site infection (SSI) rates b. A single case of aspergillosis in a severely immunosuppressed patient c. Healthcare-associated varicella infections d. An outbreak of Ventilator-associated Acinetobacter infections in the ICU

d. An outbreak of Ventilator-associated Acinetobacter infections in the ICU

A 47-year-old female bus driver is brought to the Emergency Department (ED) with a two-day history of fever, shortness of breath, and chest pain. She is diaphoretic and appears acutely ill. She is confused as to place and time. Temperature is 38°C (100.4°F), blood pressure is 88/60 mm Hg, pulse rate is 110/min, and respiration rate is 28/min. Coarse bronchial breath sounds are heard. She has had no recent known contact with ill persons. The leukocyte count is 15,000/μL (15 × 109/L). A chest radiograph shows a widened mediastinum and bilateral pleural effusions. Gram stain of a peripheral blood smear shows box car-shaped Gram-positive bacilli. A bioterrorism agent is suspected. Which of the following agents is most likely? a. Typhus fever b. Smallpox c. Tularemia d. Anthrax

d. Anthrax Rationale: A biological attack, or bioterrorism, is the intentional release of viruses, bacteria, or other germs that can sicken or kill people, livestock, or crops. Bacillus anthracis, the bacteria that causes anthrax, is one of the most likely agents to be used in a biological attack because: • Anthrax spores are easily found in nature, can be produced in a lab, and can last for a long time in the environment • Anthrax makes a good weapon because it can be released quietly and without anyone knowing. The microscopic spores could be put into powders, sprays, food, and water. Because they are so small, individuals may not be able to see, smell, or taste them. • Anthrax has been used as a weapon before There are three types of anthrax: cutaneous, gastrointestinal, and inhalational. Symptoms of inhalation anthrax include: • Fever and chills • Chest discomfort • Shortness of breath • Confusion or dizziness • Cough • Nausea, vomiting, or stomach pains • Headache • Sweats (often drenching) • Extreme tiredness • Body aches If inhalation anthrax is suspected, chest X-rays or computed tomography scans can confirm if the patient has mediastinal widening or pleural effusion, which are X-ray findings typically seen in patients with inhalation anthrax. The only way to confirm a diagnosis of anthrax is to either test directly for B. anthracis in a sample (blood, skin lesion swab, spinal fluid, or respiratory secretions) or measure antibodies or toxin in blood. Samples must be taken before the patient begins taking antibiotics. Reference: APIC Text, 4th edition, Chapter 120 - Infectious Disease Disasters: Bioterrorism, Emerging Infections, and Pandemics CBIC Core Competency: Preventing/Controlling the Transmission of Infectious Agents

Of the CDC recommendations for air-handling systems in health-care facilities, which of the following is the most highly recommended compared to the other choices? a. Do not use areas with through-the-wall ventilation units as PE rooms. b. Incorporate steam humidifiers, if possible, to reduce potential for microbial proliferation within the system, and avoid use of cool-mist humidifiers c. Remove bird roosts and nests near air intakes to prevent mites and fungal spores from entering the ventilation system d. Conduct an ICRA and provide adequate number of AII and PE rooms to meet the need of the patient population

d. Conduct an ICRA and provide adequate number of AII and PE rooms to meet the need of the patient population There is strong evidence (Category 1A) from epidemiological and clinical studies to indicate the benefit of conducting an ICRA and ensuring that the number of AII and PE rooms meet the needs of the patient populations. All of the other above options are also recommended but are not as strongly supported. Source: HICPAC 2003 Guidelines

Commercial third-party reprocessors are regulated by which government agency? a. Centers for Disease Control and Prevention (CDC) b. Environmental Protection Agency (EPA) c. Center for Medicaid/Medicare Services (CMS) d. Food and Drug Administration (FDA)

d. Food and Drug Administration (FDA) Rationale: When considering reprocessing SUDs, hospitals are faced with the decision of whether to contract with a third-party reprocessor or formulate an in-house plan. No matter the approach, the process must comply with the FDA regulations. Reference: APIC Text, 4th edition, Chapter 32 - Reprocessing Single Use Devices CBIC Core Competency: Cleaning, Sterilization, Disinfection, Asepsis

Between cases in the operating room, the floors must cleaned and disinfected as follows: a. Floors are only cleaned at the end of the day unless organic debris is present b. The entire floor must be flooded and cleaned between each case c. Spot clean the floor only if visible blood/body fluids d. It is only necessary to clean a 3 to 4-foot perimeter around the table unless a wider contamination area is identified

d. It is only necessary to clean a 3 to 4 foot perimeter around the table, unless a wider contamination area is identified Rationale: Floors in the operating room must be cleaned and disinfected after each case. Reusable string or microfiber mops may be used in between cases and should be changed after each use. If a cotton mop and bucket system is used, a clean mop head and fresh disinfectant must be used for each case. It is only necessary to clean a 3 to 4 foot perimeter around the operative table after each case unless a wider perimeter of contamination is identified. Reference: APIC Text, 4th edition, Chapter 107 - Environmental Services CBIC Core Competency: Cleaning, Sterilization, Disinfection, Asepsis

Which of the following is not an example of syndromic surveillance? a. Number of patients seen in the Emergency Room with influenza-like illness b. Number of purchases of over-the-county flu remedy medications c. Number of school absences related to illness d. Number of new cancer diagnoses reported to the cancer registry

d. Number of new cancer diagnoses reported to the cancer registry Rationale: Syndromic surveillance now includes any indicator that might signal an increase in illness in the community. Some examples of data that could be collected and analyzed as part of a syndromic surveillance program include 1) number of patients seen in an emergency department; 2) number of patients presenting to the emergency department with influenza-like illness as their chief symptom; 3) number of patients admitted to a hospital; 4) number of emergency medical services or ambulance runs performed each day, week, month, or other time period; 5) number of purchases of over-the-counter flu remedies; 6) number of purchases of over-the-counter diarrhea medications; or 7) other data available from healthcare facilities or agencies that may indicate a change or trend in the community. Reference: APIC Text, 4th edition, Chapter 117 - Public Health CBIC Core Competency: Surveillance and Epidemiologic Investigation

A patient in your facility has an infection with Cryptosporidium parvum. Which disinfectant would you recommend for use on potentially contaminated patient equipment? a. Ethyl alcohol b. Six percent hypochlorite c. Ortho-phthalaldehyde d. Six percent hydrogen peroxide

d. Six percent hydrogen peroxide Rationale: Cryptosporidium parvum are protozoa that are resistant to many disinfectants, including chlorine. The only disinfectant with known effectiveness against C. parvum at working concentrations is hydrogen peroxide. Reference: CDC Guideline for Disinfection and Sterilization in Healthcare Facilities, 2008, Page 23 CBIC Core Competency: Cleaning, Sterilization, Disinfection, Asepsis

An adult patient is admitted through the Emergency Department with a 1-day history of rash, fever, and cough. The attending physician is concerned that the patient has measles, as the patient cannot recall whether he has had measles disease or vaccine in the past. The serum sample does not contain measles-specific IgG or IgM antibodies. What is the significance of this finding? a. The patient does not have measles infection b. The patient has had the measles vaccine in the past c. The patient has a history of measles disease d. The patient might be infected with measles

d. The patient might be infected with measles Rationale: If the patient had been vaccinated or had a history of measles disease, his serum sample would most likely have contained measles-specific IgG antibody. We cannot assume that the patient does not have measles infection at only 24 hours after the onset of rash, as there might not have been enough time yet for the immune response to develop. It can take up to 72 hours after the onset of rash for IgM to be produced in response to measles infection, so this patient might be infected. Reference: APIC Text, 4th edition, Chapter 86 - Measles, Mumps, Rubella CBIC Core Competency: Identification of Infectious Disease Processes

A patient in your facility has been diagnosed with naturally-acquired inhalational Anthrax, and Environmental Services need to know how to terminally clean the patient's room. Which protocol should they follow? a. They should clean the room with a protocol similar to the one used for patients with C. difficile b. They should only use an EPA-registered disinfectant with proven effectiveness against Anthrax c. The should call the Centers for Disease Control and Prevention to come clean the room d. They should use the standard terminal cleaning protocol

d. They should use the standard terminal cleaning protocol Rationale: Anthrax is an illness caused by Bacillus anthracis, a spore-forming bacterium. Infection with Bacillus anthracis can be acquired through exposure to contaminated soil, water, or animals, including imported animal skin drums. It can also be acquired as a consequence of a bioterrorism event, where the spore form of the agent might be released into the environment. In the case of a naturally-acquired case of anthrax, there is no person-to-person transmission of the agent and no environmental contamination of the patient room with the spores. Therefore, no special cleaning protocol is needed in this case. However, a bioterrorism event with release of spores in the facility would warrant higher level decontamination and would warrant higher level decontamination and cleaning. Reference: CDC 2007 Guideline for Isolation Precautions: Preventing Transmission of Infectious Agents in Healthcare Settings, Appendix A CBIC Core Competency: Environment of Care

Objectives

describe each task or behavior the learner will be able to demonstrate after the completion of the course, as well as the conditions under which each task or behavior will be performed.


Ensembles d'études connexes

Chapter 51 Biol 2130 Population Ecology

View Set

Unit 2 Test: Selected and Short Response (Online) Copy 1

View Set

Unit 12 - Zakupy i usługi - Zwroty (Język angielski. Repetytorium dla szkoły podstawowej - część 2)

View Set

Ch 8 APEC 1101H Donald Liu UMN Fall 2015

View Set